Você está na página 1de 199

1B

Chow Wai Keung


General Editor: Ann Lui Yin Leng
Consultant: Prof Ling San
FULLY
WORKED
SOLUTIONS
9 Ratio, Rate and Speed

Class Activity 1
Objective: To discuss and explain how ratios are used in everyday life.

Questions
1. There are some 500 ml cans of red paint and white paint. When n can of red paint is mixed with m cans of white paint
to give pink paint, we say that the mixing ratio of red paint and white paint is n : m.
(a) If the mixing ratio of red paint to white paint is 1 : 3, what is the fraction of red paint in the mixture?

Fraction of red paint in mixture

1
=
1+3
1
=
4

(b) Paint P is obtained by mixing 2 cans of red paint to 4 cans of white paint. Paint Q is obtained by mixing 3 cans of
red paint to 9 cans of white paint.
(i) Calculate the mixing ratio of red paint to white paint in paints P and Q.

Paint P’s mixing ratio = 2 : 4


=1:2

Paint Q’s mixing ratio = 3 : 9


=1:3

(ii) 
Compare the fractions of red paint in paints P and Q. Which paint will result in a lighter pink? Explain your
answer.
1
Fraction of red paint in paint P =
3
1
Fraction of red paint in paint Q =
4
1 1
Since < , paint Q will result in a lighter pink.
4 3

2. The diagram shows a gear system. The rotation of the driving wheel produces the rotation of the
driven wheel through the interlocking teeth. The ratio of the number of teeth on the driven wheel
to that of the driving wheel is called the gear ratio of the system.
(a) If the number of teeth on the driven wheel is 8 and the number of teeth on the driving wheel
is 16, driven
(i) what is the gear ratio? wheel

The gear ratio driving


= No. of teeth on the driven wheel : No. of teeth on the driving wheel wheel
= 8 : 16
=1:2

33
(ii) 
what would be the number of revolution of the driven wheel when the driving wheel rotates one revolution?
Explain your answer.

When the driving wheel rotates 1 revolution, 16 teeth contacts with the driven wheel, the number of revolutions of the driven wheel
= 16 ÷ 8
=2

(b) A mechanic wants to have a gear system such that the driven wheel rotates 4 times when the driving wheel rotates
once. How would you design for him? Discuss your design with your peers.

Consider a driven wheel with 8 teeth. Since the driven wheel rotates 4 times when the driving wheel rotates once, the number of teeth on the
driving wheel
=8×4
= 32

(c) Find something with 2 gears in real life and determine its gear ratio.
Note: The concept of gear ratio is used in making the transmission system of motor cars and bicycles.

A fixed gear bike uses 2 gears. For a fixed gear bike with 45 teeth on the driving wheel and 15 teeth on its driven wheel, the gear ratio
45
=
15
=3

3. Some scientists have reported that tongue rolling ability (able to hold tongue in a tongue roller
U shape) may be due to genes.
(a) Try to find the number of tongue rollers and the number of non-tongue rollers
in your class.
(b) What is the genetic ratio of the number of tongue rollers to the number of
non-tongue rollers in your class?
(c) If your school has 1200 students, what is the estimated number of tongue
rollers in your school?
Note: Genetic ratio is used in breeding plants and animals. non-tongue roller

The answer depends on the actual data collected.

4. The aspect ratio of an image is the ratio of the width to the height of the image.
width
The designs of televisions, monitor screens and cinema screens are based on certain
aspect ratios. The popular aspect ratios are 16 : 9 and 4 : 3.
(a) Measure the widths and heights of your television screen and computer screen
height
at home. Calculate the aspect ratios of your television and computer screens.
(b) Compare your calculated aspect ratios with your classmates. What do you
realise?

The answer depends on the actual data collected.

Chapter 9  Ratio, Rate and Speed


34
Class Activity 2
Objective: To discuss some examples of rate used in daily life.

Questions
1. An exchange rate is the current market price for which one currency can be exchanged for another. On 5 July 2011, 500
US dollars (USD) could be exchanged for 613.30 Singapore dollars (SGD).
(a) Express the exchange rate in number of Singapore dollars to one US dollar.

The exchange rate = 613.20 SGD ÷ 500 USD


= 1.2264 SGD/USD

(b) The price of a book in an online bookstore is USD 40. If you bought this book on 5 July 2011, how much would
you have to pay in terms of Singapore dollars? Show your workings.

Price of book in Singapore dollars


= 1.2264 × 40
= 49.06

The book costs 49.06 SGD.

(c) Describe some occasions where the use of exchange rate is necessary.

Buying or selling a product that is in a different currency from what you use requires using an exchange rate to convert the cost of the product from

a different currency to the currency you use.

2. When you deposit money in a bank, the bank will pay you a fee, called interest, for the use of money you have deposited.
The interest in one year expressed as a percentage of the deposit is called annual interest rate. The amount you would
receive after a year depends on the annual interest rate. If a sum of $4000 deposited in a bank for 1 year can earn $120
interest,

120
the annual interest rate = × 100% = 3%.
4000

(a) If you deposit $6000 in a bank for 1 year at the annual interest rate of 3%, how much interest you will get? Show
your working clearly.

Interest
= 3% of $6000

3
=$ × 6000
100

= $180

(b) If the annual interest rate is 4% and you would like to earn $200 interest in a year, how much should you deposit?
Show your working.

Let $x be the deposit.

4% of $x = $200
4x
= 200
100
The deposit required is $5000.

35
(c) Do a research and get some interest rates for the deposit in a bank. If you were an employee of the bank, how would
you explain to your clients the different deposit options available based on their interest rates?

The answer depends on the actual data collected.

3. If you earn income in Singapore over a certain amount, you have to pay the income tax to the government in each year.
The income tax is calculated based on a tax rate table. The table shown below is part of the tax rate table for the year of
assessment 2012 for the total income in 2011.

Chargeable Income Rate (%) Gross Tax Payable ($)


First $20,000 0 0
Next $10,000 2 200
First $30,000 – 200
Next $10,000 3.50 350
First $40,000 – 550
Next $40,000 7 2800
First $80,000 – 3350
Next $40,000 11.5 4600
Source: Inland Revenue Authority of Singapore

Note that the rates in the table may vary in each year, depending on the current economic conditions. The chargeable
income is the total income minus allowable deductions, approved donation and personal reliefs.

The table on the previous page shows that if your annual chargeable income is not more than $20 000, you do not need
to pay the income tax.

If a person’s chargeable income in 2011 was $53 000,
then for the first $40 000, the gross tax payable = $550,
for the next $13 000, the tax rate = 7%,
the gross tax payable = $13 000 × 7%
= $910,
\  the income tax = $550 + $910
= $1460.

(a) The chargeable income of Ali in 2011 was $68 000. Calculate
(i) his income tax,

For the first $40 000, the gross tax payable = $550,
for the next $28 000, the tax rate = 7%,
the gross tax payable = $28 000 × 7%
= $1960
the income tax = $550 + $1960
= $2510.

(ii) 
the ratio of his income tax to his chargeable income, giving your answer in the form 1 : r, where r is correct
to 3 significant figures.

Income tax : Chargeable income


= 2 510 : 68 000
= 1 : 27 (correct to 3 s.f.)

Chapter 9  Ratio, Rate and Speed


36
(b) The chargeable income of Donna in 2011 was $36 000. Calculate
(i) her income tax,

For the first $30 000, the gross tax payable = $200,
for the next $6 000, the tax rate = 3.5%,
the gross tax payable = $6 000
= $210
the income tax = $200 + $210
= $410.

(ii) 
the ratio of her income tax to her chargeable income, giving your answer in the form 1 : r, where r is correct
to 3 significant figures.

Income tax : Chargeable income


= 410 : 36 000
= 1 : 88 (correct to 3 s.f.)

4. The speed of an object is its distance covered per unit time.


(a) Search the world record time of running. Find the speed of the record holder in m/s.

Usain Bolt holds the world record time of 9.58s for 100m.
Speed of record holder
100
= m/s
9.58
= 10.4 m/s (correct to 3 s.f.)

(b) The speed of a car is 60 km/h. How would you find its time taken to travel 40 km?

Time taken to travel 40 km


distance
=
speed
40
= h
60

1
= h
2

(c) If the speed of the car in (b) increases, would its travelling time taken to travel a distance of 40 km increase or
decrease? Explain your answer.

The distance remains the same. Since the speed of an object is its distance covered per unit time, an increase in speed means that the object

covers a larger distance per unit time. Hence the time taken will decrease.

5. The rate of rotation of a wheel is usually expressed as the number of rotations per minute (rpm).
(a) A wheel P makes 252 rotations in 9 minutes. Find its rate of rotation.

The rate of rotation of Wheel P = 252 ÷ 9 rpm


= 28 rpm

(b) Another wheel Q makes 270 rotations in 10 minutes. Determine which wheel spins faster? Explain your answer.

The rate of rotation of Wheel Q = 270 ÷ 10 rpm


= 27 rpm

Wheel P spins faster since 28 > 27.


37
(c) Discuss how the rate of rotation of the wheels of a bicycle affects the speed of the bicycle.

• One revolution covers a distance of C, the circumference of the wheel where C = 2�r.

• As the rate of rotation increases, the number of revolutions increases, covering a larger distance per unit time, thereby increasing the speed of

the bicycle.

• Similarly, as the rate of rotation decreases, the number of revolutions decreases, covering a smaller distance per unit time, thereby decreasing

the speed of the bicycle.

6. Find out some other examples of the use of rates in our daily life.

Some other examples of the use of rate in our daily life include speed (e.g. 50 km/h), hourly wage rate (e.g. $7/h) and typing speed (e.g. 100 words

per minute).

Class Activity 3
Objective: To find and compare the speeds of bicycles, cars, trains, aeroplanes and spaceships.

Questions
1. Have you ever wondered how fast you walk? How can we find out our own walking speed?
distance
Since speed = , we can measure our own walking speed by walking for a particular distance and taking note of the time.
time

2. In the Cycle Singapore 2011 Super Challenge 60 km race, the record time for the men’s event was 1 hour 15 minutes and
38.4 seconds.

(a) Find the speed of the champion’s bicycle.


15 38.4
The speed of the champion’s bicycle = 60 ÷ 1 + +
60 3600

= 47.59
= 47.6 km/h  (correct to 3 s.f.)

(b) Based on your NAPFA Test 2.4 km run, calculate your running speed in km/h.

The answer depends on the actual data collected.

(c) If you keep running at the speed in (b), how long would you take to run a distance of 60 km?

The answer depends on the actual data collected.

Chapter 9  Ratio, Rate and Speed


38
3. Singapore Formula 1 Grand Prix 2010 has a circuit of 5.073 km long. The champion car finished 61 laps in 1 hour 57
minutes and 53.579 seconds.
(a) Find the speed of the car.
57 53.579
The speed of the car = 5.073 × 61 ÷ 1 + +
60 3600

= 157 km/h  (correct to 3 s.f.)



(b) What is the general speed limit of a car along the expressway in Singapore?

The speed limit is 90km/h.

(c) If you complete the laps by driving a car at the speed in (b) above, how long would you take?

Time taken

5.073 × 61
=
90

= 3 hours 26 minutes 18.12 seconds

(d) In general, how much faster, in km/h, is the speed of a racing car compared to that of a normal car travelling along
the expressway?

157 – 90 = 67

In general, the racing car travels 67 km more in an hour compared to a car travelling along the expressway.

4. The Beijing-Shanghai High-speed Railway is a railway line connecting the cities Beijing and Shanghai in China. A train
can finish the journey in 4 hours.

(a) (i) What information do you need in order to find the speed of the train?

We need to find the distance which is 1318 km.

(ii) Hence, find the speed of the train.

The speed of the train = 1318 ÷ 4


= 330 km/h  (correct to 3 s.f.)

(b) How can you estimate the speed of our MRT train for travelling between two stations? Explain briefly and find your
estimated value together with your classmates.

The speed of the MRT is 85 km/h.

(c) If an MRT train travels along the above railway from Beijing to Shanghai, how long would it take to arrive at the
destination? How much time can you save when travelling in high-speed train instead?

Time taken = 1318 ÷ 85


= 15.5 hours  (correct to 3 s.f.)

15.5 – 4 = 11.5 hours

Travelling in the high-speed train instead of the MRT saves 11.5 hours.

39
5. The flight distance from Beijing to Shanghai is 1070 km.

(a) Given that the flight time of an aeroplane flying from Beijing to Shanghai is 2 hours 15 minutes, find the speed of
the aeroplane.
40
The speed of the aeroplane = 2570 ÷ 3 +
60

= 700.9
= 701 km/h  (correct to 3 s.f.)

(b) If you have to travel from Beijing to Shanghai, would you take a high-speed train or an aeroplane? Explain your
choice.

The answer depends on the choice made.

6. The average radius of the Earth is 6371 km. A space shuttle flew in a circular orbit at an altitude of 242 km above the
earth surface. It took 90 minutes for the space shuttle to complete one full orbit.

(a) Find the speed of the space shuttle.


90
The speed of the space shuttle = 2� (6371+ 242) ÷
60

= 27 700 km/h  (correct to 3 s.f.)



(b) If an aeroplane could fly around the orbit at the speed obtained in Question 5(a), how long would it take to complete
the orbit?

Time taken = 2� (6371 + 242) ÷ 330


= 126 hours

Chapter 9  Ratio, Rate and Speed


40
Try It! 4. Simplify the following ratios.
4 2 1
Section 9.1 (a) : :1 (b) 1.8 : 5.4 : 0.9
5 3 4
1. Simplify the following ratios. Solution
1 1 4
(a) 1 : (b) 1 : 4 4 2 1
2 3 7 (a) : :1
5 3 4
(c) 0.360 : 0.153 4 2 5
= : : LCM of 3, 4 and 5 is 60.
5 3 4
Solution 4 2 5
1 1 = 60 × : 60 × : 60 ×
(a) 1 : = 1 × 2 : ×2 5 3 4
2 2 = 48 : 40 : 75
=2:1
(b) 1.8 : 54 : 9
1 4 4 32 = (10 × 1.8) : (10 × 5.4) : (10 × 0.9)
(b) 1 : 4 = :
3 7 3 7 = 18 : 54 : 9 HCF of 9, 18 and 54 is 9.
4 21 32 21 18 54 9
= × : × = : :
3 4 7 4 9 9 9
= 7 : 24 =2:6:1
(c) 0.360 : 0.153 = 360 : 153
360 153 5. If a : b = 7 : 10 and b : c = 15 : 8, find a : b : c.
= :
9 9
= 40 : 17 Solution
LCM of 10 and 15 is 30.
a : b = 7 : 10
2. The masses of 2 bags of sugar, A and B, are 750 g and = 7 × 3 : 10 × 3
1 = 21 : 30
1 kg respectively. Find the ratio of the mass of B to
3 b : c = 15 : 8
the mass of A.
= 15 × 2 : 8 × 2
= 30 : 16
Solution
1 750 ∴  a : b : c = 21 : 30 : 16
Mass of B : Mass of A = 1 kg : kg
3 1000
4 3 6. A fast food shop offers coffee in 3 sizes – small, medium
= kg : kg and large. The volume of a small cup to that of a medium
3 4
= 12 ×
4
: 12 ×
3 cup is 5 : 8. The volume of a medium cup to that of a
3 4 large cup is 10 : 13. Find the ratio of the volume of a
= 16 : 9 small cup to that of a medium cup to that of a large cup.

3. The ratio of the number of votes for Alan to the number Solution
of votes for Betty in an election is 3 : 5. LCM of 8 and 10 is 40.
(a) Express the number of votes for Alan as a fraction Volume of small cup : Volume of medium cup
of the number of votes for Betty. = 5 : 8
(b) If Betty got 60 votes, find the number of votes for = 5 × 5 : 8 × 5
Alan. = 25 : 40
Volume of medium cup : Volume of large cup
Solution = 10 : 13
number of votes for Alan 3 = 10 × 4 : 13 × 4
(a) =
number of votes for Betty 5 = 40 : 52

(b) Let 3x and 5x be the number of votes for Alan and Volume of : Volume of : Volume of
Betty respectively. small cup medium cup large cup
Then 5x = 60 = 25 : 40 : 52
x = 12
3x = 36
Alan received 36 votes.

41
7. White paint and red paint are mixed in the ratio 5 : 3 to 11. The number of $1 coins and the number of 50-cent coins
form pink paint. Find the volume of white paint needed in a purse are in the ratio 2 : 1. If 3 of the $1 coins are
to make 4 litres of such pink paint. exchanged for 50-cent coins, the ratio becomes 1 : 2.
Find the total value of the coins.
Solution
Volume of white paint : Volume of red paint = 5 : 3 Solution
Fraction of the volume of white paint in pink paint Let 2x and x be the number of $1 coins and 50-cent coins
5 respectively.
=
5+3 Number of $1 coins after exchanging = 2x – 3
5 Number of 50-cents coins after exchanging = x + 6
=
8 2x – 3 1
5 =
\  volume of white paint in pink paint = ×4 x+6 2
8
2(2x – 3) = x + 6
= 2.5 litres
4x – 6 = x + 6
3x = 12
8. The volumes of cup P and of cup Q are in the ratio 2 : 3. x = 4
If cup Q can hold 120 cm3 of water more than cup P, \  total value of the coins = $1(2 × 4) + $0.50(1 × 4)
find the volume of cup Q. = $10

Solution
Let 2x be the volume of cup P and 3x be the volume of Section 9.2
cup Q respectively. 12. Find the rate in each of the following.
Then 3x – 2x = 120 (a) Joe ran 100 m in 12.5 seconds. Find his rate of running
x = 120 in m/s.
3x = 360 (b) 375 ml of apple juice cost $6. Find the price rate
The volume of cup Q is 360 cm3. in $/l.

Solution
9. Three boys, X, Y and Z, share 132 copies of computer 100 m
games. The ratio of X’s share to Y’s share is 2 : 3. The (a) Joe’s rate of running =
12.5 s
ratio of Y’s share to Z’s share is 1 : 2. How many copies = 8 m/s
of computer games does each boy get?
$6
(b) Price rate of juice =
Solution 0.357l
X’s share : Y’s share = 2 : 3 = $16/l
Y’s share : Z’s share = 1 : 2 = 3 : 6
∴  X’s share : Y’s share : Z’s share = 2 : 3 : 6
13. On 5 July 2011, 3600 Chinese Yuan (CNY) could be
2
Number of copies that X gets = 132 × exchanged for 683.28 Singapore dollars (SGD).
2+3+6
= 24 (a) Find the exchange rate between the Chinese Yuan
3 and the Singapore dollars in
Number of copies that Y gets = 132 ×
2+3+6 (i) SGD/CNY, (ii) CNY/SGD.
= 36 (b) A Chinese tourist spent 4500 SGD at the Sentosa
6 Resort. Find his expenditure in CNY.
Number of copies that Z gets = 132 ×
2+3+6
Give your answers correct to 4 significant figures.
= 72
Solution
10. The prices of two stocks P and Q are in the ratio of 5 : 7. (a) (i) 3600 CNY = 683.28 SGD
When the price of P is increased by $12 and the price of 683.28
1 CNY = SGD
Q is decreased by $6, the prices of two stocks become 3600
the same. Find the original price of stock P. = 0.1898 SGD
The required exchange rate is 0.1898
Solution SGD/CNY.
Let $5x and $7x be the price of stocks P and Q 3600
respectively. (ii) 1 SGD = CNY
683.28
5x + 12 = 7x – 6 = 5.2687 CNY
2x = 18 = 5.269 CNY  (correct to 4 s.f.)
x = 9 The required exchange rate is 5.269
5x = 45 CNY/SGD.
Stock P costs $45.

Chapter 9  Ratio, Rate and Speed


42
(b) His expenditure 18. The walking speed of Ali is 0.8 m/s.
= 4500 SGD (a) Find the distance he can walk in one minute.
= 4500 × 5.2687 CNY (b) Find his time taken to walk 20 m.
= 23 710 CNY  (correct to 4 s.f.)
Solution
(a) Distance travelled in one minute
14. Mrs Lim borrows $84 000 from a bank for 2 years. The
= 0.8 × 60
interest rate is 5% per annum. Find the total interest she
= 48 m
has to pay to the bank if she pays the interest at the end
Ali can walk 48 m in one minute.
of each year.
(b) Time taken to walk 20 m
Solution 20
Total interest = $84000 × 5% × 2 =
8.0
= $8400 = 25 s
Ali takes 25 seconds to walk 20 metres.
15. A worker is paid $152 for 8 hours of work.
(a) Find the hourly wage rate. 1
19. The average speed of a car for the first 1 hours of a
2
(b) If the worker works for 5 hours, find his wage. 1
journey is 64 km/h. Its average speed for the next 2
2
Solution hours is 72 km/h. Find
$152 (a) the average of the speeds 64 km/h and 72 km/h,
(a) Hourly wage rate =
8h (b) its average speed for the whole journey.
= $19/h
Solution
(b) His wage = $19 × 5 64 + 72
= $95 (a) Arerage speed =
2
= 68 km/h
16. Susan’s rate of reading a story book is three times as 1 1
fast as that of Cai. Given that Susan can read 60 more (b) Total distance travelled = 64 × 1 + 72 × 2
2 2
pages than Cai in 5 hours, find the reading rate of Cai
= 276 km
in pages per hour.
1 1
Total time taken = 1 + 2
Solution 2 2
Let x and 3x be Cai’s and Susan’s rate of reading in =4h
pages/hour respectively.
Number of pages Susan read in 5 hours = 5(3x) Its average speed for the whole journey
= 15x =
276
Number of pages Cai read in 5 hours = 5x 4
15x – 5x = 60 = 69 km/h
10x = 60
x = 6 20. Towns X and Y are 108 km apart. Sulin drives at an
Cai rate of reading is 6 pages/hour. average speed of 72 km/h from X to Y and returns from
Y to X at an average speed of 54 km/h. Find her average
Section 9.3 speed for the whole trip.
17. (a) A trolley travels 6 m in 4 seconds. Find its speed Solution
in m/s.
Total distance travelled = 108 + 108
(b) A snail crawls 4 cm in 10 seconds. Find its speed
= 216 km
in cm/s.
108 108
Solution Total time taken = +
72 54
(a) Speed of trolley
= 3.5 h
6m
=
4s
216
= 1.5 m/s Her average speed for the whole journey =
3.5
(b) Speed of snail 5
= 61 km/h
4 cm 7
=
10 s
= 0.4 cm/s

43
21. Cars A and B leave the same place and travel in the same
direction along a straight road. Car A travels at 60 km/h
and car B travels at 72 km/h. After how long will they
be 8 km apart?

Solution
Let t hours be the required time such that the cars are
8 km apart.
Distance travelled by car A in t hours = 60t km
Distance travelled by car B in t hours = 72t km
From the given information,
72t – 60t = 8
12t = 8
2
t =
3
2
The cars will be 8 km apart after hours.
3

22. The speed limit for a road is 60 km/h. Express this speed
limit in m/s.

Solution
The speed limit = 60 km/h
= 60 × 1000 m/h
1000
= 60 × m/s
3600
2
= 16 m/s
3

1
23. A man walks at a speed of 1 m/s. Express this speed
9
in km/h.

Solution
1
His walking speed = 1 m/s
9
10 1
= × km/s
9 1000
10 3600
= × km/h
9 1000

= 4 km/h

Chapter 9  Ratio, Rate and Speed


44
Exercise 9.1 (b)
3 2
a = b
4 5
Level 1
3 2
1. Express each of the following ratios in the simplest form. 20 × a = 20 × b
4 5
(a) 18 : 27 (b) 144 : 132
1 1 2 1
15a = 8b
(c) 1 : 4 (d) 2 : 1
2 2 3 5 15a 8b
2 =
(e) 0.250 : 0.375 (f) 0.48 : 2 15 15
15
8b
(g) 1.6 m : 36 cm (h) 850 g : 3.4 kg a =
15
a 8b
Solution =
b 15b
18 27
(a) 18 : 27 = : a 8
9 9 =
b 15
=2:3
\  a : b = 8 : 15
144 132
(b) 144 : 132 = :
12 12
= 12 : 11 3. Mary is 32 years old and her daughter is 4 years old.
Find the ratio of Mary’s age to her daughter’s age.
1 1 3 9
(c) 1 : 4 = :
2 2 2 2
Solution
=1:3 Mary’s age : Daughter’s age
2 1 8 6 = 32 : 4
(d) 2 : 1 = :
3 5 3 5 = 8 : 1
8 6
= × 15 : × 15
3 5
4. Due to genetics, a person may have either free earlobes
= 20 : 9
or attached earlobes. In a class of 40 students, there are
(e) 0.250 : 0.375 = 250 : 375 24 students with attached earlobes. Find the ratio of
250 375 (a) the number of students with attached earlobes to
= :
125 125 the number of students in the class,
=2:3 (b) the number of students with attached earlobes to
2 48 32 the number of students with free earlobes.
(f ) 0.48 : 2 = :
15 100 15
12 32 Solution
= : (a) Number of students Number of students
25 15
:
3 8 with attached earlobes in the class
= :
25 15 = 24 : 40
3 8
= × 75 : × 75 =3:5
25 15
= 9 : 40 (b) Number of students Number of students
:
with attached earlobes with free earlobes
(g) 1.6 m : 36 cm = 160 cm : 36 cm
= 40 : 9 = 24 : (40 – 24)
= 24 : 16
(h) 850 g : 3.4 kg = 850 g : 3400 g
= 3 : 2
=1:4

5. Simplify each of the following ratios.


2. Find a : b in each of the following.
1 1 1
3 2 (a) a : b : c = 1 : :2
(a) 2a = 3b (b) a = b 3 2 4
4 5
(b) x : y : z = 2.7 : 6 : 1.8
Solution
(a) 2a = 3b Solution
2a 3b 1 1 1
= (a) a : b : c = 1 : :2
2 2 3 2 4
3b 4 1 9
a = = : :
2 3 2 4
a 3b 4 1 9
= = 12 × : 12 × : 12 ×
b 2b 3 2 4
a 3
= = 16 : 6 : 27
b 2
\  a : b = 3 : 2

45
(b) x : y : z = 2.7 : 6 : 1.8 (c) a : b =
1
:1
= 27 : 60 : 18 2
1
27 60 18 b : c = 1 :
= : : 3
3 3 3
1 1
= 9 : 20 : 6 ∴  a : b : c = :1:
2 3
1 1
= ×6:1×6: ×6
2 3
6. Given that a : b : c = 20 : 35 : 15,
=3:6:2
(a) simplify a : b : c,
(b) find a : b, (d) a : b = 3 : 7
(c) find c : b. = 9 : 21
b : c = 3 : 7
Solution = 21 : 49
(a) a : b : c = 20 : 35 : 15 ∴  a : b : c = 9 : 21 : 49
20 35 15
= : :
5 5 5
=4:7:3 Level 2
(b) a : b = 4 : 7 9. If a = 20 and b = 36, find
(a) a : b,
(c) c : b = 3 : 7 (b) a2 : b2.

7. Given that x : y : z = 5 : 4.62 : 33,


1 Solution
2 (a) a : b = 20 : 36
(a) simplify x : y : z, (b) find y : x, = 5 : 9
(c) find x : z.
(b) a2 : b2
Solution = 202 : 362
1
(a) x : y : z = 5 : 4.62 : 33 = 52 : 92
2
= 25 : 81
11 462
= : : 33
2 100
1 1
1 42 10. If x : y = 0.75 : 0.90 and y : z = : ,
= : :3 3 4
2 100
(a) simplify x : y and y : z,
1 42
= × 50 : × 50 : 3 × 50 (b) find x : y : z.
2 100
= 25 : 21 : 150
Solution
(b) y : x = 21 : 25 (a) x : y = 0.75 : 0.90
= 75 : 90
(c) x : z = 25 : 150
75 90
= 1 : 6 = :
15 15
=5:6
8. Find a : b : c in each case.
1 1
(a) a : b = 3 : 4, b : c = 4 : 9 y : z = :
3 4
(b) a : b = 5 : 3, b : c = 4 : 1
1 1
1 1 = × 12 : × 12
(c) a : b = : 1, b : c = 1 : 3 4
2 3
(d) a : b = 3 : 7, b : c = 3 : 7 =4:3
(b) x : y = 5 : 6
Solution = 10 : 12
(a)   a : b = 3 : 4 y : z = 4 : 3
b : c = 4 : 9 = 12 : 9
∴  a : b : c = 3 : 4 : 9 ∴  x : y : z = 10 : 12 : 9
(b) a : b = 5 : 3
1 2 3 5
= 20 : 12 11. If x : y = 1 : and x : z = 1 : ,
4 3 7 6
b : c = 4 : 1
(a) simplify x : y and x : z,
= 12 : 3
(b) find x : y : z.
∴  a : b : c = 20 : 12 : 3

Chapter 9  Ratio, Rate and Speed


46
Solution 15 . Two sisters share $4200 in the ratio 5 : 7. How much
1 2 money does each sister receive?
(a) x : y = 1 :
4 3
5 2 Solution
= :
4 3 Amount received by one of the sisters
= 15 : 8 5
3 5 = $4200 ×
x : z = 1 : 5+7
7 6
= $1750
10 5
= : Amount received by the other sister
7 6
10 42 5 42 = $4200 – $1750
= × : × = $2450
7 5 6 5
= 12 : 7
(b) x : y = 15 : 8 16. The width and the height of a TV screen are in the ratio
= 60 : 32 16 : 9. If the width is 48 cm, find the height of the screen.
x : z = 12 : 7
= 60 : 35 Solution
∴  x : y : z = 60 : 32 : 35 Height of screen
9
= × 48
16
12. The sides of Square A and Square B are 15 cm and
20 cm respectively. Find the ratio of = 27
(a) their sides, The height of the screen is 27 cm.
(b) their perimeters,
(c) their areas.
Level 3
17. In the Beijing 2008 Olympics Games, the ratio of the
Solution number of gold medals won by China to that won by
(a) Ratio of their sides = 15 : 20 the United States is 17 : 12. China won 15 more gold
=3:4 medals than the United States. Find the number of gold
medals won by the United States.
(b) Ratio of their perimeters = 15 × 4 : 20 × 4
=3:4 Solution
(c) Ratio of their areas = 15 : 20 2 2
Let 17x and 12x be the number of gold medals won by

= 32 : 42 China and the United States respectively.
= 9 : 16 Then 17x – 12x = 15
5x = 15
x = 3
13 . In an alloy, copper and zinc are mixed in the ratio 5 : 3
12x = 36
by mass. If the mass of a block of the alloy is 4.8 kg,
The United States won 36 gold medals.
find the mass of copper in it.

Solution 18. For a concert in Singapore Arts Festival 2012, the price
5 of a Category 1 ticket to that of a Category 2 ticket is
Mass of copper = 4.8 ×
5+3
5 : 4, and the price of a Category 2 ticket to that of a
= 3 kg Category 3 ticket is 2 : 1.
(a) Find the ratio of the price of a Category 1 ticket to
that of a Category 2 ticket to that of a Category 3
14. The numbers of junior and senior staff in a company
ticket.
are in the ratio 2 : 5. If the total number of staff in the
(b) If the price of a Category 3 ticket is $20, find the
company is 28, how many of them are junior staff?
price of a Category 1 ticket.
Solution
Solution
Number of junior staff
(a) Category 1 : Category 2 = 5 : 4
2
= × 28 Category 2 : Category 3 = 2 : 1
7
= 8 =2×2:1×2
There are 8 junior staff in the company. =4:2
Category 1 : Category 2 : Category 3 = 5 : 4 : 2

47
(b) Price of Category 1 ticket Number of students Number of students
20 from School Q : from School R
=$ ×5
2 who participated who participated
= $50 =   9 : 11
= 18 : 22
A category 1 ticket costs $50.
The numbers of students from Schools P, Q, and
R who participated are in the ratio 15 : 18 : 22.
19 . Chicken, potatoes and onions in a recipe are mixed in
Number of students from School P
the ratio 7 : 3 : 2 by mass. If Mrs Krishnan follows the
15
recipe and uses 350 g of chicken, how many grams of = 165 ×
15 + 18 + 22
(a) potatoes,
= 45
(b) onions,
does she use?
(b) Number of students from School Q
18
Solution = 165 ×
15 + 18 + 22
(a) Let x g be the mass of potatoes Mrs Krishnan uses.
= 54
x 3
=
350 7
x = 150 22. The prices of two stocks A and B are in the ratio
She uses 150 g of potatoes. 2 : 3. If the price of A increases by $12 while the price
of B decreases by $6, the ratio of their prices becomes
(b) Let y g be the mass of onions she uses. 10 : 11. Find the original prices of the stocks.
y 2
=
350 7
Solution
y = 100 Let the original prices of Stock A and Stock B be $2x
She uses 100 g of onions. and $3x respectively.
New price of Stock A = $(2x + 12)
20. In a hydrocarbon compound, glucose, the atoms of New price of Stock B = $(3x – 6)
carbon, hydrogen and oxygen are bonded in the ratio 2x + 12 10
1 : 2 : 1. If the total number of atoms in a molecule of =
3x – 6 11
the compound is 24, find the number of 11(2x + 12) = 10(3x – 6)
(a) carbon atoms, 22x + 132 = 30x – 60
(b) hydrogen atoms, 8x = 192
in the molecule. x = 24
2x = 48
Solution 3x = 72
1
(a) Number of carbon atoms = 24 × The original price of Stock A is $48 and the original
1 + 2 +1
price of Stock B is $72.
=6
2
(b) Number of hydrogen atoms = 24 × 23. The total number of English, Mathematics and Science
1 + 2 +1
= 12 books in a book store is 125. The ratio of the number
of English books to that of Mathematics books is 3 : 4.
21. A total of 165 students from schools P, Q and R The number of Science books is 15 less than the number
participated in a National Day Parade. The numbers of of English books. Find the number of English books.
students from schools P and Q who participated are in
Solution
the ratio 5 : 6 and the numbers of students from schools
Q and R who participated are in the ratio 9 : 11. Let 3x and 4x be the numbers of English books and
Find the number of students from Mathematics books respectively.
(a) school P, (b) school Q Number of Science books = 3x – 15
who participated in the National Day Parade. From the given information,
3x + 4x + (3x – 15) = 125
Solution 3x + 4x + 3x – 15 = 125
(a) Number of students Number of students 10x = 140
: x = 14
from School P from School Q
who participated who participated \  Number of English books
=   5 : 6 = 3 × 14
= 15 : 18 = 42

Chapter 9  Ratio, Rate and Speed


48
24. The ratio of the mass of Mary to that of Peter is (a) the original number of 250 g packages,
5 : 7. The mass of Norman is 13 kg more than that of (b) the total mass of the chocolate beans.
Mary, but 9 kg less than that of Peter. Find the mass
of Mary. Solution
(a) Let n and 2n be the numbers of packs of 250 g
Solution and 750 g respectively. Repacking 2 of the 750 g
Let 5x kg and 7x kg be the masses of Mary and Peter packages gives 6 of the 250 g packages.
respectively. n+6 5
=
Mass of Norman = (5x + 13) kg 2n – 2 3
= (7x – 9) kg 3(n + 6) = 5(2n – 2)
3n + 18 = 10n – 10
\  5x + 13 = 7x – 9
10n – 3n = 18 + 10
7x – 5x = 13 + 9
7n = 28
2x = 22
n = 4
x = 11
\  The original number of 250 g packs is 4.
\  Mass of Mary = 5 × 11
= 55 kg (b) Total mass
= 4 × 250 + 2 × 4 × 750
= 7000 g
25. The ratio of the number of technicians to the number of
1 = 7 kg
apprentices in a factory is 7 : 2. If of the apprentices
4
are promoted to technicians, there are 36 more technicians 28. Find three numbers a, b and c such that
than apprentices. Find the original number of technicians. a : b : c = 3 : 4 : 5 and c  5.

Solution Solution
Let 7x and 2x be the original numbers of technicians and If a = 3k, b = 4k and c = 5k, where k ≠ 0,
apprentices respectively. then a : b : c = 3 : 4 : 5.
1 1 When k = 2, a = 6, b = 8 and c = 10.
7x +
× 2x – 1 – × 2x = 36 When k = 3, a = 9, b = 12 and c = 15.
4 4
1 1
7 x – 1 x = 36
2 2
6x = 36 Exercise 9.2
x = 6 Level 1
\  Original number of technicians = 7 × 6 1. Find the rate in each case.
= 42 (a) A 2-litre bottle of peanut oil costs $7.
The price rate is $ /l .
26. The lengths of rods A and B are 120 cm and 96 cm (b) A company produces 360 vases in 5 days.
respectively. After a rod of equal length is cut from each
rod, the ratio of the length of A to that of B becomes The rate of production is vases/day.
10 : 7. Find the length of the rod cut from A. (c) Huili typed 376 words in 8 minutes.
Solution Her rate of typing is words/min.
Let x cm be the length of the rod cut from A. (d) A carpet of area 3 m costs $240. 2

120 – x 10
= The price rate is $  /m2.
96 – x 7
7(120 – x) = 10(96 – x) (e) The mass of a 2.5-m metal bar is 10 kg.
840 – 7x = 960 – 10x
10x – 7x = 960 – 840 The rate of mass is kg/m.
3x = 120
x = 40 Solution
$7
(a) The price rate =
\  The length of the rod cut from A is 40 cm. 2l
= $3.50/l
27. Chocolate beans are packed in 250 g and 750 g packages.
(b) A company produces 360 vases in 5 days.
The numbers of 250 g packages and 750 g are in the
360 vases
ratio of 1 : 2. If 2 of the 750 g packages are repacked Rate of production =
5 days
into 250 g packages, then the ratio becomes 5 : 3. Find = 72 vases/day

49
(c) Huili typed 376 words in 8 minutes. Level 2
376 words
Rate of typing = 5. The cost of laying 12 m2 of floor tiles is $270.
8 min
(a) Find the cost of laying floor tiles per m2.
= 47 words/min (b) Find the cost of laying 20 m2 of floor tiles.
(d) A carpet of area 3 m2 costs $240.
$240 Solution
Price rate = (a) The price of laying floor tiles per m2
3 m2
= $80/m2 $270
=
12
(e) The mass of a 2.5 m metal bar is 10 kg. = $22.50/m2
10 kg
Rate of mass =
2.5 m (b) The price of laying 20 m2 of floor tiles
= 4 kg/m = $22.50 × 20
= $450
(f) The mass of 12 eggs is 720 g.
720 g
Rate of mass = 6. A pipe delivers 9 l of water in 20 seconds.
12 eggs
= 60 g/egg (a) Find the amount of water it delivers per minute.
(b) An empty aquarium tank is 60 cm long, 45 cm wide
and 30 cm high. How long does it take for the pipe
2. Rahman works 5 days a week and 8 hours a day. His to fill the empty tank completely with water?
daily wage is $184. Note: 1 litre = 1000 cm3.
(a) Find his hourly wage rate.
(b) Find his weekly wage rate. Solution
(a) Amount of water delivered per minute
Solution 9
$184 = × 60 l/min
(a) Hourly wage rate = 20
8h
= 27 l/min
= $23/h
(b) Weekly wage rate = $184 × 5/week (b) Volume of the tank = 60 × 45 × 30 cm3
= $920/week = 81 000 cm3
81 000
= l
1000
3. A car travelled 225 km on 25 litres of petrol. The price
= 81 l
of the petrol consumed was $30.
(a) Find the rate of petrol consumption in km/litre. 81
The required time taken =
27
(b) What was the price of petrol per litre?
= 3 min

Solution
225 km
(a) Rate of petrol consumption = 7. A 5-kg pack of ‘Fragrant’ brand rice is sold for $9.60
25 l
and a 10-kg pack of ‘Premium’ brand rice is sold for
= 9 km/l
$18.60.
$30 (a) Find the price per kg of the
(b) The price of petrol per litre =
25 l
(i) ‘Fragrant’ brand rice,
= $1.20/l (ii) ‘Premium’ brand rice.
(b) Which pack of rice is more value for money if the
4. There are 6 wooden cubes, each of side 5 cm. Their total qualities of both brands of rice are the same?
mass is 525 g. Find the mass per
(a) cube, Solution
(b) cm3 of cube. (a) (i) Price per kg of ‘Fragrant’ brand rice
9.60
=$
Solution 5
525 g = $1.92
(a) Mass per cube =
6 cubes (ii) Price per kg of ‘Premium’ brand rice
= 87.5 g/cube 18.60
=$
10
(b) Volume of a cube = 53
= 125 cm3 = $1.86
87.5 g (b) Since $1.86 < $1.92, the ‘Premium’ brand rice is
Mass per cm3 = more value for money.
125 cm2
= 0.7 g/cm3

Chapter 9  Ratio, Rate and Speed


50
8. On a day, 4000 Singapore dollars (SGD) could be Solution
exchanged for 25 300 Hong Kong dollars (HKD). 2
(a) Amount Jenny paid = $504 000 ×
(a) Find the exchange rate between the Singapore 2+3
dollars and the Hong Kong dollars in = $201 600
(i) SGD/HKD, (ii) HKD/SGD. $504 000
(b) Price per m2 =
(b) Janet bought a necklace for 38 000 HKD when she 120 m2
visited Hong Kong. Find the price of the necklace = $4200/m2
in SGD.
Give your answers correct to 4 significant figures.
11. A pure gold cube is of side 3 cm. Its mass is 521.1 g.
Solution (a) Find the mass of pure gold per cm3.
(a) (i) 25 300 HKD = 4000 SGD (b) Another gold cube is of side 2 cm. Its mass is
1 HKD = 4000 ÷ 25 300 SGD 120 g. Is it made of pure gold? Explain briefly
= 0.15 810 SGD Note: The mass of a material per unit volume is called
= 0.1581 SGD  (correct to 4 s.f.) density of the material.
\  The exchange rate is 0.1581 SGD/HKD.
Solution
(ii) 1 SGD = 25 300 ÷ 4000 HKD (a) Mass of pure gold per cm3 =
521.1 g
= 6.325 HKD 33 cm3
\  The exchange rate is 6.325 HKD/SGD. = 19.3 g/cm3
(b) Price of necklace (b) If the gold cube is made of pure gold,
= 38 000 HKD its mass = 19.3 × 23
= 38 000 × 0.1581 SGD = 154.4 g
= 6008 SGD (correct to 4 s.f.) However, now its mass is only 120 g.
120 g  154.4 g
∴  it is NOT made of pure gold.
9. The interest rate for time deposits in a bank is 2% per
annum.
(a) Kui Meng deposits $2500 in the bank for 3 years. 12. In a gear system, the driving wheel has 42 teeth and the
Find the total interest he will earn at the end of 3 driven wheel has 28 teeth.
years if he withdraws the interest at the end of each (a) Find the gear ratio of the system.
year. (b) If the driving wheel makes 108 rotations per minute,
(b) Rohanna deposits $8000 in the bank and she find the number of rotations per minute of the driven
withdraws the interest at the end of each year. If wheel.
she wants to earn a total of $720 of interest, how
long should she deposit her money in the bank? Solution
(a) Gear ratio = 28 : 42
Solution 28 42
= :
(a) Simple interest 14 14
= $2500 × 2% × 3 =2:3
= $150 (b) number of rotations per minute of the driven wheel
(b) Let T years be the time of deposit. = 108 ×
2
8000 × 2% × T = 720 3
160T = 720 = 72
T = 720 ÷ 160
= 4.5 13. A company rents an office and a warehouse with floor
\  The time should be 4.5 years. areas of 100 m2 and 300 m2 respectively. The rental
rate of the office in dollars per square metre ($/m2) is
1
2 times that of the warehouse.
Level 3 2
10. The price of an apartment of area 120 m2 is $504 000. If the total rent of the office and the warehouse is $4400
Jenny and her mother shared the cost of such an apartment per month, find the monthly rental rate of the warehouse
in the ratio 2 : 3. in $/m2.
(a) How much did Jenny pay?
Solution
(b) What was the price per m2 of the apartment?
Let $x/m2 be the monthly rental rate of the warehouse
and $2.5x/m2 be the rental rate of the office.

51
x(300) + 2.5x(100) = 4400 Solution
300x + 250x = 4400 (a) Property tax = $30 000 × 10%
550x = 4400 = $3000
x = 8 (b) $30 000 = $6000 + $24 000
The monthly rental rate of the warehouse is $8/m2. For the first $6000, the property tax = $0
For the next $24 000,
14. On a day, a bank had the following exchange rates: the property tax = $24 000 × 4%
1 SGD = 0.8163 USD and 1 USD = 0.6877 EUR. = $960
(a) Find the exchange rate between the Singapore \  the required property tax
dollars (SGD) and the Euros (EUR) in EUR/SGD. = $0 + $960
(b) On the Internet, a watch was sold for 430 EUR at = $960
site A in Germany and 600 USD at site B in United
States.
16. Write down two examples of average rates in daily life.
(i) Find the selling price of the watch at site A
Discuss these examples with your classmates.
in SGD.
(ii) Find the selling price of the watch at site B Solution
in SGD. Two examples of average rates in daily life are the
(iii) Which site offered a cheaper price for the average rate of rainfall and the average interest rate a
watch? bank charges.
Solution
(a) 1 SGD = 0.8163 USD
= 0.8163 × 0.6877 EUR
Exercise 9.3
= 0.56137 EUR Level 1
= 0.5614 EUR (correct to 4 s.f.) 1. Copy and complete the following table.
\  The exchange rate is 0.5614 EUR/SGD.
Distance Time Average
(b) (i) Selling price at site A travelled taken speed
= 430 EUR
(a) 64 km 2h 32 km/h
1
= 430 × SGD (b) 200 m 25 s 8 m/s
0.56137
= 765.98 SGD (correct to 2 d.p.) (c) 66 km 3h 22 km/h
(ii) Selling price at site B (d) 35 m 7s 5 m/s
= 600 USD (e) 75 km 1.5 h 50 km/h
1
= 600 × SGD (f) 39 m 13 s 3 m/s
0.8163
= 735.02 SGD (correct to 2 d.p.) Solution
64
(iii) Site B offered a cheaper price. (a) Average speed = = 32 km/h
2
200
(b) Average speed = = 8 m/s
15. The annual value of an apartment is $30 000. 25
(a) Find the property tax of the apartment if the tax (c) Distance travelled = 22 × 3 = 66 km
rate is 10%.
(b) With effect from 1 January 2011, if the property is (d) Distance travelled = 5 × 7 = 35 m
occupied by the owner, the property tax is charged 75
(e) Time taken = = 1.5 h
by a progressive owner-occupier’s tax rates as 50
follows: 39
(f ) Time taken = = 13 s
3
Annual Value ($) Tax Rate (%)
First 6000 0 2. Convert the following speeds to m/s.
Next 59 000 4 (a) 24 km/h
Amount exceeding 65 000 6 (b) 50 km/h
(c) 80 km/h
Find the property tax of the apartment according (d) 126 km/h
to the given tax rates.

Chapter 9  Ratio, Rate and Speed


52
Solution Solution
1000 Distance between stations in km
(a) 24 km/h = 24 × m/s
3600
65 × 2 30
2 60
= 6 m/s = 60
3
1000 = 2.7
(b) 50 km/h = 50 × m/s
3600 The distance between the stations is 2.7 km correct to
8 0.1 km.
= 13 m/s
9
1000
(c) 80 km/h = 80 × m/s 6. When an F-16 jet fighter flew from its base to a city,
3600
2 it had an average speed of 2000 km/h. If the distance
= 22 m/s between the base and the city is 500 km, find the time
9
1000 taken in minutes.
(d) 126 km/h = 126 × m/s
3600
= 35 m/s Solution
Time taken in minutes
500
3. Convert the following speeds to km/h. = × 60
2000
(a) 2 m/s (b) 5 m/s = 15
(c) 13 m/s (d) 20 m/s It will take the F – 16 jet fighter 15 minutes to fly from
its base to the city.
Solution
3600
(a) 2 m/s = 2 × km/h
1000
Level 2
= 7.2 km/h
7. Mr Johnson took 40 minutes to drive from Village A to
3600 Village B at an average speed of 60 km/h.
(b) 5 m/s = 5 × km/h
1000 (a) Find the distance between A and B.
= 18 km/h (b) If he increased his speed by 12 km/h, how long
3600 would he take to travel the same distance?
(c) 13 m/s = 13 × km/h
1000
= 46.8 km/h Solution
40
3600 (a) Distance between A and B = 60 ×
(d) 20 m/s = 20 × km/h 60
1000
= 40 km
= 72 km/h
(b) New speed = 60 + 12
= 72 km/h
4. Jimmy runs 1500 m in 3 minutes 45 seconds. Find his 40
average speed in m/s. New time taken = × 60
72
1
Solution = 33 min
3
Average speed in m/s
1500
= 8. Hassan ran at a uniform speed for 50 minutes. He covered
3 × 60 + 45
1500 9 km during that period of time.
= (a) Find his speed in m/s.
225
2 (b) If he reduced his speed by 1 m/s, how long would
= 6 he take to cover the same distance?
3
2
Jimmy’s average speed is 6 m/s.
3
Solution
9 × 1000
(a) His speed = = 3 m/s
5. The average speed of an MRT train between two stations 50 × 60
is 65 km/h. If it takes 2 minutes and 30 seconds to travel (b) New speed = 3 – 1 = 2 m/s
from one station to the other, find the distance between 9 × 1000
New time taken =
the two stations correct to 0.1 km. 2
= 4500 s
4500
= min
60
= 75 min

53
9. Jack walks at an average speed of 4 km/h for one hour. 12. The distance between Singapore and Kuala Lumpur is
He then cycles 6 km at 12 km/h. Find his average speed 315 km. The average speeds of a car and an aeroplane
for the whole journey. from Singapore to Kuala Lumpur are 60 km/h and
900 km/h respectively. Find, for a trip from Singapore
Solution to Kuala Lumpur,
Total distance travelled = 4 × 1 + 6 (a) the time taken by the car and the aeroplane,
= 10 km (b) the ratio of the time taken by the car to that by the
aeroplane.
6
Total time taken = 1 + = 1.5 h
12
Solution
10 (a) Time taken by car in hours
Average speed =
1.5 315
2 =
= 6 km/h 60
3 1
=5
4
10. Mr Tan ran from his home to a provision shop 400 m Time taken by aeroplane in hours
away at an average speed of 2 m/s. He walked back to 315
=
his home at an average speed of 1 m/s. Find his average 900
speed for the whole journey if 7
=
(a) he did not stay in the shop, 20
(b) he stayed in the shop for 5 minutes. (b) Time taken by car : Time taken by aeroplane
1 7
Solution =5 :
4 20
400 400
(a) Total time taken = + =
21 7
:
2 1
4 20
= 600 s 21 7
= 20 × : 20 ×
2 × 400 4 20
Average speed =
600
= 105 : 7
1
= 1 m/s
3 105 7
= :
7 7
(b) Total time taken = 600 + 5 × 60
= 900 s =15 : 1
2 × 400
Average speed =
900 13. The flight distance between Singapore and Sydney is
8 about 6316 km. A plane takes 7 h 20 min to fly from
= m/s
9 Singapore to Sydney.
(a) Find the average speed of the plane from Singapore
to Sydney. Give your answer correct to the nearest
Level 3 km/h.
11. Sound travels at 340 m/s in air. (b) Sydney’s time is 2 hours ahead of Singapore’s time.
(a) Find the distance travelled by sound in air in If the plane departs from Singapore at 09 55, find
7 seconds. Give your answer correct to the nearest its time of arrival in Sydney.
0.1 km.
(b) A boy clapped his hands near a cliff. He could Solution
hear the echo of his claps 3 seconds later. Find his (a) Average speed of the plane
distance from the cliff. 6316
=
71
3
Solution
= 861 km/h  (correct to the nearest km/h)
(a) Distance travelled in 7 s
= 340 × 7 m (b) 7 h 20 min = 7 h (5 + 15) min
= 2380 m Time of arrival in Sydney
= 2.4 km  (correct to the nearest 0.1 km) = 11 55 + 7 h 20 min
3 = 19 15
(b) Distance from the cliff = 340 × m
2
= 510 m

Chapter 9  Ratio, Rate and Speed


54
14. Along a road, Point X and Point Y are 1150 m apart. Considering the total distance travelled, we have the
Ann walks along the road from X to Y at
2
m/s. Yohan equation
3 2x + 3y = 40 × 5
1
walks along the same road from Y to X at 1
m/s. Both i.e.  2x + 3y = 200.
4
start walking at the same time. Two possible sets of x and y are:
(a) How many minutes later will they meet? x = 10, y = 60
(b) How far is the point at which they meet from X? and x = 40, y = 40.
Solution
(a) Let the time that Ann and Yohan will meet be t min 17. Cars A and B drove uniformly in the same direction along
later. a road. At 9.00 a.m., B was 2 km ahead of A whose speed
2 1 was 60 km/h. The ratio of the speeds of A and B was
× (60t) + 1 × (60t) = 1150
3 4 4 : 3.
115t = 1150
t = 10
B A
They will meet 10 min later.
2
(b) The required distance = × 60 × 10
3 (a) Find the speed of B.
= 400 m (b) Describe the change in the distance between the
cars.
15. A train travels from P to Q at an average speed of (c) Will these two cars meet? If so, when will they
80 km/h. It returns from Q to P along the same rail at meet?
an average speed of 90 km/h. Find the average speed of
Solution
the train for the whole journey.
Speed of A 4
(a) =
Speed of B 3
Solution
60 4
Let d km be the distance between P and Q. =
Speed of B 3
Let T1 be the time taken in hours to travel from P to Q.
Let T2 be the time taken in hours to travel from Q to P. 3
Speed of B = 60 ×
d 4
T1 = = 45 km/h
80
d (b) Since speed of A  speed of B,
T2 =
90 the distance between the cars is decreasing.
Average speed in km/h (c) Yes, the two cars will meet.
d+d
= Let the time the two cars meet be t hours later.
T 1 + T2
60t = 45t + 2
2d 15t = 2
= 170d
2
7200 t =
15
170d
= 2d ÷ 2 2
7200 h = × 60 min
15 15
7200
= 2d × = 8 min
170d
1440 The two cars will meet 8 min later.
=
17
12
= 84
17

16. A car travels at an average speed of x km/h for the first


2 hours. It travels at an average speed of y km/h for the
next 3 hours. Its average speed for the whole journey is
40 km/h. Find two possible sets of values of x and y.

Solution
Total time taken = 2 + 3
=5h

55
Revision Exercise 9 4. (a) Simplify each of the following ratios.
1 2
1. Two cubes are of sides 6 cm and 8 cm respectively. Find (i) a:b=1 :2
2 5
the ratio of
(a) their sides, (ii) b : c = 0.105 : 0.350
(b) their areas on one face, (b) Find the ratio a : b : c using the data in (a).
(c) their volumes. (c) Alan, Bob and Cathy share $500 in the ratio
a : b : c found in (b). Find Alan’s share correct to
Solution 2 decimal places.
(a) Ratio of their sides = 6 : 8
=3:4 Solution
1 2
(b) Ratio of their areas = 6 : 8 2 2 (a) (i) a : b = 1 : 2
2 5
= 9 : 16 3 12
= :
3 3 2 5
(c) Ratio of their volumes = 6 : 8
= 27 : 64 = 15 : 24
= 5 : 8

2. There are a total of 240 sheets of $5 notes and $10 notes. (ii) b : c = 0.105 : 0.350
The numbers of $5 and $10 notes are in the ratio 3 : 2. = 105 : 350
Find = 3 : 10
(a) the number of $5 notes, (b) a : b = 5 : 8
(b) the number of $10 notes, = 15 : 24
(c) the ratio of the values of the $5 notes and $10 notes. b : c = 3 : 10
= 24 : 80
Solution
3 ∴  a : b : c = 15 : 24 : 80
(a) Number of $5 notes = 240 × = 144
3+2 15
(c) Alan’s share = $500 ×
2 15 + 24 + 80
(b) Number of $10 notes = 240 × = 96
3+2 15
= $500 ×
119
(c) Ratio of values of $5 notes and $10 notes
= $63.03  (correct to 2 d.p.)
= 144 × 5 : 96 × 10
= 3 : 4
5. The monthly salaries of Paula and Tom are in the ratio
of 5 : 9. The monthly salary of Tom is $3200 more than
3. A metal bar of mass 3.6 kg is cut into two pieces in the
that of Paula. Find the monthly salary of Paula.
ratio 3 : 5. The length of the shorter piece is 45 cm. Find
(a) the length of the longer piece,
Solution
(b) the length of the original metal bar,
Let $5x and $9x be the monthly salaries of Paula and
(c) its rate of mass per unit length in kg/m,
Tom respectively.
(d) the mass of the shorter piece.
9x – 5x = 3200
4x = 3200
Solution
x = 800
(a) Let x cm be the length of the longer piece.
\  monthly salary of Paula
45 3
= = $800 × 5
x 5
5 = $4000
x = 45 ×
3
= 75 6. Mrs Lim deposits $3800 in a bank at the interest rate of

The length of the longer piece is 75 cm. 3% per annum.
(a) Find the simple interest if the time of deposit is 4
(b) Length of original metal bar = 45 + 75 years.
= 120 cm (b) If the simple interest is $171, find the time of
3.6 kg deposit.
(c) Rate of mass per unit length =
1.2 m
= 3 kg/m Solution
45 (a) The required simple interest
(d) Mass of the shorter piece = 3 ×
100 = $3800 × 3% × 4
= 1.35 kg = $456

Chapter 9  Ratio, Rate and Speed


56
(b) Let t years be the time of deposit. Solution
3800 × 3% × t = 171 $15
(a) Unit price x of peanut oil =
2l
114t = 171
171 = $7.50/l
t =
114 $30
1 (b) Unit price y of olive oil =
t = 1 21l
2 3

1 = $12/l
The time of deposit is 1 years.
2
(c) x : y = 7.5 : 12
= 75 : 120
7. The annual value of a shop is $36 000. If the property
= 5 : 8
tax rate of the shop is 10%, find the amount of property
tax. (d) Peanut oil is cheaper. It is a better buy.
2l
Solution (e) Consumption rate =
16 days
Amount of property tax 1
= $36 000 × 10% = l/day
8
= $3600 1 1
(f ) Required number of days = 2 ÷
2 8
8. On a day, the exchange rate between the Singapore = 20
dollars (SGD) and the US dollars (USD) is 1
1 SGD = 0.816 USD, correct to 3 decimal places. The 10. A man took 4 hours to drive 360 km from Singapore
2
exchange rate between the Australian dollars (AUD) to Kuala Lumpur. He used 37.5 litres of petrol for the
and the US dollars is 1 AUD = 1.073 USD, correct to entire journey.
3 decimal places. (a) Find his average speed.
(a) Find the exchange rate, correct to 3 decimal places, (b) Find the petrol consumption rate in km/l.
between Singapore dollars and Australian dollars (c) He drove at an average speed of 110 km/h on a
in AUD/SGD. highway for 2 hours during his journey. Find his
(b) Mr Lim has a son studying in a university in average speed for the remaining part of his journey.
Sydney. He has to remit 25 000 AUD to him. Find
the required amount of Singapore dollars correct
Solution
to the nearest dollar. 360
(a) Average speed =
41
Solution 2

(a) Exchange rate in AUD/SGD = 80 km/h


1
= 1 360 km
1.073 (b) Petrol consumption rate =
0.816 37.5l
= 0.76 048 (correct to 5 d.p.) = 9.6 km/l
= 0.760 (correct to 3 d.p.) (c) Let x km/h be the required average speed.
(b) Amount of Singapore dollars required Consider the total distance travelled.
25 000 1
= 110 × 2 + x 4 – 2 = 360
0.76 048 2

= $32 873.97 5
220 + x = 360
2
1 5
9. A 2-litre bottle of peanut oil is sold for $15. A 2 -litre x = 140
2 2
bottle of olive oil is sold for $30. 2
(a) Find the unit price x of peanut oil in $/l. x = 140 ×
5
(b) Find the unit price y of olive oil in $/l. = 56
(c) Find the ratio x : y. His average speed for the remaining part of his
(d) Suppose both types of oil are equally good for journey is 56 km/h.
cooking, which one is a better buy?
(e) The bottle of peanut oil can be used for 16 days.
Find its consumption rate in l/day.
(f ) If the consumption rate of the bottle of olive oil
is the same as that of peanut oil, how long can it
last?

57
11. A car starts from rest. After travelling 125 m in 10 s, (b) (i) Average speed for the return journey
its speed picks up to 25 m/s. It travels at this speed for 120
=
20 seconds. Then brakes are applied. The car stops in 21
6
6 seconds and the braking distance is 95 m.
(a) Express the speed 25 m/s in km/h. 5
= 55 km/h
(b) Find the average speed of the car during the period 13
at which its speed increases. (ii) Average speed for the whole trip
(c) Find the average speed of the car during the period
120 + 120
the brakes were applied. =
2 + 21
(d) Find the average speed of the car for the whole 6
journey. 6
= 240 ×
25
Solution = 57.6 km/h
3600
(a) 25 m/s = 25 × km/h
1000

= 90 km/h
125
(b) Required average speed =
10
= 12.5 m/s
95
(c) Required average speed =
6
5
= 15 m/s
6

(d) Total distance travelled


= 125 + 25 × 20 + 95
= 720 m
Total time taken = 10 + 20 + 6
= 36 s
∴ average speed for the whole journey
720
=
36
= 20 m/s

12. Towns P and Q are 120 km apart. Mr Tan drove from


P to Q and was scheduled to reach Q after 2 hours. His
average speed was 54 km/h for the first 40 minutes.
(a) What was his average speed for the remaining
journey if he managed to arrive just on time?
(b) The time taken for his return journey is 2 hours
and 10 minutes. Find his average speed for
(i) the return journey,
(ii) the whole trip.

Solution
(a) Let x km/h be the required average speed.
40 40
54 × +x2– = 120
60 60
4
36 + x = 120
3
4
x = 84
3
x = 63
His average speed for the remaining journey is
63 km/h.

Chapter 9  Ratio, Rate and Speed


58
10 Percentage

Class Activity 1
Objective: To understand the meaning of percentage in some daily life examples and make connections between percentages
and fractions/decimals.

Questions
1. The percentage of oxygen in dry air by volume is about 21%.

(a) What fraction of dry air is oxygen?


Fraction of oxygen in dry air
21
=
100

(b) Find the volume of air that a normal man inhales in each time.

A normal man inhales about 500 ml of air each time.



(c) Hence, find the volume of oxygen that a normal man inhales in each time.

Volume of oxygen inhaled each time


21
= × 500
100
= 105 ml

A normal man inhales about 105 ml of oxygen each time.

2. In the 2011/12 NBA season, the basketball player Jeremy Lin had a field goal percentage of 44.6%.

(a) Find the meaning of field goal percentage on the Internet.

Field goal percentage is the ratio of number of field goals made to the number of field goals attempted.

(b) If the total number of shots attempted by Jeremy was x in the season, what would be the number of goals he made?
Express your answer as a decimal of x.
number of goals made
= 44.6%
number of goals attempted
number of goals made 44.6
=
x 100
number of goals made = 0.446x

3. On 13 April 2012 news, SMRT announced that “To date, about 245 buses or 23 per cent of SMRT's bus fleet were
wheelchair accessible.”
Rephrase the above statement using fraction.
23
To date, about 245 buses or of SMRT’s bus fleet were wheelchair accessible.
100

59
4. A club had 13 800 members in the year 2012. Mr X and Mr Y contested in the chairman election in that year. Total valid
votes polled for Mr X was 7289 and that for Mr Y was 6103. Mr X won by obtaining 54.4% of the votes.

(a) What fraction of valid votes did Mr X obtain?

Fraction of valid votes Mr X obtained


54.4
=
100
544
=
1000
68
=
125

(b) Explain how the percentage 54.4% is derived.

7289
× 100%
7289 + 6103
= 54.4%

(c) Find the percentage of votes obtained by Mr Y, correct to 3 significant figures.

Percentage of votes obtained by Mr Y


6103
= × 100%
7289 + 6103
= 45.6%

Chapter 10  Percentage


60
Class Activity 2
Objective: To compare two quantities by more or less percentage.

Questions
The mass of Tim is 60 kg. The mass of Fred is 5% more than that of Tim.

1. What is the mass of Fred?

Mass of Fred
= 105% × 60 kg
105
= × 60 kg
100
= 63 kg

2. By how many percent is the mass of Tim less than that of Fred?
63 – 60
× 100%
63
= 4.76%

Mass of Tim is 4.76% less than mass of Fred.

3. Discuss whether the statement ‘If A is 5% more than B, then B is 5% less than A’ is correct or not.

The statement is incorrect. The correct statement should be ‘If A is 5% more than B, B is 4.76% less than A.’

4. Illustrate by an example to show whether the statement ‘If A is 20% less than B, then B is 20% more than A’ is correct
or not.

Consider the height of two plants. Plant A is 80 cm tall while plant B is 100 cm tall.

100 – 80
× 100% = 20%
100

Height of plant A is 20% less than height of plant B.

100 – 80
× 100% = 25%
80

Height of plant B is 25% more than height of plant A.

Therefore, the statement ‘If A is 20% less than B, then B is 20% more than A’ is incorrect.

The correct statement should be ‘If A is 20% less than B, then B is 25% more than A’.

61
Class Activity 3
Objective: To work out the calculation of GST, service charge and discount on the receipts.

Questions
1. Refer to Receipt 1 from a restaurant.

(a) Explain how the subtotal amount of $26.40 is obtained?

The subtotal is the sum of all the prices listed above it.

i.e. $(9.80 + 3.50 + 3.30 + 4.20 + 2.00 + 0.60 + 3.00) = $26.40

(b) A 10% service charge (S.C.) is included in the bill. How is the 10% service charge calculated?

The service charge is based on the subtotal amount.


i.e. service charge = $26.40 × 10%
= $2.64

(c) The GST rate is 7%. How is the 7% GST calculated?

The GST is based on the sum of the subtotal amount and the service charge.
i.e. GST = $(26.40 + 2.64) × 7%
= $2.03 (correct to 2 d.p.)

(d) What is the meaning and purpose of the round amount?

The purpose of the rounded amount is to make the total amount end to the nearest 5 cents.

(e) If you are the manager of the restaurant, how would you explain to a customer who queries how the total amount
of $31.05 is obtained?

The total amount = $(26.40 + 2.64 + 2.03 – 0.02)


= $31.05

Chapter 10  Percentage


62
2. Refer to Receipt 2 from a restaurant.

(a) Explain how the 15% discount is calculated.

The 15% discount is based on the price of the set menu.


i.e. discount = $518 × 15%
= $77.70

(b) Explain how the subtotal amount (SUBTTL) of $477.30 is obtained.

The subtotal amount is equal to the sum of the prices of the items above it minus the discount.
i.e. subtotal amount
= $(5.00 + 20.00 + 7.00 + 5.00 + 518.00 – 77.70)
= $477.30

(c) Show how the GST is calculated.

The GST is 7% of the subtotal amount.


i.e. GST = $477.30 × 7%
= $33.41 (correct to 2 d.p.)

(d) Explain how the total amount of $510.70 is obtained.

The total amount is the sum of the subtotal amount and the GST, rounded to the nearest 10 cents.
i.e. total amount = $477.30 + $33.41
= $510.71
= $510.70 (correct to the nearest 10 cents)

3. Refer to Receipt 3 from a supermarket.

(a) What is the relationship between the subtotal and total amounts in this case?

The total amount is obtained by rounding the subtotal amount correct to the nearest 10 cents.
i.e total amount = $7.42 – $0.02
= $7.40

(b) There are three figures on a row indicating GST rate, amount before GST and GST amount. Are you able to explain
how they are calculated? Show your working clearly.

The GST rate is 7% is set by the government.


Amount before GST = $7.42 ÷ (1 + 7%)
= $6.93 (correct to 2 d.p.)
GST amount = total amount – amount before GST
= $7.42 – $6.93
= $0.49

63
Try It! 4. 650 people attended a family-day function organised by
a community centre. If 24% of them were men, 32% of
Section 10.1
them were women and the rest were children, find the
1. A company has 50 employees and 34 of them are women. number of
Find the percentage of (a) men,
(a) female employees, (b) women,
(b) male employees. (c) children,
Solution attending the function.
34
(a) Percentage of female employees = × 100%
50
Solution
= 68% (a) Number of men = 650 × 24%
(b) Percentage of male employees = 100% – 68% = 156
= 32% (b) Number of women = 650 × 32%
= 208
2. Alan’s height is 135 cm and Suling’s height is 150 cm.
(c) Number of children = 650 – 156 – 208
Express = 286
(a) Alan’s height as a percentage of Suling’s height,
(b) Suling’s height as a percentage of Alan’s height.
5. Tank A has 20 goldfish and tank B has 25 goldfish. Both
Solution tanks have the same number of goldfish with short tail
Alan’s height fin. If the percentage of goldfish with short tail fin in
(a) The required percentage = × 100%
Suling’s height tank B is 36%, find the percentage of goldfish with short
135 tail fin in tank A.
= × 100%
150
= 90% Solution
Number of short tail fin goldfish in tank B
Suling’s height
(b) The required percentage = × 100% = 36% × 25
Alan’s height
= 9
150
= × 100% Percentage of short tail fin goldfish in tank A
135
1 9
= 111 % = × 100%
9 20
= 45%
3. The following table shows the test results of 3 groups
of students.
Total number Number of Section 10.2
Group
of students students who pass 6. Kumar was fined $187 while driving. If the fine was
A 20 13 110% of his daily wage, find Kumar’s daily wage.

B 24 15 Solution
C 25 16 Let $w be Kumar’s daily wage.
w × 110% = 187
Which group of students has the highest pass percentage? 187
w =
1.1
Solution
Percentage of pass in group A = 170
13 Kumar’s daily wage is $170.
= × 100%
20
= 65% 7. Bob’s mass is 25% more than that of Ada. Carl’s mass is
Percentage of pass in group B 5% less than that of Bob. If the total mass of Ada, Bob
15
= × 100% and Carl is 165 kg, find Bob’s mass.
24
= 62.5% Solution
Percentage of pass in group C Let x be Ada’s mass.
16 Ada’s mass = x kg
= × 100%
25 Bob’s mass = 125% of x kg
= 64% = 1.25x kg
Hence, group A has the highest percentage of students Carl’s mass = 95% of 1.25x kg
who passes. = 1.1875x kg

Chapter 10  Percentage


64
x + 1.25x + 1.1875x = 165 Method 2:
3.475x = 165 Sales volume for this week = (100 + 16)% × 250
x = 48 116
= × 250
1.25x = 60 100
Bob’s mass is 60 kg. = 290 CDs

11. A boy’s height is 133 cm at the end of a year. His height


Section 10.3 has increased by 6.4% during the year. Find his height
8. Mr Chen’s monthly expenditure increased from $3200 to at the beginning of the year.
$3400. Find the percentage increase in his expenditure.
Solution
Solution Let the boy’s height at the beginning of the year be
Percentage increase in his expenditure h cm.
3400 – 3200 133 = h × (100% + 6.4%)
= × 100%
3200 106.4
200 =h×
133
= × 100% 100
3200 133
h =
= 6.25% 1.064

= 125

9. There are 40 students in a class. In term 1, 25% of His height at the beginning of the year was 125 cm.
students got grade A in Mathematics. In term 2, 30% of
students got grade A in Mathematics. Find the percentage 12. The water level in a tank decreased from 30 cm to
increase in the number of students who got grade A in 25.5 cm. Find the percentage decrease in the water level.
Mathematics.
Solution
Solution Decrease in height of water level = 30 – 25.5
Number of students who got grade A in term 1 = 4.5 cm
= 25% × 40
= 10 Percentage decrease in height of water level
4.5
= × 100%
Number of students who got grade A in term 2 30
= 30% × 40 = 15%
= 12
Increase in number of students who got grade A 13. The volume of air in a balloon is 1600 cm3. If 12.5% of
= 12 – 10 air is released, find the new volume of air in the balloon.
= 2
Solution
Percentage increase in the number of students who got
New volume
grade A
= Original volume × (100% – Percentage decrease)
2
= × 100% = 1600 × (100% – 12.5%)
10
87.5
= 20% = 1600 ×
100
= 1400
10. A shop sold 250 CDs last week. The sales of CDs
increased by 16% this week. How many CDs were sold
14. Mrs Li’s mass dropped to 62 kg after her mass decreased
this week?
by 7%. What was her original mass? Give your answer
correct to 3 significant figures.
Solution
Method 1: Solution
Increase
Percentage increase = × 100% Let Mrs Li’s original mass be m kg.
Original value
62 = m × (100% – 7%)
Increase = 250 × 16% 93
= 40 = m ×
100
62
Sales volume for this week = (250 + 40) m = = 66.7  (correct to 3 s.f.)
  0.93
= 290 CDs
Mrs Li’s original mass was 66.7 kg.

65
15. The number of girls in a dancing club is twice as many 570
= M × (100% – 24%)
as the number of boys. If the number of girls decreased 76
= M ×
by 20%, by what percentage must the number of boys 100
be increased so that there is an equal number of boys 100
 M = 570 ×
and girls in the club? 76
= 750
Solution The marked price of the TV set is $750.
Let x be the number of boys.
(b) The new selling price
Number of girls = 2x
= $750 × (100% – 15%) × (100% – 9%)
Number of girls after 20% decrease
= $750 × 0.85 × 0.91
= 2x × (100% – 20%)
= $580.125
= 1.6x
Increase in the selling price
Required percentage increase in number of boys = $580.125 – $570
1.6x – x = $10.125 
= × 100%
5 = $10.13  (correct to 2 d.p.)
= 0.6 × 100%
= 60%
19. A plumber charges $20 for a job excluding GST. If the
GST rate is 7%,
(a) how much is the GST for the service provided?
Section 10.4 (b) how much does a customer have to pay for the
16. During a sale, a watch was sold for $784. If the marked service provided inclusive of GST?
price of the watch is $980, find the percentage discount.
Solution
Solution (a) GST
= Service charge GST rate
Discount
= $980 – $784 = $20 × 7%
= $196 = $1.40
196
Percentage discount = × 100% (b) Total amount = Service charge + GST
980
= $20 + $1.40
= 20%
= $21.40

17. The marked price of a mobile phone is $380. If the phone 20. The marked price, inclusive of GST, of a vacuum cleaner
is sold at a discount of 25%, find in a shop is $214. The GST rate is 7%.
(a) its discount, (b) its selling price. (a) Find its price before GST.
(b) Find the amount of GST levied on it.
Solution (c) If the vacuum cleaner is sold at 10% discount, find
(a) Discount
= $380 × 25% the decrease in the GST levied.
= $95
Solution
(b) Method 1:
(a) Price before GST
Selling price = $380 – $95
214
= $285 =$
1.07
Method 2: = $200
Selling price
= Marked price × (100% – Discount %) (b) GST = Marked price – Price before GST
75 = $214 – $200
= $380 × = $285 = $14
100
(c) Selling price of vacuum cleaner
18. A TV set is sold at $570 after a 24% discount. = $214 × (100% – 10%)
(a) Find the marked price of the TV set. = $192.60
(b) If the TV set was sold at a 15% discount and then GST levied
at a further 9% discount on the discounted price, 192.60
= $192.60 –
1.07
how much more or less would this selling price be?
= $192.60 – $180
Solution = $12.60
(a) Let $M be the marked price of the TV set. Decrease in GST levied
Selling price = $14 – $12.60
= Marked price × (100% – Discount %) = $1.40

Chapter 10  Percentage


66
Exercise 10.1 3. Find the value of the following.
Level 1 (a) 40% of 120 cm
(b) 15% of 360°
1. Express P as a percentage of Q in each of the following.
2
(a) P = $250, Q = $1000 (c) 66 % of 54 s
3
(b) P = 12 m, Q = 15 m (d) 2% of $200
(c) P = 20 kg, Q = 25 kg
Solution
(d) P = 375 cm3, Q = 200 cm3
(a) 40% × 120 cm
Solution = 48 cm
(a) The required percentage (b) 15% × 360°
250 = 54°
= × 100%
1000
2
= 25% (c) 66 % of 54 s
3
(b) The required percentage = 36 s
12
= × 100% (d) 2% of $200
15
= $4
= 80%
(c) The required percentage 4. Mr Singh’s monthly salary is $4000. He rents a room
20
= × 100% which costs him $1200 per month. What percentage of
25
his salary is spent on rent?
= 80%
(d) The required percentage Solution
375 Percentage of salary spent on rent
= × 100%
200 1200
= × 100%
= 187.5% 4000
= 30%
2. Express Y as a percentage of X in each of the following.
(a) X = 40 s, Y = 30 s 5. In a class of 36 students, 12 of them wear glasses. Find
(b) X = 36 °C, Y = 24 °C the percentage of students who do not wear glasses in
(c) X = 38 cm, Y = 95 cm the class.
(d) X = 44 g, Y = 77 g
Solution
Solution Percentage of students who do not wear glasses
(a) The required percentage =
36 – 12
× 100%
30 36
= × 100%
40 2
= 66 %
= 75% 3

(b) The required percentage


24 6. There are 500 cars passing through a certain junction in
= × 100% an hour. If 13% of the cars exceed the speed limit, find
36
1 the total number of cars that have exceeded the speed
= 66 %
2 limit.
(c) The required percentage
Solution
95
= × 100% Number of cars exceeding speed limit
38
= 500 × 13%
= 250%
= 65
(d) The required percentage
77
= × 100% 7. A drink contains 32% sugarcane juice. Find the amount
44
of sugarcane juice in 375 cm3 of the drink.
= 175%
Solution
Amount of sugarcane juice
= 375 × 32%
= 120 cm3

67
Level 2 Solution
8. Gopal earns an income of $4000 a month. He spends (a) The required percentage
45
$3360 and saves the rest. = × 100%
40
(a) What percentage of his income is his expenditure?
(b) What percentage of his income is his savings? = 112.5%
(b) The required percentage
Solution 40
3360 = × 100%
45
(a) The required percentage = × 100%
4000
= 88.9%  (correct to 3 s.f.)
= 84%
(b) The required percentage = 100% – 84% 12. The length of a rectangle is 160% of its breadth. If the
= 16% breadth of the rectangle is 50 cm, find
(a) the length of the rectangle,
9. There are 80 books on a bookshelf. 28 of them are English (b) the perimeter of the rectangle.
books, 32 of them are Mathematics books and the rest
are Science books. Find the percentage of Solution
(a) English books, (a) Length of rectangle
(b) Mathematics books, = 160% × 50
(c) Science books = 80 cm
on the shelf. (b) Perimeter of rectangle
= 2(50 + 80)
Solution = 260 cm
28
(a) Percentage of English books = × 100%
80
= 35% Level 3
(b) Percentage of Mathematics books =
32
× 100% 13. In the year 2010, the population of Singapore was
80 5 076 700 and the population of Hong Kong was 7 097
= 40% 600. Express
(a) the population of Singapore as a percentage of the
(c) Percentage of Science books
population of Hong Kong,
= 100% – 35% – 40%
(b) the population of Hong Kong as a percentage of
= 25%
the population of Singapore.
Give your answer correct to 3 significant figures.
10. In a 400-ml can of mixed juice, 41% is mango juice,
32.5% is pineapple juice and the rest is water. Find the Solution
volume of (a) Population of Singapore as a percentage of the
(a) mango juice, population of Hong Kong
(b) pineapple juice, 5 076 700
= × 100%
(c) water 7 097 600
in the can. = 71.5%  (correct to 3 s.f.)
(b) Population of Hong Kong as a percentage of the
Solution
population of Singapore
(a) Volume of mango juice = 400 ml × 41%
7 097 600
= 164 ml = × 100%
5 076 700
(b) Volume of pineapple juice = 400 ml × 32.5% = 140%  (correct to 3 s.f.)
= 130 ml
(c) Volume of water = (400 – 164 – 130) ml 14. At a shooting practice, Peter hit the target 18 times out
= 106 ml of 25 shots and Cliff hit it 15 times out of 20 shots. Who
had a higher percentage of hitting the target during the
practice?
11. Ali’s mass is 45 kg and May’s mass is 40 kg.
(a) Express Ali’s mass as a percentage of May’s mass.
Solution
(b) Express May’s mass as a percentage of Ali’s mass.
Peter’s percentage of shots that hit targets
18
= × 100%
25
= 72%

Chapter 10  Percentage


68
Cliff’s percentage of shots that hit targets Solution
15 (a) Area of living room
= × 100%
20 = 30% × 60 m2
= 75% = 18 m2
Cliff is better at shooting. (b) Required percentage
9
= × 100%
60
15. The vases produced by 3 production lines are inspected.
The following table shows the results. = 15%

Number of Number of 18. The number of pages of magazines A and B are 64 and
Line
vases inspected defective vases 96 respectively. The space used for advertisement in
1 30 3 magazine B is twice that in magazine A. If the percentage
2 50 4 of space used for advertisement in magazine B is 40%,
3 21 2 find the percentage of space used for advertisement in
magazine A.
Which production line had the lowest percentage of
defective vases? Solution
Number of pages of advertisement in magazine B
Solution = 40% × 96
Percentage of defective vases from Line 1 = 38.4
3
= × 100% Number of pages of advertisement in magazine A
30
= 38.4 ÷ 2
= 10%
= 19.2
Percentage of defective vases from Line 2
4
Required percentage
= × 100% 19.2
30 = × 100%
64
= 8%
= 15%
Percentage of defective vases from Line 3
2
= × 100% 19. The following table shows the number of different atoms
21
in three compounds.
11
= 9 % Number of atoms
21
Production line 2 has the lowest percentage of defective Compound Carbon Hydrogen Oxygen
vases. (C) (H) (O)
Benzene,
6 6 0
16. The capacity of a large cup is 50% more than that of a C6H6
small cup. By how many per cent is the capacity of the Ethanol,
small cup less than that of the large cup? 2 6 1
C2H5OH

Solution Glucose,
6 12 6
Let x be the capacity of the small cup. C6H12O6
Capacity of big cup
(a) Find the percentage of hydrogen atoms in each
= (100% + 50%) × x
compound.
= 1.5x
(b) Which compound has the highest percentage of
Required percentage hydrogen atoms?
1.5x – x
= × 100%
1.5x Solution
1 (a) Percentage of hydrogen atoms in Benzene
= 33 %
3 6
= × 100%
6+6+0
2
17. The area of a 3-room HDB flat is 60 m . = 50%
(a) The area of the living room is 30% of the area of Percentage of hydrogen atoms in Ethanol
the flat. Find the area of the living room. 6
(b) The area of the kitchen is 9 m2. Express the area = × 100%
2+6+1
of the kitchen as a percentage of the total area of 2
the flat. = 66 %
3

69
Percentage of hydrogen atoms in Glucose 2. 25% of the staff in an office is infected by flu. If
12 the number of staff infected by flu is 6, find the total
= × 100%
6 + 12 + 6 number of staff in the office.
= 50%
Solution
(b) Ethanol has the highest percentage of hydrogen
Let n be the total number of staff.
atoms.
n × 25% = 6
n = 6 ÷ 25%
= 6 × 100/25
Exercise 10.2 = 24
Level 1 The total number of staff is 24.
1. Find the unknown quantity in each case.
(a) 30% of a is 18.
3. A piece of alloy contains 4 kg of copper. If the alloy
(b) 37.5% of $b is $108.
contains 80% copper by mass, find the mass of the piece.
2
(c) 22 % of c kg is 44 kg.
9
Solution
(d) 150% of d cm2 is 126 cm2. Let m kg be the mass of the piece.
(e) 0.5% of e °C is 7 °C. m × 80% = 4
1 m = 4 ÷ 80%
(f ) % of f hours is 12 hours.
3
= 4 × 100/80
= 5
Solution
The piece of alloy is 5 kg.
(a) a × 30% = 18
18
a =
0.3 4. Adam attempts 65% of the questions in a test. If he
= 60 attempts 52 questions, find the total number of questions
The required quantity is 60. in the test.
(b) b × 37.5% = 108
Solution
108
b = Let the number of questions in the test be n.
0.375
n × 65% = 52
= 288 52
The required quantity is $288. n =
0.65
2 = 80
(c) c × 22 % = 44
9 There are 80 questions in the test.
44
c = 200
× 1
9 100

= 44 ×
9 Level 2
2
5. 45% of the members in a council are women. There are
= 198 72 female council members. Find
The required quantity is 198 kg. (a) the total number of council members,
(d) d × 150% = 126 (b) the number of male council members.
126
d = Solution
1.5
= 84 (a) Let n be the total number of council members.
The required quantity is 84 cm2. n × 45% = 72
72
(e) e × 0.5% = 7 n =
0.45
7 = 160
e =
0.005
The total number of council members is 160.
= 1400
The required quantity is 1400 °C. (b) Number of male council members = 160 – 72
= 88
1
(f )
f × % = 12
3
12
f = 6. Last Sunday, 85% of the customers of a supermarket in
1
× 1 a housing estate were residents of the same estate. 2380
3 100

= 3600 of the customers that day were residents. Find


The required quantity is 3600 hours. (a) the total number of customers,

Chapter 10  Percentage


70
(b) the number of customers who were not residents (c) Which stall prepared the greatest number of glasses
on that day. of drinks? What was this figure?

Solution Solution
(a) Let N be the total number of customers. (a) Number of glasses of drinks sold by Stall A
N × 85% = 2380 = 175
2380 Number of glasses of drinks sold by Stall B
N =
0.85 = 220 × 85%
= 2800 = 187

The total number of customers is 2800. Number of glasses of drinks sold by Stall C
= 180
(b) Number of customers who were not residents
Stall B sold the greatest number of glasses of drinks.
= 2800 – 2380
The number of glasses of drinks sold was 187.
= 420
(b) Percentage of glasses of drinks sold by
175
7. John is 10% taller than Mary. If the difference between Stall A = × 100%
200
their heights is 16 cm, find Mary’s height. = 87.5%
Percentage of glasses of drinks sold by
Solution Stall B = 85%
Let h cm be the height of Mary. Percentage of glasses of drinks sold by
Height of John = h × (1 + 10%) Stall C = 80%
= 1.1h Stall A sold the highest percentage of its glasses
1.1h – h = 16 of drinks.
0.1h = 16 This percentage figure was 87.5%.
16
h = (c) Number of glasses of drinks by
0.1
= 160 Stall A = 200
\  The height of Mary is 160 cm. Number of glasses of drinks by
Stall B = 220
Number of glasses of drinks by
8. The price of book A is 15% less than that of book B. Stall C = 180 ÷ 80%
If the total price of book A and book B is $74, find the = 225
price of book B. Stall C prepared the greatest number of glasses of
drinks, which was 225 glasses of drinks.
Solution
Let $p be the price of Book B.
10. In Lihua’s music CD collection, 38% are Chinese music
Price of Book A = $p × (1 – 15%)
CDs, 44% are English music CDs and the rest are
= $0.85p
Japanese music CDs. There are 45 Japanese music CDs.
p + 0.85p = 74
(a) Find the total number of CDs in her collection.
1.85p = 74
74
(b) Find the number of Chinese music CDs.
p = 7
1.85 (c) If 63 %
of the English music CDs are CDs with
11
= 40 pop songs, find the number of CDs with English
\  The price of Book B is $40. pop songs.

Solution
Level 3 (a) Percentage of classical music CDs
9. At a food centre, there are three drink stalls. On a certain = 100% – 38% – 44%
day, Stall A sold 175 glasses out of 200 glasses of sugar = 18%
cane juice, Stall B sold 85% of its 220 glasses of soya Total number of CDs = 45 ÷ 18%
bean drink and Stall C sold 180 glasses of herbal tea = 250
which is 80% of its prepared glasses of herbal tea. (b) Number of Chinese music CDs = 250 × 38%
(a) Which stall sold the greatest number of glasses of = 95
drinks? How many glasses of drinks were sold by
(c) Number of CDs with English pop songs
this stall?
7
(b) Which stall sold the highest percentage of = 250 × 44% × 63 %
11
its prepared glasses of drinks? What was this 700 1
percentage figure? = 250 × 0.44 × ×
11 100
= 70

71
11. The master bedroom in a flat is 20% larger than In general,
the children’s bedroom. The area of living room is x% of y = xy%
200% that of the master bedroom. If the living room is and y% of x = xy%
14 m2 larger than the children’s bedroom, find ∴  x% of y and y% of x are equal.
(a) the area of the children’s bedroom,
(b) x × 60% = 30
(b) the total area of three rooms.
30
x =
0.6
Solution
(a) Let x m2 be the area of the children’s bedroom. = 50
Area of the master bedroom y × 30% = 60
60
= x (1 + 20%) y =
0.3
= 1.2x m2
Area of the living room = 200
= 1.2x × 200% ∴  x  y
= 2.4x m2 (c) A club P has 30 members. If 60% of the members
2.4x – x = 14 are male, then there are 18 male members.
1.4x = 14 A club Q has 60 members. If 30% of the members
14 are male, then there are 18 male members.
x =
1.4 The number of male members of these two clubs
= 10 are equal.
The area of the children’s bedroom is 10 m2.
(b) Total area of three rooms
= x + 1.2x + 2.4x Exercise 10.3
= 4.6x Level 1
= 4.6 × 10 1. Copy and complete the following table.
= 46 m2
Original Increased Percentage
Increase
value value increase
12. A piece of architectural bronze, made of an alloy of (a) $25 $28 $3 12%
copper, zinc and lead, contains 57% of copper and 40%
of zinc by mass. If the difference in the mass of copper (b) 16 s 20 s 4s 25%
and that of zinc is 5.1 kg, find (c) 40 g 52 g 12 g 30%
(a) the mass of the piece, (d) 20 cm 23 cm 3 cm 15%
(b) the mass of lead in the piece.
(e) 50 °C 55 °C 5 °C 10%

Solution (f ) 60 copies 69 copies 9 copies 15%
(a) mass of piece
5.1 Solution
= kg
57% – 40% (a) Increase
= $28 – $25
= 30 kg = $3
3
(b) mass of lead in piece Percentage increase = × 100%
25
= (100% – 57% – 40%) × 30 kg = 12%
= 0.9 kg
(b) Increased value = 16 + 4
= 20 s
13. (a) Is 60% of 30 equal to 30% of 60? Explain your 4
answer. Generalise your findings using variables x Percentage increase = × 100%
16
and y.
= 25%
(b) If 60% of x is 30 and 30% of y is 60, is x equal to
y? Explain your answer. (c) Increased value = 40 × (100% + 30%)
(c) Write a real-life problem that involves the = 52 g
mathematics in (a). Increase = 40 × 30%
= 12 g
Solution
(d) Original value = 23 – 3
(a) 60% of 30 = 30 × 60% = 18
= 20 cm
30% of 60 = 60 × 30% = 18 3
∴  60% of 30 and 30% of 60 are equal. Percentage increase = × 100%
20
= 15%

Chapter 10  Percentage


72
(e) 55 = Original value × (100% + 10%) 3. Ronald’s monthly salary is $3000. If his salary increased
Original value = 55 °C ÷ 1.1 by 6%, find his new monthly salary.
= 50 °C
Increase = 55 °C – 50 °C Solution
= 5 °C New salary = $3000 × (100% + 6%)
= $3180
(f ) Original value = 9 ÷ 15%
= 60 copies
Increased value = 60 + 9 4. The price of a watch increases by 8% to $777.60. Find
= 69 copies the original price of the watch.

Solution
2. Copy and complete the following table.
Let $P be the original price of the watch.
Original Decreased Percentage P × (100% + 8%) = 777.60
Decrease
value value decrease P = 777.60 ÷ 1.08
(a) 2 = 720
36 kg 30 kg 6 kg 16 3 %
The original price of the watch was $720.
(b) 2h 1.5 h 0.5 h 25%
(c) 80 m 64.8 m 15.2 m 19% 5. The price of a computer system decreases from $1250
(d) 15 cm3 12 cm3 3 cm3 20%
to $1050. Find the percentage decrease in price.

(e) $70 $49 $21 30% Solution


(f ) 55 ml 44 ml 11 ml 20% Decrease
= $1250 – $1050
= $200
Solution 200
Percentage decrease in price = × 100%
(a) Decrease
= 36 – 30 1250
= 6 kg = 16%
6
Percentage decrease = × 100%
36
6. Rahim’s monthly expenditure is $2464 after it decreased
2 by 12%. Find his original monthly expenditure.
= 16 %
3
(b) Decreased value = 2 – 0.5 Solution
= 1.5 h Let $P be the original monthly expenditure.
Percentage decrease =
0.5
× 100% P × (100% – 12%) = 2464
2 2464
P =
= 25% 0.88

(c) Decreased value = 80 m × (100% – 19%) = 2800


= 64.8 m His original monthly expenditure was $2800.
Decrease = 80 m × 19%
= 15.2 m Level 2
7. The mass of a baby boy at birth was 3.6 kg. When he
(d) Original value = 12 + 3 was 6 months old and 12 months old, his masses were
= 15 cm3 7.2 kg and 10.0 kg respectively. What was the percentage
3
Percentage decrease = × 100% increase in his mass at 12 months old when compared
15
with
= 20% (a) his mass at birth?
(e) $49 = Original value × (100% – 30%) (b) his mass at 6 months old?
Original value = $49 ÷ 0.7
= $70 Solution
Decrease = $70 – $49 (a) Increase
= 10.0 – 3.6
= $21 = 6.4 kg
6.4
(f ) Original value = 11 ml ÷ 20% Percentage increase = × 100%
3.6
= 55 ml 7
= 177 %
Decreased value = 55 – 11 9
= 44 ml (b) Increase
= 10.0 – 7.2
= 2.8 kg

73
Percentage increase =
2.8
× 100% (b) Number of customers at 2 p.m.
7.2 = 65 × (1 – 20%)
8
= 38 % = 65 × 80%
9
= 52
(c) The required percentage change
8. The regular price of an airline ticket is $960. (52 – 50)
(a) Its price increases by 30% during the peak season. = × 100%
50
Find the price of the ticket during the peak season. = 4%
(b) If the regular price drops to $748.80, find the
percentage decrease in the price.
Level 3
Solution
11. Refer to the news on 1 July 2011 below:
(a) Price of ticket during the peak season
HDB flat prices in the second quarter of 2011 rose by
= $960 × (100% + 30%)
2.9 per cent to a record, according to initial estimates
= $1248
by the Housing Board.
(b) Decrease = $960 – $748.80
The increase is the fastest since the third quarter of 2010.
= $211.20
Resale flat prices in the first quarter had risen by 1.6
211.20
Percentage decrease = × 100% per cent from the fourth quarter of 2010.
960
= 22% Suppose the price of an HDB flat was $400 000 on 1
January 2011. Estimate its price on
(a) 31 March 2011, (b) 30 June 2011.
9. The original height of a plant was 25 cm. One month
later, its height increased by 20%. In the second month, Solution
it increased by 15% of the plant’s height at the end of (a) Price on 31 March 2011
the first month. Find its height after = $400 000 × (1 + 1.6%)
(a) 1 month, = $406 400
(b) 2 months.
(b) Price on 30 June 2011
Solution
= $406 400 × (1 + 2.9%)
(a) Plant’s height after 1 month = $418 185.60
= 25 cm × (100% + 20%)
= 30 cm 12. Refer to the news on 30 June 2011 below:
(b) Plant’s height after 2 months Households can expect to pay an average 6.6 per cent
= 30 cm × (100% + 15%) more for their electricity in the July-September quarter
= 34.5 cm as tariffs go up.
SP Services said higher fuel oil prices have resulted
in higher power generation costs. The average fuel oil
10. There were 65 customers in a cafe at 1 p.m. on one day. price over the last three months has risen by 13.8 per
The number of customers increased by 30% from 12 p.m. cent from US$88.63 to US$100.84 per barrel.
to 1 p.m. and it decreased by 20% from 1 p.m. to 2 p.m. (a) Show how SP Services obtain the figure of 13.8%
(a) Find the number of customers in the cafe at 12 p.m. increase in oil price. Show your workings clearly.
(b) Find the number of customers in the cafe at 2 p.m. (b) Assume that a household consumes the same
(c) Find the percentage change in the number of units of electricity energy in June and July. If its
customers in the café from 12 p.m. to 2 p.m. electricity bill in July is $2.80 more than in June,
find the amount of its electricity bill in June.
Solution
(a) Let n be the number of customers at 12 p.m. Solution
Number of customers at 1 p.m (a) Percentage increase in oil price
= n × (1 + 30%) 100.84
= 1.3n = × 100% – 100%
88.63
1.3n = 65 = 13.8%  (correct to 3 s.f.)  (shown)
65
n = (b) Amount of electricity bill in June
1.3
n = 50 $2.80
=
The number of customers at 12 p.m. is 50. 6.6%
= $42.42

Chapter 10  Percentage


74
13. The population of a city was 3 264 000 in the year 2000 15. The number of red ants in an ant colony is twice that of
and 3 587 000 in the year 2010. Giving your answers black ants. If the number of black ants increased by 60%,
correct to 3 significant figures, find find the required percentage decrease in the number of
(a) the percentage increase in the population between red ants so that the numbers of red ants and black ants
2000 and 2010, are the same.
(b) the population of the city in the year 2020, assuming
that the current growth rate of population remains Solution
the same for the next 10 years. Let x be the number of black ants.
Number of red ants = 2x
Solution Number of black ants after 60% increase = 160% × x
(a) Percentage increase in population = 1.6x
3 587 000 – 3 264 000
= × 100% Decrease in number of red ants = 2x – 1.6x
3 264 000
= 0.4x
= 9.8958% (correct to 5 s.f.)
= 9.90% (correct to 3 s.f.) 0.4x
Required percentage decrease = × 100%
2x
(a) Population in the year 2020
= 20%
= 3 587 000 × 9.8958%
= 3 940 000 (correct to 5 s.f.)
16. When the price of petrol increases by 20%, a motorist
decreases his volume of petrol consumption by 10%.
14. The total cost of producing a batch of books comprises
Find the percentage increase in his petrol bill.
the following costs:
Typesetting: $4000, Paper: $6000, Printing: $2500.
Solution
Suppose the costs of typesetting and printing increase by
Let $p be the price of petrol per litre. Let v litres be the
15% and 8% respectively, and the cost of paper decreases
amount of petrol consumed.
by 10%.
(a) Find the new cost of Petrol bill before increase = $pv
(i) typesetting, Price of petrol per litre after increase
(ii) paper, = (100 + 20%) × $p
(iii) printing, = $1.2p
(iv) producing the books.
(b) Find the percentage change in the cost of producing Petrol consumed after increase = (100% – 10%) × v
the books. = 0.9v
Petrol bill after increase = $(1.2p × 0.9v)
Solution
= $1.08pv
(a) (i) New cost of typesetting
1.08pv – pv
= $4000 × (100% + 15%) Percentage increase in petrol bill = × 100%
pv
= $4600
= 8%
(ii) New cost of paper
= $6000 × (100% – 10%)
= $5400 Exercise 10.4
(iii) New cost of printing In this exercise, the GST rate is taken as 7%.
= $2500 × (100% + 8%)
= $2700 Level 1
(iv) New cost of producing the books 1. Copy and complete the following table.
= $(4600 + 5400 + 2700) Marked Selling
Discount Discount %
= $12 700 price price
(b) Original cost of producing the books (a) $125 $100 $25 20%
= $(4000 + 6000 + 2500) (b) $70 $49 $21 30%
= $12 500 (c) $240 15%
$204 $36
Increase in cost = $12 700 – $12 500
= $200 (d) $320 $256 $64 20%
200 (e) $450 $351 $99 22%
Percentage increase = × 100%
12 500  
(f ) $500 $345 $155 31%
= 1.6%

75
Solution 4. When a pair of shoes is sold at 12.5% discount, the
(a) Discount
= $125 – $100 selling price is $78.75. Find
= $25 (a) the marked price of the pair of shoes,
25 (b) the amount of discount.
Discount % = × 100%
125
= 20% Solution
(a) Let $M be the marked price.
(b) Selling price = $70 – $21 M × (100% – 12.5%) = 78.75
= $49 78.75
21 M =
0.875
Discount % = × 100%
70
= $90
= 30% The marked price is $90.
(c) Selling price = $240 × (100% – 15%) (b) Amount of discount = $(90 – 78.75)
= $204 = $11.25
Discount = $240 × 15%
= $36
5. Copy and complete the following table.
(d) Marked price = $256 + $64
Price before Amount of
= $320 Marked price
GST GST
64
Discount % = × 100% (a) $210 $200 $10
320
= 20% (b) $378 $360 $18
(c) $105 $100 $5
= Marked price × (100% – 22%)
(e) 351
$351 (d) $493.50 $470 $23.50
Marked price =
0.78 (e) $420 $400 $20
= $450 (f) $546 $520 $26
Discount
= $450 – $351
= $99
Solution
(f ) Marked price = $155 ÷ 31% (a) Marked price = $200 × (100% + 5%)
= $500 = $210
Selling price = $500 – $155 GST = $200 × 5%
= $345 = $10
(b) Marked price = $360 × (100% + 5%)
2. A table is sold at a discount of 20% of its marked price. = $378
Find its selling price if its marked price is $980. GST = $360 × 5%
= $18
Solution
(c) Price before GST = $5 ÷ 0.05
Selling price = $980 × (100% – 20%)
= $100
= $784
Marked price = $100 + $5
3. A rice cooker is sold at a discount of 15%. If the discount = $105
is $24, find
(d) Price before GST = $23.50 ÷ 0.05
(a) its marked price,
= $470
(b) its selling price.
Marked price = $470 + $23.50
Solution
= $493.50
(a) Let $M be the marked price. (e) Price before GST = $420 ÷ 1.05
M × 15% = 24 = $400
M =
24 GST = $420 – $400
0.15 = $20
= 160
(f ) Price before GST = $546 ÷ 1.05
The marked price is $160.
= $520
(b) Selling price = $160 – $24 GST = $546 – $520
= $136 = $26

Chapter 10  Percentage


76
6. An electrician charges Mr Lin $300, excluding GST, for (c) Original price of 4 T-shirts = $30 × 4
wiring his home. = $120
(a) What is the amount of GST that Mr Lin has to pay Selling price of 4 T-shirts = $30 × 3
on the charges? = $90
(b) What is the total amount, inclusive of GST, that 120 – 60
Discount percentage = × 100%
Mr Lin has to pay? 120
= 25%
Solution
(a) Amount of GST = $300 × 5%
= $15 10. At a jewellery store, the manager increases the price of
a diamond ring marked at $3500 by 10%. He then sells
(b) Total amount = $300 + $15 it to a customer at a 10% discount. Find
= $315 (a) the new marked price,
(b) the selling price of the ring,
7. The marked price, inclusive of GST, of a colour laser (c) the actual discount percentage based on the original
printer in a computer shop is $856. Find marked price.
(a) its price before GST,
(b) the amount of GST required. Solution
(a) New marked price = $3500 × (100% + 10%)
Solution = $3850
(a) Price before GST = $856 ÷ 1.07 (b) Selling price = $3850 × (100% – 10%)
= $800 = $3465
3500 – 3465
(b) Amount of GST = $856 – $800 (c) Discount percentage = × 100%
3500
= $40
= 1%

Level 2 11. The marked price, inclusive of GST, of a notebook


8. At a sale in a department store, goods are sold at a 10% computer is $1679.90. It is sold at a 10% discount.
discount. The VIP members of the store are entitled to (a) Find its marked price before GST.
a further discount of 5% on the discounted price. If the (b) Find the original GST on it.
marked price of a sofa set is $1200, how much does a (c) What is the selling price of the computer?
VIP member have to pay? (d) Find the GST after discount.
(e) What is the percentage decrease in the GST?
Solution
Amount a VIP member has to pay Solution
= $1200 × (100% – 10%) × (100% – 5%) (a) Marked price before GST = $1680 ÷ 1.05
= $1026 = $1600
(b) Original GST = $1680 – $1600
9. In a shop, the price of a T-shirt is $30. During a promotion, = $80
a customer can buy two T-shirts and get one free. Find (c) Selling price = $1680 × (100% – 10%)
the actual discount percentage when a customer buys = $1512
(a) 1 T-shirt,
(d) GST after discount = $1512 ÷ 1.05 × 5%
(b) 3 T-shirts,
= $72
(c) 4 T-shirts.
80 – 72
(e) Percentage decrease in GST = × 100%
80
Solution
= 10%
(a) Discount = $0 for one T-shirt.
Discount percentage = 0%
(b) Original price of 3 T-shirts = $30 × 3 Level 3
= $90 12. During a Christmas sale in a department store, electrical
Selling price of 3 T-shirts = $30 × 2 appliances were sold at 5% discount and garments at a
= $60 20% discount storewide. Mrs Li bought a hair dryer with
90 – 60 a marked price of $50 and two skirts at a discounted
Discount percentage = × 100%
90 price of $120 each. Find
1 (a) the selling price of the hair dryer,
= 33 %
3 (b) the marked price of each skirt,
(c) the total amount of discount given to Mrs Li,

77
(d) the average percentage discount on Mrs Li’s 14. Refer to the receipt below issued by a restaurant in which
purchases. the prices listed are not inclusive of GST. Find
(a) the subtotal amount,
Solution (b) the service charge amount,
(a) Selling price of the hair dryer (c) the GST amount,
= $50 × (100% – 5%) (d) the total amount, which is rounded up to the nearest
= $47.50 5 cents.
(b) Marked price of each skirt
= $120 ÷ (100% – 20%)
= $150
(c) Total amount of discount
= $[(50 – 47.50) + 2 × (150 – 120)]
= $62.50
(d) Original price of all the goods = $(50 + 2 × 150)
= $350
62.50
Average percentage discount = × 100%
350
6
= 17 %
7

13. On average, a bakery sells 500 fish-fillet buns at $1.50


each a day. When the price of each bun increases by
20%, the total sales volume drops by 25%.
(a) Find the new price of each bun.
(b) Find the number of buns sold at the new price.
(c) What is the percentage change in the revenue with
Solution
the increase in the price?
(a) Subtotal amount
(d) Find the percentage discount required on the new
= 3.60 + 43.00 + 10.00 + 2.50 + 1.00 + 2.00
price to adjust the price to its original level.
= $62.10
Solution (b) Service charge amount
(a) New price of each bun = $1.50 × (100% + 20%) = $62.10 × 10%
= $1.80 = $6.21
(b) Number of buns sold = 500 × (100% – 25%) (c) GST amount
= 375 = $(62.10 + 6.21) × 7%
(c) Total revenue under new price = $1.80 × 375 = $4.78 (correct to 2 d.p.)
= $675 (d) Total amount
Original revenue = $1.50 × 500 = $62.10 + $6.21 + $4.78
= $750 = $73.09
Percentage decrease in revenue = $73.10 (correct to the nearest 5 cents)
750 – 675
= × 100%
750 15. Refer to the receipt below issued by a supermarket in
= 10% which the prices listed are inclusive of GST. Find
(a) the subtotal amount,
(d) Discount required = $1.80 – $1.50 (b) the total amount,
= $0.30 (c) the rounding adjustment, where the total amount
Required percentage discount is rounded down to the nearest 5 cents,
0.3 (d) the change,
= × 100%
1.8 (e) the total amount exclusive of GST,
= 16 %
2 (f) the GST amount.
3

Chapter 10  Percentage


78
2. A piece of alloy contains 60% copper and 40% zinc by
mass. The mass of the alloy is 25 kg.
(a) Find the mass of copper in the alloy.
(b) If 5 kg of zinc is added to the alloy, find the
percentage of copper by mass in the new alloy.

Solution
(a) Mass of copper = 25 × 60%
= 15 kg
(b) Percentage of copper in new alloy
15
= × 100%
25 + 5
= 50%

3. 26% of ballpoint pens are red, 38% are blue and the
remaining ones are black. There are 57 blue ballpoint
pens in the box.
(a) Find the total number of ballpoint pens in the box.
(b) How many of them are black?
(c) If 10 more blue ballpoint pens were put into the box,
Solution what would the new percentage of blue ballpoint
(a) Subtotal amount pens be?
= 9.25 + 4.88 + 4.12 + 2.50 + 2.20 + 3.55 + 2.19
= $28.69 Solution
(a) Total number of ballpoint pens = 57 ÷ 38%
(b) Total amount = 150
= $28.69 – $0.04
= $28.65 (b) Percentage of black ballpoint pens
= 100% – 26% – 38%
(c) Rounding adjustment = 36%
= –$0.04 Number of black ballpoint pens = 150 × 36%
(d) Change = 54
= $50.70 – $28.65 (c) New percentage of blue ballpoint pens
= $22.05 57 + 10
= × 100%
(e) Total amount exclusive of GST 150 + 10
= $28.69 ÷ 107% 67
= × 100%
= $26.81 (correct to 2 d.p.) 160
= 41.875%
(f) GST amount
= $28.69 – $26.81
= $1.88 4. 30% of the members of a club are female and 20% of
them own cars. 30 female members in the club own cars.
(a) How many female members are there?
Revision Exercise 10 (b) What is the total number of members?
1. In a particular month, Mr Lin spent $900 which was (c) How many new female members have to be
30% of his monthly salary on food. recruited to increase the percentage of female
(a) Find his monthly salary. members to 37.5%?
(b) If he saved 12% of his monthly salary, find his
monthly savings. Solution
(a) Number of female members = 30 ÷ 20%
Solution = 150
(a) His monthly salary = $900 ÷ 30% (b) Total number of members = 150 ÷ 30%
= $3000 = 500
(b) His monthly savings = $3000 × 12%
= $360

79
(c) Let n be the number of new female members 8. The cost of making a table is $600, of which 40% is the
required. labour cost and the remaining is the material cost.
150 + n = (500 + n) × 37.5% (a) Find the cost of
150 + n = 187.5 + 0.375n (i) labour,
0.625n = 37.5 (ii) material used.
n = 60 (b) If the cost of labour is increased by 15% while
60 new female members have to be recruited. the cost of material is decreased by 20%, find the
percentage change in the cost of making the table.
5. Huaming earned $36 000 in the year 2010 and $38 880
Solution
in the year 2011.
2
(a) Find the percentage increase in his annual income (a) (i) Cost of labour = $600 × = $240
2+3
in the year 2011. 2
(b) If Huaming’s income in the year 2010 was 10% (ii) Cost of material used = $600 ×
2+3
less than that of the previous year, what was his = $360
annual income in the year 2009?
(b) New cost of labour = $240 × (100% + 15%)
Solution = $276
38 880 – 36 000 New cost of material used = $360 × (100% – 20%)
(a) Percentage increase = × 100% = $288
36 000
2880
New cost of making the table = $(276 + 288)
= × 100% = $564
36 000
= 8% Percentage decrease in the cost of making the table
600 – 564
(b) Income in the year 2009 = × 100%
600
= $36 000 ÷ (100% – 10%) 36
= $40 000 = × 100%
600
= 6%
6. Mr Wong’s monthly expenditure was $2125 after he had
reduced it by 15%. The original monthly expenditure was 9. At a clearance sale of a department store, handbags were
80% of his monthly income. Find sold at 40% discount and skirts at 25% discount. Jenny
(a) the original monthly expenditure, bought 2 handbags for $144 each and 3 skirts which
(b) his monthly income. were marked at $60 each. Find
(a) the marked price of each handbag,
Solution (b) the selling price of each skirt,
(a) Original monthly expenditure (c) the total discount on Jenny’s purchases,
= $2125 ÷ (100% – 15%) (d) the overall percentage discount,
= $2500 (e) the total amount of the GST on the handbags and
(b) Monthly income = $2500 ÷ 80% skirts bought, given that the GST rate is 7%.
= $3125
Solution
(a) Marked price for each handbag
7. Mrs Raj’s mass increased by 20% to 66 kg. = $144 ÷ (100% – 40%)
(a) Find her original mass. = $240
(b) If her mass dropped by 15%, find her new mass.
(c) What percentage is her new mass of her original (b) Selling price of each skirt
mass? = $60 × (100% – 25%)
= $45
Solution (c) Total marked price = $(240 × 2 + 60 × 3)
(a) Original mass = 66 ÷ (100% + 20%) = $660
= 55 kg Total selling price = $(144 × 2 + 45 × 3)
(b) New mass = 66 × (100% – 15%) = $423
= 56.1 kg Total discount = $660 – $423
56.1
= $237
(c) Required percentage = × 100% 237
55
(d) Overall percentage discount = × 100%
= 102% 660
10
= 35 %
11

Chapter 10  Percentage


80
(e) Total amount of GST = $423 × 7% (ii) Let $m billion be the amount of tourists
= $29.61 spending in the first quarter of 2010.
m × (1 + 35.7%) = 4.98
4.98
10. Refer to the news on 22 June 2011 below. m =
1.357
The Certificate of Entitlement (COE) premium for cars
= 3.67  (correct to 3 s.f.)
above 1600cc (Category B) is at $67 700, an increase
\  The required amount is $3.67 billion.
of $4700 (from $n), or y per cent.
Premiums for cars of up to 1600cc (Category A) lowered (b) The growth may be due to the opening of new
by 5.89 per cent to $50 244 (from $z). casinos and integrated resorts.
(a) Find the value of n.
(b) Find the value of y, correct to 3 significant figures.
12. Refer to the news on 1 July 2011 below.
(c) Find the value of z, correct to the nearest integer.
The price index of private homes rose 1.9 per cent to
202.8 points in the second quarter of 2011, compared
Solution
with the 2.2 per cent increase in the previous quarter.
(a) n = 67 700 – 4700
(a) Find the price index of private homes in
= 63 000
(i) the first quarter of 2011,
4700 (ii) the fourth quarter of 2010.
(b) y% = × 100%
6300 (b) Find the percentage increase in the price index from
= 7.46% (correct to 3 s.f.) the fourth quarter of 2010 to the second quarter of
 y = 7.46 2011.
(c) z × (1 – 5.89%) = 50244 (c) Is the result in (b) equal to the sum of 1.9% and
50244
2.2%?
z = Give your answers correct to 1 decimal place.
0.9411
= 53 389 (correct to the
Solution
nearest integer)
(a) (i) Price index in the first quarter of 2011
= 202.8 ÷ (1 + 1.9%)
11. The Singapore Tourism Board’s quarterly report released = 199.02
on 2 June 2011 mentioned the following. = 199.0 (correct to 1 d.p.)
The tourism sector posted strong growth across the board
in the first quarter of this year, with visitor arrivals hitting (ii) Price index in the fourth quarter of 2010
record highs. The 3.12 million tourists, up 15.7 per cent = 199.02 ÷ (1 + 2.2%)
from the same period last year, also spent 35.7 per cent = 194.74
more. Total tourism receipts amounted to $4.98 billion, = 194.7 (correct to 1 d.p.)
with the sightseeing and entertainment component, which (b) The required percentage increase
includes gaming at the integrated resorts, posting the = (202.8-194.74)/194.74 × 100%
highest growth of 321 per cent. = 4.1389%
(a) Giving your answers correct to 3 significant figures, = 4.1% (correct to 1 d.p.)
estimate from the above report for the first quarter
(c) 1.9% + 2.2% = 4.1%
of 2010
≠ 4.1389%
(i) the number of tourists,
\ the result in (b) is not equal to the sum of 1.9%
(ii) the total amount of tourism receipts.
and 2.2%.
(b) Suggest a reason for the growth in tourism.

Solution 13. Refer to the receipt below issued by a restaurant. Find


(a) (i) Let n millions be the number of tourists in (a) the subtotal amount,
the first quarter in 2010. (b) the service charge amount,
n × (1 + 15.7%) = 3.12 (c) the GST amount,
3.12
n =
1.157
= 2.70  (correct to 3 s.f.)
\ The required number of tourists is 2.70
million

81
(d) the total amount, which is rounded up to the nearest
5 cents.

Solution
(a) Subtotal amount
= 8.80 + 5.00 + 14.80 + 5.80 + 8.00 + 2.40 + 0.80
= $45.60
(b) Service charge amount
= $45.60 × 10%
= $4.56
(c) GST amount
= ($45.60 + $4.56) × 7%
= $3.51 (correct to 2 d.p.)
(d) Total amount
= 45.60 + 4.56 + 3.51
= 53.67
= $53.70 (correct to the nearest 5 cents)

Chapter 10  Percentage


82
11 Number Patterns

Class Activity 1
Objective: To recognise number patterns.

Questions
1. In each of the following cases, the first four terms of a sequence are given. Find the 5th and 6th terms of each sequence.

(a) 2, 5, 8, 11, 14 , 17 , ...

(b) 30, 25, 20, 15, 10 , 5 , ...

(c) 1, 3, 9, 27, 81 , 243 , ...

(d) 96, 48, 24, 12, 6 , 3 , ...

2. Look at the following patterns of dots.

Let an be the number of green dots and Tn be the total number of dots in the nth pattern of the diagram.

(a) Copy the following table and write down the terms T1 , T2 , T3 and T4 in your table.

Figure number, n Total number of dots in Figure n, Tn Number of green dots in Figure n, an
1 1 1

2 4 3

3 9 5

4 16 7

(b) What will T5 and T6 be?

T5 = 52
= 25

T6 = 62
= 36

83
(c) What pattern do you observe? Describe it.

Tn = n 2, which is a square number. Tn is a series of square numbers 12, 22, 32, …, n 2, … .

(d) What will T11 be?

T11 = 112
= 121

(e) Write down the terms a1, a2, a3 and a4 in your table above.

(f) What will a5 and a6 be?

a5 = 7 + 2
= 9

a6 = 9 + 2
= 11

(g) How are the values of n and an related in the sequence?

an = 2n – 1, which is an odd number. an is a sequence of odd numbers 1, 3, 5, 7, …, 2n – 1, … .

Chapter 11  Number Patterns


84
Class Activity 2
Objective: To recognise number patterns in a real-world situation.

A restaurant uses square tables where each of them can seat 4 customers with a chair on each side (see
Figure 1). Tables can be placed together to form a long rectangular table to handle larger groups of customers (see Figure 2
and Figure 3).

Figure 1 Figure 2 Figure 3

Questions
1. Draw a diagram showing the arrangement with chairs when 4 tables are placed together.

Figure 4

2. Copy and complete the following table.

Figure number, n Number of tables in Figure n Number of chairs in Figure n, Cn


1 1 4
2 2 4+2=6
3 3 4 + 2 + 2 = 8
4 4 4 + 2 + 2 + 2 = 10

5 5 4 + 2 + 2 + 2 + 2 = 12

6 6 4 + 2 + 2 + 2 + 2 + 2 = 14

85
3. What pattern do you observe about Cn in the table above? Explain your answer.

The rule is to add 2 to the previous term to get to the next term.

4. Write down the number of chairs available when 18 square tables are placed together.

C18 = 4 + (18 – 1) × 2
= 38

5. How are the values of n and Cn related? Express Cn in terms of n.

Cn = 4 + (n – 1) × 2
= 2n + 2

Chapter 11  Number Patterns


86
Try It! Suppose each stick is 3 cm long and the matchsticks are
arranged as shown. Let the number of matchsticks in the
Section 11.1
nth pattern be Tn and the perimeter of the nth pattern be
1. Find the next two terms of the following sequences. Pn cm.
(a) 2, 6, 10, 14, ...
(b) 7, 1, –5, –11, ... (a) Complete the following table.
Solution Number of
Perimeter
(a) 2, 6, 10, 14, … Figure matchsticks
of nth
A term is obtained by adding 4 to the previous number, n in the nth
pattern, Pn
term. pattern, Tn
The 5th term = 14 + 4 1 4 4 × 3 = 12
= 18
The 6th term = 18 + 4 2
= 22 3
(b) 7, 1, –5, –11, … 4
A term is obtained by adding –6 to the previous
term. (b) Find T5 , P5 , T20 , P20 .
The 5th term = –11 + (– 6)
= –17 Solution
The 6th term = –17 + (– 6) (a)
= –23 Number of
Perimeter
Figure matchsticks
of nth
2. Find the next two terms of the following sequences. number, n in the nth
pattern, Pn
(a) 2, 6, 18, 54, ... pattern, Tn
(b) 625, –250, 100, – 40, ... 1 4 4 × 3 = 12
Solution 2 4+3=7 6 × 3 = 18
(a) 2, 6, 18, 54, … 3 4 + 3 + 3 = 10 8 × 3 = 24
A term is obtained by multiplying the previous term 4 4 + 3 + 3 + 3 = 13 10 × 3 = 30
by 3.
The 5th term = 54 × 3
= 162 (b) T5 = 4 + 3(5 – 1)
The 6th term = 162 × 3 = 16
= 486 P5 = 2 × (5 + 1) × 3
(b) 625, –250, 100, – 40, …   = 36
A term is obtained by multiplying the previous T20 = 4 + 3(20 – 1)
2 = 61
term by – .
5
P20 = 2 × (20 + 1) × 3
2
The 5th term = – 40 ×  
– = 126
5
= 16
2
The 6th term = 16 × – Section 11.2
5
4. The general term of a sequence is Tn = n(n + 3). Find its
= – 6.4
2nd term and 9th term.

3. In the diagram below, study and compare the matchstick Solution
patterns. Tn = n(n + 3)
T2 = 2(2 + 3)
= 10
T9 = 9(9 + 3)
= 108

87
5. The diagram below shows a sequence of dot patterns 7. In a multi-storey building, the first floor is 5 m above
where n is the figure number. Let Tn be the number of the ground level and each subsequent floor is 3 m above
dots in the nth pattern. the previous floor.
(a) How high is the 5th floor above the ground level?
The diagram below shows a sequence of dot patterns.
(b) Express the height of the nth floor above the ground
level in terms of n.
(c) When you are 155 m above the ground level, which
floor are you on?

Solution
(a) 17 metres
(b) Height of nth floor = 5 + (n – 1) × 3
= 5 + 3n – 3
Figure number = 3n + 2
(c) Height of nth floor = 155
(a) Write down the number of dots for Figures 4 and
155 = 3n + 2
5 in the table below.
3n = 153
Number of dots in n = 51
Figure number, n
Figure n, Tn I would be at the 51st floor.
1 1
2 1+4=5
3 1+4+4=9
4 1 + 4 + 4 + 4 = 13
5 1 + 4 + 4 + 4 + 4 = 17

(b) Find the general term Tn of the sequence.


(c) Find the 18th term T18.

Solution
(a) The sequence of Tn is 1, 5, 9, 13, 17, …
The general term Tn = 1 + (n – 1) × 4
= 1 + 4n – 4
= 4n – 3
(b) The 18th term T18 = 4(18) – 3
= 69

6. Consider the sequence 4, 7, 10, 13, … .


(a) Find its general term.
(b) Hence, find its 15th term.

Solution
(a) 4, 7, 10, 13, …
The general term Tn = 4 + (n – 1) × 3
= 4 + 3n – 3
= 3n + 1
(b) T15 = 3(15) + 1
= 46

Chapter 11  Number Patterns


88
xercise 11.1
E (d) – 4, 12, –36, 108, …
Level 1 T5 = 108 × (–3)
1. Write down the next two terms of each sequence. = –324
(a) 11, 13, 15, 17, … T6 = –324 × (–3)
(b) 1, 4, 7, 10, … = 972
(c) 16, 12, 8, 4, …
(d) 49, 38, 27, 16, … 3. Write down the next two terms of each sequence.
(a) 1, 4, 9, 16, …
Solution (b) 1, 3, 6, 10, …
(a) 11, 13, 15, 17, …
1 1 1
T5 = 17 + 2 (c) 1, , , ,…
2 4 8
= 19 1 2 3 4
T6 = 19 + 2 (d) , , , , …
2 3 4 5
= 21
(b) 1, 4, 7, 10, … Solution
T5 = 10 + 3 (a) 1, 4, 9, 16, …
= 13 The sequence can be written as
T6 = 13 + 3 12, 22, 32, 42, …
= 16 T5 = 52
= 25
(c) 16, 12, 8, 4, …
T6 = 62
T5 = 4 – 4
= 36
= 0
T6 = 0 – 4 (b) 1, 3, 6, 10, ...
= – 4 The sequence can be written as
(d) 49, 38, 27, 16, … 1, 1 + 2, 1 + 2 + 3, 1 + 2 + 3 + 4, …
T5 = 16 – 11 T5 = 1 + 2 + 3 + 4 + 5
= 5 = 15
T6 = 5 – 11 T6 = T5 + 6
= – 6 = 15 + 6
= 21

2. Write down the next two terms of each sequence. 1 1 1


(a) 1, 2, 4, 8, … (c) 1, , , , …
2 4 8
(b) 375, 75, 15, 3, …
The sequence can be written as
(c) 1, –1, 1, –1, …
(d) – 4, 12, –36, 108, … 1, 1 × , 1 ×
1 1
×
1
, 1 ×
1
×
1
×
1
, …
2 2 2 2 2 2
Solution 1 1
T5 = ×
(a) 1, 2, 4, 8, … 8 2
T5 = 8 × 2 1
=
= 16 16
T6 = 16 × 2 T6 = T5 ×
1
= 32 2
1 1
(b) 375, 75, 15, 3, … = ×
16 2
1 1
T5 = 3 × =
5 32
3
=
5
1 2 3 4
3 1 (d) , , , , …
T 6 = × 2 3 4 5
5 5
3 The sequence can be written as
=
25
1 1+1 1+1+1 1+1+1+1
, , , , …
(c) 1, –1, 1, –1, … 2 2+1 2+1+1 2+1+1+1
T5 = –1 × (–1) T5 =
5
= 1 6
T6 = 1 × (–1) T6 =
6
= –1 7

89
Level 2 7. A sequence is formed by 2 × 12, 2 × 22, 2 × 32,
4. Find the 7th term of each sequence. 2 × 42, …
(a) 2, 4, 6, 8, … (a) Write down the first 4 terms of the sequence.
(b) 23, 20, 17, 14, … (b) Find the 8th term of the sequence.
(c) 5, 10, 20, 40, …
(d) 216, –144, 96, – 64, … Solution
(a) 2 × 12, 2 × 22, 2 × 32, 2 × 42, …
Solution T1 = 2 × 12
(a) 2, 4, 6, 8, … = 2
T5 = 8 + 2 = 10 T2 = 2 × 22
T6 = 10 + 2 = 12 = 8
T3 = 2 × 32
T7 = 12 + 2 = 14
= 18
(b) 23, 20, 17, 14, … T4 = 2 × 42
T5 = 14 – 3 = 11 = 32
T6 = 11 – 3 = 8 (b) T8 = 2 × 82
T7 = 8 – 3 = 5 = 128

(c) 5, 10, 20, 40, …


T5 = 40 × 2 = 80 Level 3
T6 = 80 × 2 = 160 8. A sequence is formed by 1 × 2, 2 × 3, 3 × 4, 4 × 5, …
T7 = 160 × 2 = 320 (a) Find the values of the first 4 terms of the sequence.
(b) Find the 10th term of the sequence.
(d) 216, –144, 96, –64, …
2 128 Solution
T5 = – 64 × – =
3 3 (a) 1 × 2, 2 × 3, 3 × 4, 4 × 5, …
128 2 256 T1 = 1 × 2
T6 = × – =– = 2
3 3 9
256 2 T2 = 2 × 3
T7 = – × – = 6
9 3
T3 = 3 × 4
512
= = 12
27
T4 = 4 × 5
= 20
5. The first 4 terms of a sequence are 1, 8, 27 and 64. Write
(b) T10 = 10 × 11
down the 5th term and the 6th term of the sequence.
= 110
Solution
1, 8, 27, 64, … 9. In the diagram, the ladder has 7 rungs. The lengths of
The sequence can be written as the lowest three rungs are 45 cm, 39 cm and 33 cm as
13, 23, 33, 43, … shown. Assume that the lengths from the lowest rung to
T5 = 53 the highest rung follow an arithmetic sequence. Find the
= 125 length of the highest rung.
T6 = 63
= 216

6. In a sequence, the 1st term is 17. Each subsequent term


is 4 more than the previous term. Write down the 3rd
term and the 4th term of the sequence.

Solution
T1 = 17
T2 = 17 + 4
= 21
T3 = 17 + 4 + 4
= 25
T4 = 17 + 4 + 4 + 4
= 29

Chapter 11  Number Patterns


90
3
Solution 3
a3 = × 320
T1 = 45 4
T2 = 45 – 6 3
= × 180
= 39 4
T3 = 39 – 6 = 135
= 33
(b) Determine the value of a6.
T4 = 33 – 6 6
= 27 3
a6 = × 320
4
T5 = 27 – 6
= 19 = 57.0  (correct to 3 s.f.)
T6 = 19 – 6
= 13 12. The diagram shows a sequence of figures formed by
T7 = 13 – 6 small square tiles, where n is the figure number.
= 7
The length of the highest rung is 7 cm.

10. The first row in a cinema has 20 seats. Each subsequent


row has 2 seats more than the row in front of it. Find
(a) the number of seats in the 6th row, (a) Draw the figure for n = 5.
(b) the total number of seats in the first 6 rows. (b) If Tn is the number of small tiles in the nth figure,
find T1, T2, T3, T4 and T5.
(c) If each tile is of side 2 cm and Pn cm is the perimeter
of the nth figure, find Pn for n = 1, 2, 3, 4 and 5.

Solution
(a) The 5th figure is as shown below.
Solution
(a) 30 seats
(b) Total number of seats
= 20 + 22 + 24 + 26 + 28 + 30
= 150 (b) By counting the tiles,
T1 = 4
11. A ball is dropped onto a floor from a height of 320 cm. T2 = 6
3 T3 = 8
Each time it falls, it rebounds to of the height from T4 = 10
4
where it fell. Let an be the height of the nth rebound in T5 = 12
centimetre.
(c) P1 = 10 × 2
= 20
P2 = 20 + 4
= 24
P3 = 24 + 4
= 28
P4 = 28 + 4
= 32
P5 = 32 + 4
(a) Find the values of a1, a2 and a3. = 36
(b) Determine the value of a6.
13. The diagram shows a sequence of huts formed by
Solution matchsticks, where n is the figure number.
3
(a) a1 = × 320
4
= 240
2
3
a2 = × 320
4
3
= × 240
4
= 180

91
The matchsticks, of length 3 cm each, are arranged end to (b) The next 3 terms are
end. If Tn is the number of matchsticks in the nth figure T8 = 8 + 13
and Pn is its respective perimeter, find = 21
(a) Tn for n = 1 to 5, (b) T8, T9 = 13 + 21
(c) Pn for n = 1 to 5, (d) P8. = 34
T10 = 21 + 34
Solution = 55
(a) By counting, (c)
T1 = 5
T2 = 9
T3 = 13
T4 = 17
T5 = 21 Ignoring the first term, the difference between
successive terms of the Fibonacci sequence is the
(b) T8 = 21 + 4 + 4 + 4
Fibonacci sequence itself.
= 33
(c) P1 = 5×3
= 15 Exercise 11.2
P2 = 8×3 Level 1
= 24
1. Find the first 3 terms of each sequence from the given
P3 = 11 × 3
general term Tn.
= 33
(a) Tn = 2n + 1 (b) Tn = 7 – 3n
P4 = 14 × 3
n
= 42 (c) Tn = 2(n – 1)2 (d) Tn =
n+2
P5 = 17 × 3
= 51 Solution
(a) Tn = 2n + 1 (b) Tn = 7 – 3n
(d) P8 = 51 + 9 + 9 + 9 T1 = 7 – 3(1)
T1 = 2(1) + 1
= 78 = 4
= 3
T2 = 2(2) + 1 T2 = 7 – 3(2)
14. The sequence 1, 1, 2, 3, 5, 8, 13, … is known as the = 5 = 1
Fibonacci sequence. T3 = 2(3) + 1 T3 = 7 – 3(3)
(a) What is the rule used to obtain the terms in the = 7 = –2
sequence?
n
(b) Write down the next 3 terms of the sequence. (c) Tn = 2(n – 1)2 (d) Tn =
n+2
(c) Complete the following where each term in the T1 = 2(1 – 1)2
lower row is the difference of the two terms just 1
= 0 T1 =
2 1+2
above it. T2 = 2(2 – 1)
= 2 1
=
T3 = 2(3 – 1) 2 3
2
= 8 T2 =
2+2
1
=
Write a paragraph to explain what you have 2
discovered. 3
T3 =
3+2
Note: You may access the website www.starpub.com.sg/
dm/s1e/weblinks_1b.html for more information 3
=
regarding this famous sequence. 5

Solution 2. The general term of a sequence is Tn = n(n + 2).


(a) 1, 1, 2, 3, 5, 8, 13, … Find the 11th term.
In the Fibonacci sequence, starting from the
Solution
3rd term, each term is equal to the sum of the two
Tn = n(n + 2)
preceding terms, whereas the first two terms are
each equal to 1. T11 = 11(11 + 2)

= 143

Chapter 11  Number Patterns


92
3. The general term of a sequence is Tn = n3 – 1. Find the (ii) 4, 16, 36, 64, 100, …
7th term. (iii) 16, 36, 64, 100, 144, …

Solution Solution
Tn = n3 – 1 (a) Tn = 2n
T7 = 73 – 1 (b) (i) The sequence 3, 5, 7, 9, 11, …
= 342 can be written as
n
1 + 2, 1 + 4, 1 + 6, 1 + 8, 1 + 10, …
2 \  Tn = 1 + 2n
4. The general term of a sequence is Tn = 108 × . Find
3
the 3rd term. (ii) The sequence 4, 16, 36, 64, 100, …
can be written as
Solution 4(1)2, 4(2)2, 4(3)2, 4(4)2, 4(5)2, …
n
2 \  Tn = 4n2
Tn = 108 ×
3
3 (iii) The sequence 4, 16, 36, 64, 100, …
2
T3 = 108 × = 32 can be written as
3
4(1 + 1)2, 4(2 + 1)2, 4(3 + 1)2, …
\  Tn = 4(n + 1)2
Level 2
5. The general term of a sequence is Tn = 7n + 4. Find the 9. The general term of a sequence is Tn = 2n2.
sum of its 5th term and 6th term. (a) Write down the first 4 terms of the sequence.
(b) The first 4 terms of another sequence are 5, 11, 21,
Solution 35.
Tn = 7n + 4 (i) Suggest a formula for the general term of the
T5 = 7(5) + 4 = 39 new sequence.
T6 = 7(6) + 4 = 46 (ii) Find the 6th term of the new sequence.
The required sum = T5 + T6 Solution
= 39 + 46 (a) T1 = 2(1)2
= 85 = 2
T2 = 2(2)2
6. Consider the sequence 39, 37, 35, 33, … . = 8
(a) Find its general term Tn in terms of n. T3 = 2(3)2
(b) Hence, find the 18th term. = 18
T4 = 2(4)2
Solution = 32
(a) 39, 37, 35, 33, … (b) The sequence 5, 11, 21, 35, …
Tn = 39 + (n – 1) × (–2) can be written as
= 41 – 2n 2 + 3, 8 + 3, 18 + 3, 32 + 3 , …
(b) T18 = 41 – 2(18) (i) Tn = 2n2 + 3
= 5
(ii) T6 = 2(6)2 + 3
= 75
7. Express the nth term of the sequence 8, 27, 64, 125, ...
in terms of n.
10. Consider the sequence 4, 10, 18, 28, … . Ali and Sumin
Solution observe different number patterns as shown in the table.
The sequence 8, 27, 64, 125, …
can be written as Ali’s Sumin’s
Term, Tn
(1 + 1)3, (2 + 1)3, (3 + 1)3, (4 + 1)3, … pattern pattern
\  Tn = (n + 1)3 1 1×4 12 + 3 × 1
2 2×5 22 + 3 × 2
8. (a) For the sequence 2, 4, 6, 8, 10, … , express the nth
3 3×6 32 + 3 × 3
term in terms of n.
(b) Hence, express the nth term of each of the following 4 4×7 42 + 3 × 4
sequences in terms of n.
(i) 3, 5, 7, 9, 11, ...

93
(a) Find the general term an based on Ali’s number (a) Observe the patterns. Copy and complete the
patterns. following table.
(b) Find the general term Sn based on Sumin’s number
patterns. Figure Number of Total number
(c) Is an = Sn? Explain your answer. number, n red dots of dots
1 1 1×2
Solution 2 1+2 2×3
(a) an = n × (n + 3) = n(n + 3) 3
(b) Sn = n2 + 3 × n = n2 + 3n 4
(c) Since n(n + 3) = n2 + 3n, an = Sn. 5

(b) What is the ratio of the number of red dots to the
11. The diagram below shows a sequence of patterns. total number of dots in each pattern?
(c) Hence, find a formula in terms of n, for the sum
1 + 2 + 3 + … + n.
(d) Evaluate the sum 51 + 52 + 53 + … + 200 using
the result in (c).

Solution
(a) Observe the patterns. What are the missing numbers
  (a) Figure Number of Total number
in the brackets?
number, n red dots of dots
1 = 12 1 1 1×2
1 + 3 = (  )2 2 1+2 2×3
1 + 3 + 5 = (  )2 3 1+2+3 3×4
1 + 3 + 5 + 7 = (  )2 4 1+2+3+4 4×5
5 1+2+3+4+5 5×6
(b) Express the sum 1 + 3 + 5 + 7 + 9 as a square
number. Number of red dots 1
(b) = =1:2
(c) Express the sum 1 + 3 + 5 + … + (2n – 1) as a Total number of dots 2
square number, where n is a positive integer. (c) For the nth pattern,
number of red dots = 1 + 2 + 3 + … + n
Solution
total number of dots = n(n + 1).
(a) 1 = 12
From the result in (b), we have
1 + 3 = (2)2
1
1 + 3 + 5 = (3)2 1 + 2 + 3 + … + n = n(n + 1).
2
1 + 3 + 5 + 7 = (4)2
1
(d) 1 + 2 + 3 + … + 200 = × 200 × (200 + 1)
(b) 1 + 3 + 5 + 7 + 9 = 52 2

(c) 1 + 3 + 5 + … + (2n – 1) = n2 = 20 100


1
1 + 2 + 3 + … + 50 = × 50 × (50 + 1)
2
12. The diagram below shows a sequence of rectangular = 1275
array of dots. ∴ 51 + 52 + 53 + … + 200 = 20 100 – 1275
= 18 825

Level 3
13. Study the rectangles formed by small square tiles in the
diagram.

Chapter 11  Number Patterns


94
Suppose each side of the small tile is 2 cm and Pn cm is Solution
the perimeter of the nth rectangle. (a) T2 = 5 + 11
(a) Find P1, P2 and P3. = 16
(b) State the general term Pn.
T3 = 16 + 11
(c) Hence, find the perimeter of the 15th rectangle.
= 27
Solution T4 = 27 + 11
(a) P1 = 6×2 = 38
= 12
(b) Tn = 5 + (n – 1) × 11
P2 = 10 × 2
= 5 + 11n – 11
= 20
= 11n – 6
P3 = 14 × 2
= 28
16. (a) Design a sequence of tiling patterns using small
(b) Pn = 12 + (n – 1) × 8 square tiles as your building blocks.
= 8n + 4 (b) Describe your pattern in words.
(c) P15 = (8 × 15 + 4) (c) Let Tn be the number of tiles in the nth pattern.
= 124 Find a formula for Tn.
The perimeter of the 15th rectangle is 124 cm.
Solution
(a) A simple sequence of tiling patterns is as shown
14. The diagram below shows a sequence of tiling patterns below.
formed by using small white and red tiles, where n is
the pattern number.

(b) The first pattern has 1 tile.


For n  2, the nth pattern is formed by adding
a tile to the right-end and a tile to the top of the
(a) Rn is the number of red tiles in the nth square. Find (n – 1)th pattern.
(i) R4, (ii) Rn. (c) Tn = 1 + (n – 1) × 2
(b) Wn is the number of white tiles in the nth square. = 2n – 1
Find
(i) W4, (ii) Wn.

Solution Revision Exercise 11
(a) (i) R4 = 4 × 4 (ii) Rn = n × n 1. Find the next two terms in each sequence.
= 16 = n2 (a) 25, 27, 29, 31, ...
(b) 41, 35, 29, 23, ...
(b) (i) W4 = 62 – 42
(c) 1, 4, 16, 64, ...
= 20
(d) 5, –10, 20, – 40, ...
(ii) Wn = (n + 2)2 – n2
= n2 + 4n + 4 – n2 Solution
= 4n + 4 (a) T5 = 31 + 2
= 33
T6 = 33 + 2
15. The diagram shows part of a staircase. The height between
= 35
the ground and the first step is 5 cm. The height between
any two steps on a staircase is 11 cm. Let Tn be the height (b) T5 = 23 – 6
of the nth step above the ground in centimetre. = 17
(a) Find the values of T2, T3 and T4. T6 = 17 – 6
(b) Express Tn in terms of n. = 11
(c) T5 = 64 × 4
= 256
T6 = 256 × 4
= 1024

95
(d) T5 = – 40 × (–2) Solution
= 80 (a) Since
T6 = 80 × (–2) 1 1
19 – 23 = 16 – 19
= –160 2 2
1
= 12 – 16
1 2
2. If the general term Tn of a sequence is Tn = , find
2n + 1 1
the sum of the first 3 terms. = –3 ,
2
1
Solution T5 = 12 – 3
2
Sum of the first 3 terms 1
T1 + T2 + T3 =8
2
1 1 1
= + + 1
2(1) + 1 2(2) + 1 2(3) + 1 T 6 = 9 – 3
2
1 1 1 1
= + + =4
3 5 7 2
35 + 21 + 15
= (b) Since
105
9 – 4
71
= = 14 – 9
105
= 19 – 14
= 5
3. Find the general term of each sequence. the nth term = 4 + (n – 1) × 5
(a) 2, 3, 4, 5, … = 4 + 5n – 5
(b)
1 2 3 4
, , , , … = 5n – 1
4 5 6 7
(c) 1 × 2, 2 × 3, 3 × 4, 4 × 5, …
6. The first 5 terms of a sequence are 3, 9, 27, 81 and 243
respectively.
Solution
(a) Find the next two terms of the sequence.
(a) 2, 3, 4, 5, …
(b) Express the nth term of the sequence in terms of
Tn = n + 1
n.
1 2 3 4 (c) Hence, find the nth term of the sequence
(b) , , , , …
4 5 6 7 5, 11, 29, 83, 245, ... in terms of n.
n
Tn =
n+3 Solution
(a) The sequence
(c) 1 × 2, 2 × 3, 3 × 4, 4 × 5, … 3, 9, 27, 81, 243, …
Tn = n(n + 1) can be written as
3, 32, 33, 34, 35, …
4. For the sequence 3, 8, 15, 24, 35, … , the nth term is \  the 6th term = 36
n(n + 2). Find the 31st term. = 729
the 7th term = 37
Solution
= 2187
T31 = 31(31 + 2)
= 1023 (b) The nth term = 3n
(c) The sequence
5. (a) Find the next two terms of the sequence 5, 11, 29, 83, 245, …
1 1 can be written as
23, 19 , 16, 12 , … .
2 2 3 + 2, 9 + 2, 27 + 2, 81 + 2, 243 + 2, …
(b) Express the nth term of the sequence
\ the nth term = 3n + 2
4, 9, 14, 19, … in terms of n.

7. Consider the following pattern.


1 × 5 + 4 = 9
2 × 6 + 4 = 16
3 × 7 + 4 = 25
4 × 8 + 4 = 36
     

Chapter 11  Number Patterns


96
(a) Write down the equation for 9. Study the square patterns formed by matchsticks in the
(i) the 5th line of the pattern, diagram where n is the pattern number. Let Tn be the
(ii) the nth line of the pattern. number of matchsticks in the nth pattern.
(b) If k is a positive integer such that
k(k + 4) = 1085,
find the value of k using the answer of (a)(ii).

Solution
(a) (i) The 5th line is
5 × 9 + 4 = 49
(ii) The nth line is
(a) Copy and complete the following table.
n × (n + 4) + 4 = (n + 2)2
(b) When k(k + 4) = 1085, n 1 2 3 4
k(k + 4) + 4 = 1085 + 4 Tn
= 1089
(b) Express Tn in terms of n.
From (a)(ii), (c) How many matchsticks are required for a 10 × 10
(k+2)2 = 1089 square pattern?
k + 2 = 33
k = 31
Solution

8. Consider the patterns formed by green and white tiles in   (a) n 1 2 3 4


the diagram below. Tn 4 12 24 40

(b) In the nth pattern, there are (n + 1) rows and


(n + 1) columns of n matchsticks each.
∴ Tn = n(n + 1) + n(n + 1)
= 2n(n + 1)
(a) Draw the pattern for n = 4.
(c) The required number of matchsticks for a 10 × 10
(b) Let Gn be the number of green tiles and Wn be the
square pattern = T10
number of white tiles in the nth pattern. Copy and
= 2 × 10 × (10 + 1)
complete the following table.
= 220
n 1 2 3 4
Gn 10. The diagram shows a sequence of triangle patterns, where
Wn n is a pattern number. Each small triangle in a pattern is
the same size as the small triangle for n = 1.
(c) Find the formula in terms of n for the general term.
(i) Gn,
(ii) Wn.

Solution
(a) The diagram below shows the pattern for n = 4.

Tn is the number of small triangles in the nth pattern.


Pn cm is the perimeter of the nth pattern. Suppose
sP1 = 7.
(a) Draw the 4th pattern.
(b) Copy and complete the following table.
  (b) n 1 2 3 4 n 1 2 3 4
Gn 7 8 9 10 Tn
Wn 2 4 6 8 Pn

(i) Gn = n + 6 (c) Find the formula in terms of n for the general term
(i) Tn, (ii) Pn,
(ii) Wn = 2n

97
Solution
(a) The diagram below shows the 4th pattern.

(b) n 1 2 3 4
Tn 1 4 9 16
Pn 7 14 21 28

(c) (i) Tn = n2

(ii) Pn = 7n

Chapter 11  Number Patterns


98
12 Coordinates and Linear
Functions

Class Activity 1
Objective: To specify and plot points on a two-dimensional Cartesian plane in a battleship game.

Tasks
1. Prepare your mathematical ocean on a graph paper. It is a two-dimensional Cartesian plane where both the values of x
and y range from –5 to 5.

2. You are given four battleships. The names and the lengths of these battleships are as follows:
Aircraft Carrier (denoted by AAAAA) – 5 units Cruiser (denoted by CCCC) – 4 units
Destroyer (denoted by DDD) – 3 units Submarine (denoted by SS) – 2 units
Place these battleships randomly on your ocean with the following conditions:
– The ships must either be in a horizontal or vertical position.
– The ships cannot overlap one another.
An example is shown below.
y Keep track of all the missiles fired
Fired to opponent Fired at me
5
Coordinates Hit Miss Coordinates Hit Miss
A A4 A A A (  ,  ) (  ,  )
(  ,  ) (  ,  )
(  ,  ) (  ,  )
3 Miss
(  ,  ) (  ,  )
C (  ,  ) (  ,  )
2 Hits (  ,  ) (  ,  )
C (  ,  ) (  ,  )
1 S S (  ,  ) (  ,  )
C (  ,  ) (  ,  )
_5 _4 _3 O x (  ,  ) (  ,  )
_2 _1 1 2 3 4 5
C (  ,  ) (  ,  )
_1 (  ,  ) (  ,  )
D (  ,  ) (  ,  )
_2 (  ,  ) (  ,  )
D (  ,  ) (  ,  )
_3 (  ,  ) (  ,  )
(  ,  ) (  ,  )
D
_4 (  ,  ) (  ,  )
(  ,  ) (  ,  )
_5 (  ,  ) (  ,  )

3. Play the battleship game with a classmate. You and your classmate will take turns to fire missiles by calling out ordered
pairs. If your opponent calls out an ordered pair that your battleships are located, the missile is a hit. For example, for
the battleships located as shown in the diagram, the ordered pair (2, 1) hits the submarine while (2, 3) is a miss. Every
missile fired at you must be made known to your opponent whether it is a hit or a miss. You can use the given table to
keep track of all the missiles fired.

4. A ship will sink if all the integral coordinates pairs of the ship are being hit. The first person who sinks all opposing
battleships is the winner.

Answers depend on students’ own strategies.

99
Class Activity 2
Objective: To use a function machine to understand the concept of function and represent a function using verbal, tabular,
graphical and algebraic forms.

A function can be considered as a machine. It takes an input, applies a rule to it, and then produces an output. We may view
the process as follows:
5, 10, 15, 20, 25
Input   Rule   Output Input x

For the function machine on the right, some input values


and their corresponding output values are as shown below.
Rule

Input: 5 10 15 20 25


Output: 8 13 18 23 28 Output y
8, 13, 18, 23, 28

Questions
1. Guess the rule of the function. In other words, how do you get 8 from 5, 13 from 10, 18 from 15, etc? Can you observe
the rule? Express the rule in your own words.

(i) When x = 5, y = 5 + 3 = 8.

(ii) When x = 10, y = 10 + 3 = 13.

(iii) When x = 15, y = 15 + 3 = 18.

From observation, the rule is to add 3 to the input to get the output.

2. We may represent a function by a table. Copy and complete the following table.

Input 5 10 15 20 25
Ouput 8 13 18 23 28

3. If we let x be the value of an input and y be the value of its y


corresponding output, then x and y can form an ordered pair (x, y).
30
(a) Form the ordered pairs for the values of input and output above.

(5, 8), (10, 13), (15, 18), (20, 23), (25, 28) 25

(b) Plot the ordered pairs on the given coordinate plane. 20

15

10

x
O 5 10 15 20 25

Chapter 12  Coordinates and Linear Functions


100
(c) What can you observe about the relationship of the ordered pairs from the diagram?

The points lie on a straight line. They are collinear.

4. (a) If the value of x is 27, what is the corresponding value of y?

When x = 27,
y = 27 + 3
= 30

(b) Suggest an equation connecting x and y.

y=x+3

(c) How many different values of y are corresponding to each value of x?

Only one value of y is corresponding to each value of x.

5. Try to form your own function machine and represent some pairs of input and output values on a coordinate plane.

101
Class Activity 3
Objective: To recognise how the graph of the linear function y = ax + b varies when either a or b varies.

Tasks

Graph of y = ax + b
6

4
A: (–1.50, 0.00)
B: (0.00, 3.00)
B
Slope AB = 2.00
2

A
15 10 5 5 10 15

y = 2·x + 3 2

1. Using Sketchpad, draw the graph of y = ax + b, where a = 2 and b = 3 on a coordinate plane.

2. Mark the points A and B where the graph cuts the x-axis and y-axis respectively.

3. Measure the gradient of the line using the command in the software.

Gradient = 2

4. (a) By keeping b fixed, observe how the graph changes when a varies from –5 to 5.

The slope of the line changes as a varies.

(b) How does the line slope when a is negative and when a is positive?

When a is positive, the line slope increases upwards when viewed from left to right.

When a is negative, the line slope decreases downwards when viewed from left to right.

(c) By keeping a fixed, observe how the graph changes when b varies from –3 to 3.

The y-intercept changes as b varies.

Chapter 12  Coordinates and Linear Functions


102
5. Write down, in terms of a and/or b,

(a) the coordinates of the point where the graph cuts the y-axis,

(0, b)

(b) the gradient of the graph,


for the graph of the linear function y = ax + b.

Gradient = a

103
Class Activity 4
Objective: To understand the physical interpretation of the gradient of a linear graph.

Questions
A car drives along a straight road which passes through a city P. Its distance, y km, from city P at time x hours is given by the
function y = 50x + 20 for 0 < x < 4.

1. Copy and complete the following table for the line y = 50x + 20. Then, draw the graph of y = 50x + 20 for
0 < x < 4 on the diagram on the right.

x 0 1 2 3 4
y = 50x + 20 20 70 120 170 220

250

200 y = 50x + 20

150

100

50

x
O 1 2 3 4

2. (a) Write down the y-intercept of the graph.

y-intercept = 20

(b) What do you think is the physical interpretation of the y-intercept?

The y-intercept is the distance in km of the car from City P at the time x = 0.

3. (a) Describe a way of finding the speed of the car in the first hour from the graph.

The speed of the car can be found by finding the gradient of the graph.

(b) Hence, find the speed of the car during the first hour.

Speed of car during first hour = 50 km/h

(c) Check if the speed of the car is the same throughout the journey. Explain your answer.

The gradient of the car for each hour is always 50 km/h. Therefore, the speed of the car is the same throughout the journey.

4. (a) Write down the gradient of the graph.

Gradient of the graph = 50

(b) How is the gradient of the graph related to the movement of the car?

The gradient of the graph represents the speed of the car.

Chapter 12  Coordinates and Linear Functions


104
Try It! Section 12.2
Section 12.1 2. A function y of x is given by y = 6 – 5x.
(a) Sketch the function machine of the function.
1. y
(b) When the input of the function machine is x = 4,
3
find the value of the output y.
(c) When the output of the function machine is y = 6,
find the value of the input x.
2

P Solution
1
(a) Input x
x
_3 _2 _1 O 1 2 3
_1
Q
6 – 5x
_2

_3

Input y

(a) State the coordinates of the points P and Q in the (b) When x = 4,
above diagram. y = 6 – 5 × 4
(b) State the quadrants in which P and Q lie. = –14
(c) Use a scale of 1 cm to 1 unit on both axes, plot
(c) When y = 6,
the points R(1, 3), S(–2, 3), T(0, –1) and U(2, –2)
6 = 6 – 5x
on a sheet of graph paper.
–5x = 0
x = 0
Solution

y
3. Ali attempted 10 multiple-choice questions in a quiz.
S(–2, 3)
3 R(1, 3)
He got x questions answered correctly and y questions
answered wrongly.
(a) Copy and complete the following table.
2

P
x 2 4 6 8 10
1
y 8
x
_3 _2 _1 O 1 2 3 (b) Plot the ordered pair (x, y) on the coordinate plane
as shown below.
_1 T(0, –1)
Q y
_2
U(2, –2) 10
_3
8

6
(a) Coordinates of P = (3, 1),
Coordinates of Q = (–2, –1)
4
(b) P is in the first quadrant.
Q is in the third quadrant.
2
(c) Points R, S, T, U are plotted on the above diagram.
x
O 2 4 6 8 10

105
(c) What can you say about the points plotted? (c) y
(d) Write down an equation that represents the
relationship between x and y. 300

Solution
250
(a)
x 2 4 6 8 10
200
y 8 6 4 2 0

(b) y 150

10 100

8 50

6 x
O 50 100 150 200 250

4
(d) The points form a straight line. They are collinear.
2

x Section 12.3
O 2 4 6 8 10 5. (a) Using a scale of 1 cm to 1 unit on both axes, draw
1
the graph of y = – x – 1 for values of x from –4
2
(c) The points form a straight line. They are collinear. to 4.

(d) The required equation is y = 10 – x. (b) Does the point B(1, –1) lie on the graph?

Solution
4. A restaurant has 10% service charge. Let $y be the amount
including service charge when the food ordered is $x. (a) x – 4 –2 0 2 4
  
(a) Express y as a function of x in the form of an y=– x–1
1
1 0 –1 –2 –3
equation. 2
(b) Copy and complete the following table. y
x 50 100 150 200 250 4
y
y = _ 12 x _ 1 2

(c) Using a scale of 1 cm to 50 units on both the x-axis x


_4 _2 O 2 4
and the y-axis, plot the ordered pairs (x, y) in (b) B(1, _ 1)
on the coordinate plane. _2
(d) What can you say about the points plotted?
_4
Solution
(a) The required equation is y = 1.1x
(b) When x = 1,
(b) Copy and complete the following table.
1
y = – x – 1
x 50 100 150 200 250 2
1
y 55 110 165 220 275 = – (1) – 1
2
3
= –
2
≠ –1
The point B(1, –1) does not lie on the graph of
1
y = – x – 1.
2

Chapter 12  Coordinates and Linear Functions


106
6. Draw the graph of the linear function Solution
y = 2 for –3 < x < 3. (a) t 0 2 4 6 8
Solution h = 40 – 5t 40 30 20 10 0
y
h
y=2
2 40

30
x h = 40 _ 5t
–3 –2 –1 O 1 2 3
20
–2
10

t
O 2 4 6 8

7. Mrs Singh bought x mangoes which cost $2 each, where


0 < x < 5. She paid the fruit seller with a $10 note and (b) When t = 5, the water level will be 15 cm.
the change she received was $y.
(a) Express y as a function of x.
(b) Using a scale of 1 cm to 1 unit on the x-axis and Section 12.4
1 cm to 2 units on the y-axis, draw the graph of 9. In the diagram below, the lines L3 and L4 are x = 4 and
the function in (a) for 0 < x < 5. y = –1 respectively. State the gradients of the lines L3
(c) Using the graph, find the change Mrs Singh received and L4.
if she bought 3 mangoes. y
L3
Solution 3
(a) The required equation is y = 10 – 2x.
2
(b) y
1
10
_1 x
O 1 2 3 4 5
8 _1
L4

y = 10 – 2x Solution
4 Gradient of L3 is undefined, gradient of L4 is 0.

2 10. The vertices of PQR are P(–2, –1), Q(3, –1) and
R(1, 2). Find the gradients of the sides of PQR.
x
O 1 2 3 4 5
Solution

y
(c) She would receive $4 change.
R(1, 2)
2
8. Water is being drained from a tank. The water level h
cm at time t minutes is given by h = 40 – 5t. 1
(a) Using a scale of 1 cm to 1 unit on the t-axis and
1 cm to 5 units on the h-axis, draw the graph of x
_2 _1 O 1 2 3
h = 40 – 5t from t = 0 to t = 8.
(b) Use the graph to find the time when the water level _ Q(3, _ 1)
P(_ 2, _ 1) 1
is 15 cm.

107
Since PQ is horizontal, (b) When t = 0,
y = –2 × 0 + 30
gradient of PQ = 0,
= 30
–3
gradient of QR = \ 30 is the initial volume of water in cm3 in the
2
filter.
3
gradient of PR =
3 (c) –2 is the rate of change of the volume of water in
= 1. the filter in cm3/min.

11. The diagram shows the line y = 3x – 6.


(a) State the coordinates of the point A at which the
line cuts the x-axis.
(b) State the coordinates of the point B at which the
line cuts the y-axis.
(c) Draw an appropriate horizontal and vertical change
triangle. Use the triangle to find the gradient of the
line.
y

A x
_2 O 2 4 6

_2
y = 3x _ 6
_4

_6
B

Solution
(a) A(2,0)
(b) B(0,-6)
(c) Gradient = 3

12. The volume, y cm3, of water in a filter after t minutes is


given by the function y = –2t + 30, where 0 , t , 15.
(a) Find the volume of water drained from t = 10 to
t = 11.
(b) Interpret the physical meaning of the constant term
30.
(c) Interpret the physical meaning of the gradient –2
in the function.

Solution
(a) y = –2t + 30
When t = 10,
y = –2 × 10 + 30
= 10
When t = 11,
y = –2 × 11 + 30
= 8
\  Volume of water drained
= 10 – 8
= 2 cm2

Chapter 12  Coordinates and Linear Functions


108
Exercise 12.1 4. Use a sheet of graph paper to draw a Cartesian plane.
Level 1 Then plot and label the following points.
1. (a) Write down the coordinates of the points A to J in
A(0, 4), B(5, 3), C(1, 3), D(0, 6)
the given diagram. E(–3, 2),
F(–2, 0), G(–5, –2), H(3, – 4)
(b) Which points are in the first quadrant?
(c) Which points are in the fourth quadrant? Solution
(d) Which points are on the y-axis? The points are plotted and labelled in the following
diagram.
y
C y
4
6 D(0, 6)
E
3
B 5
2 D

4 A(0, 4)
1
B(5, 3)
A 3 C(1, 3)
x
_5 _4 _3 _2 _1 O 1 2 3 4 5 E(_3, 2)
_1 H
2

_2
1
J F(_2, 0)
x
_3 _5 _4 _3 _2 _1 O 1 2 3 4 5
F I
_1
G _4

_2
G(_5, _2)
_3
Solution
(a) The coordinates of the points are: _4 H(3, _4)

A(3, 0), B(5, 2), C(2, 4), D(0, 2),


E(–4, 3), F(–5, –3), G(–2, – 4), H(0, –1),
I(1, –3), J(4, –2).
Level 2
(b) The points B and C are in the first quadrant.
5. (a) Plot the points A(3, 3), B(0, 3) and C(–3, 3) on a
(c) The points I and J are in the fourth quadrant. Cartesian plane.
(d) The points D and H are on the y-axis. (b) Which point(s) is/are in the second quadrant?
(c) Join OA and OC. Then measure ∠AOB and ∠BOC.
(d) What is the relationship between the line segments
2. Write down the x-coordinate of each point. OA and OC?
(a) A(3, –2) (b) B(–2, 7)
(c) C(–1, 4) (d) D(0, 0) Solution
(a) y
Solution C(_ 3, 3) A(3, 3)
3 B(0, 3)
(a) x-coordinate of A = 3
(b) x-coordinate of B = –2 2
(c) x-coordinate of C = –1
1
(d) x-coordinate of D = 0
x
_3 _2 _1 O 1 2 3
3. Write down the y-coordinate of each point. _1
(a) P(4, –5) (b) Q(3, 2)
(c) R(–6, 7) (d) S(9, 0)
(b) The point C(–3, 3) is in the second quadrant.
Solution
(a) y-coordinate of P = –5 (c) ∠AOB = 45°; ∠BOC = 45°
(b) y-coordinate of Q = 2 (d) 1. OA = OC 2.  OA ⊥ OC
3. OA and OC are symmetrical about the y-axis.
(c) y-coordinate of R = 7
(d) y-coordinate of S = 0

109
6. (a) Plot the points P(1, –2) and Q(– 4, –2) on a Cartesian 8. (a) Plot the points A(–3, 1), B(0, –1), C(1, –1) and
plane. D(4, –1) on a Cartesian plane.
(b) State the quadrant in which P lies. (b) Which of the above points are in the fourth
(c) State the quadrant in which Q lies. quadrant?
(d) Join OP and OQ. Then measure ∠POQ. (c) If a point is in the fourth quadrant, what is the sign
(e) Find the coordinates of a point R such that the of its
x-axis is the perpendicular bisector of the line (i) x-coordinate?
segment QR. (ii) y-coordinate?

Solution Solution
(a) y (a) y
R(_ 4, 2)
2 A(_ 3, 1)
1
1 x
_3 _2 _1 O 1 2 3 4
_4 _3 _2 _1 O x _1
1 2 D(4, _ 1)
B(0, _ 1) C(1, _ 1)
_1

_2
Q(_ 4, _ 2) P(1, _ 2) (b) C(1, –1) and D(4, –1) are in the fourth quadrant.

(c) (i) The sign of its x-coordinate is positive.
(b) P(1, –2) lies in the fourth quadrant. (ii) The sign of its y-coordinate is negative.

(c) Q(– 4, –2) lies in the third quadrant.
(d) ∠POQ = 90° Level 3
(e) Coordinates of R = (– 4, 2)   9. (a) On a sheet of graph paper, draw a Cartesian plane
using a scale of 1 cm to represent 2 units on both
axes and then plot the following points.
7. (a) Plot the points S(–5, 0) and T(0, 2) on a Cartesian
plane. A(6, 0), B(3, 1), C(2, 6), D(0, 2),
(b) Join ST. Find the area of SOT. E(– 4, 2), F(–2, 0), G(– 6, – 6), H(–1, –3),
(c) State the coordinates of the midpoint M of S and I(4, – 6), J(2, –2)
T. (b) Join the points successively from A to J with straight
(d) State the relationship between the lengths of MO, line segments. Join the points J and A. What design
MS and MT. do you get?

Solution Solution
(a) y (a) and (b)
2 T(0, 2) y
M C
1 6
S(_ 5, 0)
_5 _4 _3 _2 _1 O x 4

_1 E 2 D
B
F A
_6 _4 _2 O x
2 4 6
_2
(b) Area of SOT =
1
× OS × OT J
2
H
_4
1
= ×5×2
2 _6
= 5 units2 G I

1
(c) Coordinates of M = –2 , 1
5

(d) MO = MS = MT

Chapter 12  Coordinates and Linear Functions


110
10. The diagram shows a map of Singapore on a Cartesian Solution
plane. (a) y – 7 = x
(b) y = 3x
1
(c) y = x
2

(d) y = 2x + 3

2. Find the value of the output y when the input is


x = 3 for each of the following function machines.
(a) (b)
Input x Input x

(a) Write down the coordinates of the following places.
(i) Changi International Airport
(ii) Yishun x+7 5 – 2x

(iii) Punggol
(iv) Queenstown
(b) Name the places located at the following Output y Output y
coordinates.
(i) (0, 1) Solution
(ii) (–6, 0) (a) When x = 3,
(iii) (–2, 4) y = x + 7
(iv) (4, –1) = 3 + 7
(c) Find the units of distance between = 10
(i) Jurong and Bukit Timah on the map,
(b) When x = 3,
(ii) Sembawang and Queenstown on the map.
y = 5 – 2x
= 5 – 2 × 3
Solution
= –1
(a) (i) Changi International Airport is at (6, 0).
(ii) Yishun is at (0, 3).
3. A function y of x is given by y = x + 1.
(ii) Punggol is at (3, 2). (a) Copy and complete the following table.
(ii) Queenstown is at (–2, –2). x 0 1 2 3 4
(b) (i) (0, 1) is Ang Mo Kio. y
(ii) (–6, 0) is Jurong.
(b) Plot the ordered pairs (x, y) of the above table on
(iii) (–2, 4) is Sembawang. a copy of the coordinate plane shown below.
(iv) (4, –1) is Bedok. y
(c) (i) The required distance
= –2 – (–6) 5
= 4 units
(ii) The required distance 4
= 4 – (–2)
= 6 units 3

2
Exercise 12.2
Level 1
1
1. Express y as a function of x in the form of an equation
in each of the following.
x
(a) y is 7 less than x. O 1 2 3 4
(b) y is three times of x.
(c) y is one-half of x.
(d) y is 3 more than twice of x.

111
Solution (b) y
(a) y = x + 1
6
x 0 1 2 3 4
y 1 2 3 4 5
4

(b) y
2
5
x
_4 _2 O 2 4
4

3

Level 2
2
5. (–3, 3), (–2, 2), (1, –1) and (4, – 4) are four ordered pairs
of a function y of x.
1 (a) Determine which of the following can be the
function.
x (i) y = –x
O 1 2 3 4 (ii) y = x + 6
(b) Find the value of y when x = 3.

Solution
1
4. A function y of x is given by y = 4 – x. (a) All the given points satisfy the equation y = –x.
2
When x = 1,
(a) Copy and complete the following table. y = x + 6
x –4 –2 0 2 4 = 1 + 6
= 7
y \ (1, –1) does not satisfy the equation y = x – 6.
Hence, the required function is y = –x.
(b) Plot the ordered pairs (x, y) of the above table on
a copy of the coordinate plane shown below. (b) When x = 3,
y = –x
y = –3

6
6. A function y of x has values of x and y as shown in the
table below.
4
x 1 2 3 4 5
2 y 3 4 5 6 7

x (a) Determine which of the following can be the


_4 _2 O 2 4 function.
(i) y = 2x
(ii) y = x + 2
Solution
1 (b) Find the value of y when x = 0.
(a) y = 4 – x
2
Solution
x –4 –2 0 2 4 (a) All the given points satisfy the equation y = x + 2.
y 6 5 4 3 2 When x = 1,
y = 2x
= 2(1)
= 2
\  (1, 3) does not satisfy the equation y = 2x.
Hence, the required function is y = x + 2.

Chapter 12  Coordinates and Linear Functions


112
(b) When x = 0, (b) y
y = x + 2
= 0 + 2 10
= 2
8
7. A table of values of the function y = 1 – 2x is given
below.
6
x –4 –2 0 q
y 9 p 1 –1 4

(a) Find the values of p and q.


2
(b) Plot the ordered pairs (x, y) of the table on a copy
of the coordinate plane shown below.
y x
_4 _2 O 2 4

10 _2

8
(c) The plotted points lie on a straight line.
6

8. The diagram shows the graph of a function.


4
y
2 2

x x
_4 _2 O 2 4 _4 _2
O 2 4
_2 _2


(c) What can you say about the points plotted? (a) If (–2, a) and (b, 2) are points on the graph, find
the values of a and b.
Solution (b) Write down the equation of the function.
(a) y = 1 – 2x
(–2, p) is a point on the function. Solution
p = 1 – 2(–2) (a) From the graph,
\  p = 5 When x = –2, y = –1
(q, –1) is a point on the function. \  a = –1
–1 = 1 – 2q When y = 2, x = 4
2q = 2 \  b = 4
1
\  q = 1 (b) The equation of the function is y = x.
2

Level 3
9. Let y be the total number of wheels of x bicycles.
(a) Copy and complete the following table.

x 1 2 3 4
y
(b) Using a scale of 1 cm to 1 unit on both the x-axis
and the y-axis, plot the ordered pairs (x, y) of the
above table on the coordinate plane.
(c) Write down the equation connecting x and y.

113
Solution (b) y
(a) x 1 2 3 4
11
y 2 4 6 8

(b) y 10

8 9

7 8

6 7

5 6

4 5

3 4

2 3

1 2

x 1
O 1 2 3 4

x
O 1 2 3 4
(c) The required equation is y = 2x.

10. A lift is at the 3rd floor. It rises 2 floor levels per second. (c) The required equation is y = 2x + 3.
Let y be the floor level of the lift after x seconds. (d) When y = 15,
(a) Copy and complete the following table. 15 = 2x + 3
x 0 1 2 3 4 2x = 12
x = 6
y 3 \  The time taken is 6 seconds.
(b) Using a scale of 1 cm to 1 unit on both the x-axis
and the y-axis, plot the ordered pairs (x, y) of the 11. Patrick has $8 in his pocket. Let $y be the amount left
above table on the coordinate plane. after he spends $x on his lunch.
(c) Write down the equation connecting x and y. (a) Copy and complete the following table.
(d) Hence, find the time taken for the lift to reach the
x 2 3 4 5 6
15th floor.
y
Solution
(a) (b) Using a scale of 1 cm to 1 unit on both the x-axis
x 0 1 2 3 4
and the y-axis, plot the ordered pairs (x, y) of the
y 3 5 7 9 11 above table on the coordinate plane.
(c) What can you say about the points plotted in (b)?
(d) Write down the equation connecting x and y.
(e) Hence, find the value of x when y = 7.

Chapter 12  Coordinates and Linear Functions


114
Solution (c) From the equation, when x = 10,
(a) y = –5 × 10 + 20
x 2 3 4 5 6
= –30
y 6 5 4 3 2
This means the meat will read a temperature of
(b) –30C°.
y However, a freezer has a minimum temperature
which is usually above –30°C. The temperature
6 of the meat will not decrease further after reaching
the minimum temperature.
5 \ We cannot use the equation to work out the
temperature of the meat after 10 hours.
4

Exercise 12.3
3
Level 1
For Question 1 to 7, use a scale of 1 cm to 1 unit on both the
2
x-axis and the y-axis to draw the graphs.

1
1. (a) Copy and complete the following table.
x –4 –2 0 2 4
x y=x–2
O 1 2 3 4 5 6
(b) Draw the graph of y = x – 2 for values of x from – 4
to 4.
(c) The plotted points lie on a straight line.
Solution
(d) The required equation is y = 8 – x.
  (a) x –4 –2 0 2 4
(c) When y = 7,
y=x–2 –6 –4 –2 0 2
7 =8–x
x = 8 – 7 (b) y
x = 1
2 y=x_2

12. A piece of meat at 20 °C is placed into a freezer. _4 _2 O x


2 4
Its temperature y °C after x hours are measured and
_2
summarised in the following table.
_4
x 1 2 3 4 5
y 15 10 5 0 –5 _6

(a) Find an equation connecting x and y.


1
(b) Hence, find the temperature of the meat after 3
2
hours. 2. (a) Copy and complete the following table.
(c) Can we use the equation in (a) to work out the x –4 –2 0 2 4
temperature of the meat after 10 hours? Explain
briefly. y = –2x + 1 9

(b) Draw the graph of y = –2x + 1 from x = –4 to


Solution
x = 4.
(a) The required equation is y = –5x + 20.
1
(b) When x = 3 ,
2
7
y = –5 + 20
2
1
= 2
2
Hence, the required temperature is 2°C.

115
Solution 4. (a) Draw the graph of y = 3(x – 1) for values of x from
  (a) –3 to 3.
x –4 –2 0 2 4
(b) Find the point at which the graph cuts the x-axis.
y = –2x + 1 9 5 1 –3 –7
Solution

(b) (a) x –3 0 3
y
y = 3(x – 1) –12 –3 6
8
y
6
6
4
4
2
2 y = 3(x _ 1)
x
_4 _2 O 2 4
_2 O x
_2 y = _ 2x + 1 _4 2 4
_2
_4
_4

_6

1 _8
3. (a) Draw the graph of y = x + 3 from x = – 4 to
2
x = 4. _ 10
(b) Does the point A(1, 3) lie on the graph?
_ 12
Solution
  (a) x – 4 0 4
1
y= x +3 1 3 5 (b) The graph cuts the x-axis at (1, 0).
2

y 5. Draw the following lines on the same diagram.


(a) y = 2 (b) y = –3
6
y = 12 x + 3
Solution
4
A(1, 3) y
2
(a) y = 2
2
_4 _2 O x
2 4
1

(b) When x = 1,
_1 x
1 O 1 2
y = (1) + 3
2
1 _1
= 3
2
= 3.5 _2
The point A(1, 3) does not lie on the graph.
_3 (b) y = _3

Chapter 12  Coordinates and Linear Functions


116
Level 2 y
6. (a) Draw the following lines on the same diagram for y= _ 6
values of x from –3 to 3. x+
3
(i) y = x + 1
1
(ii) y = x + 1 y= _ 4
3 x
y= _ +1
1
(iii) y = – x
2
+ 1 (iv) y = 1 y= _ x 2
x _
(b) What is the common property of the lines in (a)? 2
x
(c) What is the common property of the equations in _3 _2 _1 O 1 2 3
(a)? _2
Solution
_4
(a) x –3 0 3
_6
y=x+1 –2 1 4
1 0 1 2
y= x+1
2
1 (b) All the lines in (a) are parallel to each other.
y=– x+1 2.5 1 –0.5
2
(c) The coefficient of x in each equation is –1.
y=1 1 1 1

8. (a) Draw the graph of y = 12 – 3x from x = –2 to


y x = 5 using a scale of 1 cm to 1 unit on the x-axis
and 1 cm to 5 units on the y-axis.
4 y =x + 1
(b) The points (a, 0), (1, b) and (c, 5) are some points
y = _ 12 x + 1 3 on the graph. Use the graph to find the values of
y = 13 x + 1 a, b and c graphically.
2
y=1 Solution
1
(a) x –2 0 2 5
x
_3 _2 _1 O 1 2 3 y = 12 – 3x 18 12 6 –3
_1

_2 y

20

y = 12 _ 3x
(b) All the lines in (a) pass through the point (0, 1).
10 (1, b)
(c) The constant term of each equation is 1.
(c, 5)
7. (a) Draw the following lines on the same diagram for (a, 0)
_2 O x
values of x from –3 to 3. 2 4 6
(i) y = –x
(ii) y = –x + 1
(iii) y = –x – 2 (b) When y = 0, the graph cuts the x-axis at (4, 0).
(iv) y = –x + 3 ∴  a = 4.
(b) What is the common property of the lines in (a)? The line x = 1 meets the graph at (1, 9).
(c) What is the common property of the equations of ∴  b = 9.
the lines in (a)?
The line y = 5 meets the graph at (2.3, 5).
Solution ∴  c = 2.3.
  (a) x –3 0 3
y = –x 3 0 –3
y = –x + 1 4 1 –2
y = –x – 2 1 –2 –5
y = –x + 3 6 3 0

117
Level 3 (c)
9. The volume V cm3 of a gas, at 1 atmosphere pressure, y
formed in a chemical reaction at time t minutes is given
by V = 5t. 100
(a) Using a scale of 1 cm to 1 unit on the t-axis and
1 cm to 5 units on the V-axis, draw the graph of 80
V = 5t from t = 0 to t = 6. y = 20 + 10t
(b) Use your graph to find the time taken to produce 60
20 cm3 of the gas.
40
Solution
(a) 20
x 0 3 6
y 0 15 30 O
t
1 2 3 4 5 6 7 8

V
30 V = 5t (d) Draw the horizontal line y = 70 on the diagram in
(c) to meet the line. From the graph, the time is 5
20 minutes.
10 (e) When water reaches the temperature 100°C, it will
boil and the temperature will stay at 100°C until
t all water becomes vapour.
O 1 2 3 4 5 6
\ We cannot use the equation to find the

temperature of water after 10 minutes.
(b) Draw the horizontal line V = 20 on the diagram in
(a) to meet the line. From the graph, the time is
11. Water is drained from a burette. The height, H cm,
100 minutes.
of water at time t minutes is given by the function
H = 36 – 4t for 0  t  9.
10. The initial temperature of water in a kettle is 20 °C. On (a) Using a scale of 1 cm to 1 unit on the t-axis and
heating, the temperature of water increases by 10 °C 1 cm to 10 units on the H-axis, draw the graph of
per minute. Let y °C be the temperature of water after t H against t.
minutes. (b) Using the graph, find the time that the height of
(a) Copy and complete the following table of values water is 14 cm.
of t and y.
Solution
x 0 2 4 6 8 (a) t 0 5 9
y
H 36 16 0
(b) Write down the equation of the function
connecting t and y. H
(c) Using a scale of 1 cm to 1 unit on the t-axis and
40
1 cm to 20 units on the y-axis, draw the graph of
y against t. H = 36 _ 4t
30
(d) Using the graph, find the time that the temperature
of water is 70 °C. 20
(e) Can we use the equation in (b) to find the
temperature of water after 10 minutes? Explain 10
briefly.
O t
1 2 3 4 5 6 7 8 9
Solution
(a) x 0 2 4 6 8
y 20 40 60 80 100 (b) Draw the horizontal line H = 14 on the diagram
in (a) to meet the line. From the graph, the time is
(b) y = 20 + 10t 5.5 minutes.

Chapter 12  Coordinates and Linear Functions


118
12. Su Meng is driving to a city. His distance d km from S

the city at time t hours after starting is given by 1500

d = 180 – 60t for 0  t  3.


1400
(a) Using a scale of 2 cm to 1 unit on the t-axis and
1 cm to 50 units on the d-axis, draw the graph of 1300

d = 180 – 60t for 0  t  3.


1200
(b) Use the graph to find the time when Su Meng is
30 km away from the city. 1100

1000
Solution S = 300 + 100t
900
(a) t 0 1 2 3
800
d = 180 – 60t 180 120 60 0
700

d 600

200 500

d = 180 _ 60t 400

300
100
200

30 100
t
O 1 2 3 O t
1 2 3 4 5 6 7 8 9 10 11 12

(b) From the graph, when d = 30, t = 2.5.
Soo Meng is 30 km from the city after 2.5 hours.
Exercise 12.4
13. Describe a real-life example where 2 variables are in Level 1
a linear relationship and draw a graph to represent the 1. Find the gradients of the lines L1, L2 and L3 in the
relationship. following diagram.
y
L2
Solution
3 L1
The following graph shows the equation over 12 months
L3
where S represents savings in dollars in a savings account
2
and t represents time in months. From the graph, we can
see that the savings account opened with a balance of
1
$300 and has a monthly saving of $100.
x
_3 _2 _1 O 1 2 3
_1

Solution
3
Gradient of L1 =
3
=1
3
Gradient of L2 =
1
=3
–2
Gradient of L3 =
3
2
= –
3

119
2. Find the gradients of the lines L4, L5 and L6 in the Solution
following diagram. (a)
y x –2 –1 0 1 2
L4 y 7 5 3 1 –1
3 L5

(b) Gradient of line


2
vertical change
=
horizontal change
1
–2
=
1
x
_2 _1 O 1 2 3 = –2
_1
L6
Level 2
For questions 5 to 8, use a scale of 1 cm to 1 unit on both axes.
Solution
5. Draw the line through the given pair of points and find
(–2, 1) and (3, 3) are on the line L4.
its gradient.
2
∴  gradient of L4 = (a) O(0, 0), A(2, 2)
3
(1, 2) and (2, 0) are on the line L5. (b) B(1, 2), C(3, 3)
–2
(c) D(–2, 3), E(2, 1)
∴  gradient of L5 = = –2 (d) F(–5, 2), G(1, –3)
1
(–2, 1) and (2.5, –1) are on the line L6. (e) H(–2, 1), I(3, 1)
–2 4
(f ) J(–3, 2), K(–3, –2)
∴  gradient of L6 = =–
4.5 9
Solution
y
3. Find the gradients of the sides of ABC in the following D(_ 2, 3) C(3, 3)
diagram. 3
y F(_ 5, 2) B(1, 2)
J(_ 3, 2) 2 A(2, 2)
3
B H(_ 2, 1) E(2, 1) I(3, 1)
1

2 x
_5 _4 _3 _2 _1 O 1 2 3
A 1 _1
C

x K(_ 3, _ 2) _2
_2 _1 O 1 2 3
_3 G(1, _ 3)

Solution
The vertices of ABC are A(–2, 1), B(2.5, 3) and 2
C(2.5, 1). (a) Gradient of OA = =1
2
2 4
Gradient of AB = = (b) Gradient of BC =
1
4.5 9
2
Since BC is vertical, gradient of BC is undefined.
–2 1
Since AC is horizontal, gradient of AC = 0. (c) Gradient of DE = =–
4 2
–5 5
4. (a) Copy and complete the table for the line (d) Gradient of FG = =–
6 6
y = –2x + 3.
(e) Gradient of HI = 0
x –2 –1 0 1 2 (f ) Since the line JK is vertical, gradient of JK is
y undefined.

(b) Using a scale of 1 cm to 1 unit on both axes, draw


the graph of y = –2x + 3 for –2  x  2.
(c) Hence, find the gradient of the line.

Chapter 12  Coordinates and Linear Functions


120
6. (a) Draw the ABC whose vertices are A(–2, 0), 8. (a) Draw the parallelogram PQRS whose vertices are
B(3, –2) and C(0, 2). P(– 4, –2), Q(5, 1), R(6, 4) and S(–3, 1).
(b) Hence, find the gradients of the sides of ABC. (a) Hence, find the gradients of the sides of PQRS.
(c) What can you say about the gradients of the opposite
Solution sides of a parallelogram?
y
Solution
2 C(0, 2) (a) y
R(6, 4)
4
1
A(_ 2, 0) 2
x S(_ 3, 1)
_2 _1 O 1 2 3 Q(5, 1)
_1 x
_4 _2 O 2 4 6
_2
_2
B(3, _ 2) P(_ 4, _ 2)

–2 2 3 1
Gradient of AB = =– (b) Gradient of PQ = =
5 5 9 3
–4 4 3
Gradient of BC = =– Gradient of QR = =3
3 3 1
2 3 1
Gradient of CA = =1 Gradient of RS = =
2 9 3
3
Gradient of SP = =3
1
7. (a) Draw the quadrilateral ABCD whose vertices are
A(–1, –1), B(7, 3), C(4, 6) and D(0, 4). (c) The gradients of the opposite sides of a parallelogram
(b) Hence, find the gradients of the sides of ABCD. are equal.
(c) What type of quadrilateral is ABCD?
9. Find the gradients and y-intercepts of the lines L1, L2 and
Solution
L3 in the diagram below.
(a) y
C(4, 6) y
6
3
4 D(0, 4)
2
2 B(7, 3)
L3
1
_2 O x
2 4 6
A(_ 1, __1) _2 _1 O x
2 1 2 3
_1
L2
4 1 _2
(b) Gradient of AB = =
8 2
–3 _3
Gradient of BC = = –1 L1
3
2 1
Gradient of CD = =
4 2
5 Solution
Gradient of DA = =5
1 Gradient of L1
vertical change
(c) ABCD is a trapezium. =
horizontal change
–2
=
1
= –2

121
Gradient of L2 11. The distance y metres, of a boy at time t seconds from
vertical change a fixed point F is given by
=
horizontal change
y = 3 + 0.5t for  0  t  10.
2
=
2 (a) Draw the graph of y = 3 + 0.5t for 0  t  10.
= 1 (b) Find the gradient of the graph.
(c) What is the physical meaning of the gradient in
Gradient of L3 this case?
vertical change
=
horizontal change
Solution
2
=   (a) t 0 2 4 6 8 10
3
y = 3 + 0.5t 3 4 5 6 7 8

Level 3 y
10. Paul is cycling along a horizontal road. His distance,
8
y km, from the starting point after t hours is given by y = 3 + 0.5t
the function y = 16t, for 0 , t , 3. 6
(a) Using a scale of 2 cm to 1 unit on the t-axis and
1 cm to 10 units on the y-axis, draw the graph of 4
y = 16t.
(b) Find the distance travelled by Paul in 2
(i) the first hour,
t
(ii) the second hour. O 2 4 6 8 10
(c) Interpret the physical meaning of the gradient of
the graph.
5
(b) Gradient of the graph = = 0.5
10
Solution
  (a) t 0 1 2 3 (c) The gradient is the speed of the boy in m/s. That
is, the boy’s speed away from the fixed point F is
y = 16t 0 16 32 48
0.5 m/s.
y
12. The volume V cm3 of a piece of melting ice at time
50 t minutes is given by
V = 24 – 3t for 0  t  8.
40
(a) Using a scale of 1 cm to 1 unit on the t-axis and
y = 16t 2 cm to 5 units on the V-axis, draw the graph of
30 V = 24 – 3t for 0  t  8.
(b) Find the gradient of the graph.
20 (c) What is the physical meaning of the gradient in
this case?
10
(d) Interpret the physical meaning of the constant term
24.

t
O 1 2 3

(b) (i) Draw the vertical line t = 1 on the diagram


in (a) to meet the line. From the graph, the
distance is 16 km.
(ii) Draw the vertical line t = 2 on the diagram
in (a) to meet the line. From the graph, the
distance is 32 km.
(c) The gradient of the graph represents Paul’s cycling
speed.

Chapter 12  Coordinates and Linear Functions


122
Solution 14. Mr Lee bought a TV set by monthly instalments. The
total amount still needed to be paid, $y, after x months
  (a) t 0 2 4 6 8
is given by the function y = 3600 – 400x.
V = 24 – 3t 24 18 12 6 0 (a) Interpret the physical meaning of the constant term
3600 in the function.
V (b) Interpret the physical meaning of the gradient – 400

25 in the function.
(c) Find the number of months of instalments.
20
V = 24 _ 3t Solution
15 (a) y = 3600 – 400x
When x = 0,
10
y = 3600 – 400 × 0
5 = 3600.
The constant term 3600 is the initial loan in dollars.
t
O 2 4 6 8 (b) The gradient –400 means the monthly instalment
is $400.
–24 (c) When y = 0,
(b) Gradient of the graph = = –3
8 0 = 3600 – 400x
(c) The gradient is the rate of change of the volume 400x = 3600
of the piece of ice in cm3/min. That is, the piece x = 9
of ice melts at 3 cm3/min. Number of months of instalments is 9.
Note: The negative sign of the gradient indicates
that the volume is decreasing.
15. (a) Name three public buildings that have access ramps
at the entrance for people in wheelchairs.
13. The cost, $y, of screen printing x T-shirts is given by the (b) Search online for the maximum gradient for a local
function y = 8x + 150. access ramp for wheelchairs.
(a) Find the cost of printing the 100th T-shirt. (c) Based on your result in (b), find the minimum
(b) Interpret the physical meaning of the constant term horizontal length of a ramp required for a vertical
150. rise of 0.5 m.
(c) Interpret the physical meaning of the gradient 8 in
the function. Solution
(a) Three public buildings that have access ramps are:
Solution Singapore Conference Hall, Victoria Concert Hall,
(a) y = 8x + 150 Singapore Indoor Stadium.
When x = 99, 1
(b) The maximum gradient of an access ramp = in
y = 8 × 99 + 150 10
Singapore.
= 942
In UK, the recommended maximum gradient is
When x = 100, 1 1
and the preferred maximum gradient is .
y = 8 × 100 + 150 15 20
= 950
(c) When rise = 0.5 m,
Cost of printing the 100th T-shirt 0.5 1
= 950 – 942 =
10 run
= $8 run =5m
(b) When x = 0, Note: For more information, you can ask students
y = 8 × 0 + 150 to visit the website of Disabled People’s
= 150 Association, Singapore at http://www.dpa.
The constant term 150 is the fixed cost of screen org.sg/access/ramp.htm.
printing in dollars.
(c) The gradient, 8, is the variable cost of printing a
T-shirt in dollars.

123
Revision Exercise 12 (a) What is the gradient of the line?
In this exercise, use a scale of 1 cm to 1 unit for both the axes (b) Write down the coordinates of the point at which
unless specified otherwise. the line cuts the x-axis.
(c) Write down the coordinates of the point at which
1. Four points A, B, C and D are given as shown in the the line cuts the y-axis.
diagram below.
Solution
(a) State the coordinates of A, B, C and D.
–7
(b) Which point is in the second quadrant? (a) Gradient of the line AB =
7
(c) Find the gradient of
= –1
(i) CD, (ii) BC,
y (b) The line cuts the x-axis at (2, 0).

6
(c) The line cuts the y-axis at (0, 2).
C

4 3. The vertices of ABC are A(1, 6), B(–4, 3) and


C(5, –2).
2
(a) Copy the following x-y plane on a sheet of graph
B A paper and draw ABC.
x
_4 _2 O 2 4
y
_2
6
D

2
Solution
(a) The coordinates are x
_4 _2 O 2 4 6
A(4, 0), B(–1, 0), C(–1, 5), D(– 4, –3).
_2
(b) Point C is in the second quadrant.
(c) (i) Gradient of AB = 0
(b) Hence, find the gradients of the sides of ABC.
(ii) Since BC is vertical, gradient of BC is
undefined.
Solution
8
(iii) Gradient of CD = (a) y
3
A(1, 6)
(d) (i) The equation of AB is y = 0. 6

(ii) The equation of BC is x = –1.


B(_4, 3) 4

2
2. The graph shows the line joining A(–3, 5) and
B(4, –2). x
_4 _2 O 2 4 6
y
_2
C(5, _2)
6
A(_3, 5)
4 3
(b) Gradient of AB =
5
2 –5
Gradient of BC =
9
x
_4 _2 O 2 4 6 5
=–
9
_2
B(4, _2) –8
Gradient of CA =
4
= –2

Chapter 12  Coordinates and Linear Functions


124
4. The vertices of a quadrilateral ABCD are (b) When x = 2,
A(–3, –2), B(–5, 1), C(2, 3) and D(4, 0). 1
y = x + 2
(a) Copy the following x-y plane on a sheet of graph 3
paper and draw quadrilateral ABCD. 1
= (2) +2
3
y 2
= 2
3
4
≠ 3
2 The point (2, 3) does not lie on the line L.
4
x
(c) Gradient of the line L =
_6 _4 _2 O 12
2 4
1
_2
=
3
(d) The line L cuts the y-axis at (0, 2).
_4

6. The monthly salary of Nelson is $300 more than twice the


monthly salary of Joanne. Let $x and $y be the monthly
(b) Hence, find the gradients of the sides of ABCD.
salaries of Joanne and Nelson respectively.
(a) Find the monthly salary of Nelson if Joanne’s
Solution
monthly salary is
(a) y
(i) $2000, (ii) $3000.
4 (b) Express y in terms of x.
C(2, 3)
(c) Using the scale of 1 cm to $1000 on both the axes,
B(_5, 1) 2 draw the graph of y against x for 0  x  4000.
D(4, 0) (d) Use the graph to find the monthly salary of Joanne
_6 _4 _2 O x
2 4 if Nelson’s monthly salary is $5000.
_2
A(_3, _2) Solution
(a) (i) Monthly salary of Nelson
= 2 × 2000 + 300
–3 3 = $4300
(b) Gradient of AB = =–
2 2
(ii) Monthly salary of Nelson
2
Gradient of BC = = 2 × 3000 + 300
7
–3
= $6300
3
Gradient of CD = =– (b) y = 2x + 300
2 2
2 (c) x 0 1000 2000 4000
Gradient of DA =
7
y 300 2300 4300 8300
1
5. (a) The equation of a line L is y = x + 2. Draw the y
3
graph of L from x = – 6 to x = 6. 8000
(b) Does the point (2, 3) lie on the line L?
(c) Find the gradient of the line L. 7000
(d) At which point does the line L cut the y-axis? y = 2x + 300
6000
Solution
(a) x – 6 0 6 5000

1
y= x+2 0 2 4 4000
3

y 3000

L
4 (2, 3) 2000
y= 3x
1
+2
2 1000

_6 _4 _2 O x x
2 4 6 O 1000 2000 3000 4000

125
(d) Draw the horizontal line y = 5000 on the diagram (c) When will the temperature of the meat be 0 °C?
in (c) to meet the line. (d) What is the physical meaning of 20 in the equation
The monthly salary of Joanne is $2400 (correct to y = 20 – 4t ?
the nearest hundred).
Solution

7. In a chemical reaction, the volume V cm of a crystal at 3 (a)


t 0 2 4 6
3
time t minutes is given by the function V = t + 2 for y = 20 – 4t 20 12 4 – 4
4
0  t  8.
(a) Draw the graph of the function. y
(b) Use the graph to find the time that the volume of
20
the crystal is 5 cm3.
(c) Interpret the physical meaning of the constant term y = 20 _ 4t
15
2.
(d) Interpret the physical meaning of the gradient in 10
the function.
5
Solution
t
(a) O 1 2 3 4 5 6
t 0 2 4 6 8
_5
V 2 3.5 5 6.5 8

V
–16
8 (b) Gradient of the graph =
4
= – 4
(c) When y = 0, t = 5.
6
The required time is 5 hours later.
(d) When t = 0, y = 20.
v = 43 t + 2
Hence 20 is the initial temperature of the meat in
4 °C.

t
O 2 4 6 8


(b) Draw the horizontal line V = 5 on the diagram in
(a) to meet the line.
The required time is 4 minutes.
(c) When t = 0, V = 2.
The constant term is the initial volume of the crystal.
3
(d) The gradient is the rate of increase of the volume
4
of the crystal in cm3/min.

8. A piece of meat is kept in a refrigerator. Its temperature


y °C at time t hours is given by y = 20 – 4t for
0  t  6.
(a) Using the scale of 2 cm to 1 unit on the t-axis and
2 cm to 5 units on the y-axis, draw the graph of
y = 20 – 4t for 0  t  6.
(b) Find the gradient of the graph.

Chapter 12  Coordinates and Linear Functions


126
Review Exercise 3 (b) Amount of donation to B
5
1. In 2008, December was the most popular month for = $6000 ×
3+5
marriages in Singapore. A total of 24 596 marriages were
registered in 2008, of which 2867 took place in December. = $3750
Find the percentage of marriages in December in 2008,
giving your answer correct to 3 significant figures. 5. The ratio of the volumes of coffee in three cups, A, B
and C, is 5 : 4 : 7. If the volume of coffee in cup C is
Solution 60 mℓ more than that in cup A, find the volume of coffee
The required percentage in cup B.
2867
= × 100%
24596 Solution
= 11.7%  (correct to 3 s.f.) Let 5x ml, 4x ml and 7x ml be the volumes in cups A,
B and C respectively.
2. The total population of Singapore was 4 027 900 in 2000 7x – 5x = 60
and was 5 076 700 in 2010. Find the percentage increase 2x = 60
in the total population from 2000 to 2010, giving your x = 30
answer correct to 3 significant figures. Volume of coffee in cup B
= 4x
Solution = 4 × 30
The percentage increase = 120 ml
5076700 – 4027900
= × 100%
4027900
= 26.0%  (correct to 3 s.f.) 6. In a regular polygon, the ratio of an interior angle to an
exterior angle is 13 : 2. Find
(a) the size of an exterior angle,
3. Rita has dinner with her family in a restaurant. The price (b) the number of sides of the polygon.
of the food ordered is $150. There is a 10% service charge
on the amount and a 7% GST on both the amount and Solution
the service charge. Find (a) Let 13x° and 2x° be the sizes of an interior angle
(a) the service charge, and an exterior angle of the regular polygon.
(b) the total amount of the bill. 13x° + 2x° = 180° (adj. ∠s on a st. line)
15x = 180
Solution x = 12
(a) Service charge = $150 × 10%
= $15 Size of an exterior angle
= 2 × 12°
(b) GST amount = 24°
= ($150 + $15) × 7%
= $11.55 (b) Number of sides of the polygon
360°
Total amount =  (ext. ∠ sum of polygon)
24°
= $150 + $15 + $11.55 = 15
= $176.55
7. In a sequence, the first term is 17. Each subsequent term
4. Mr Singh donates $6000 to two organisations, A and B. is 4 more than the previous term.
The ratio of the donation to A to the donation to B is (a) Write down the 3rd term and the 4th term of the
3 : 5. Find sequence.
(a) the donation to A as a percentage of the total (b) Express the nth term of the sequence in terms of
donation, n.
(b) the amount of the donation to B.
Solution
Solution (a) the 2nd term = 17 + 4
(a) Percentage of donation to A = 21
3
= × 100% the 3rd term = 21 + 4
3+5
= 37.5% = 25
the 4th term = 25 + 4
= 29

127
(b) the nth term = 17 + (n – 1) × 4 (b) Gradient of the line
= 17 + 4n – 4 –5 – 2
=
= 4n + 13 3+4
= –1
8. (a) The nth term of a sequence is given by n(n + 3). (c) The line cuts the x-axis at (–2, 0).
Find the values of the first 4 terms.
(b) Hence, express the nth term of the sequence 10. The price, $y, for x bowls of fish ball noodle is given by
3, 9, 17, 27, … in terms of n. y = 3x.
(a) Find the price of 4 bowls of fish ball noodle.
Solution
(b) Draw the graph of y = 3x for 0 < x < 4, using the
(a) the nth term = n(n + 3)
scale of 1 cm to 1 unit on both axes.
\ the 1st term = 1 × (1 + 3) (c) Find the gradient of the graph in (b).
=4
Solution
the 2nd term = 2 × (2 + 3)
(a) When x = 4,
= 10
y = 3 × 4
the 3rd term = 3 × (3 + 3) = 12
= 18 The required price is $12.
the 4th term = 4 × (4 + 3) (b) x 0 2 4
= 28
y = 3x 0 6 12
(b) The sequence
3, 9, 17, 27, … y

can be written as 12
4 – 1, 10 – 1, 18 – 1, 28 – 1 , …
\ the nth term = n(n + 3) – 1 10

8
9. (a) Using the scale of 1 cm to 1 unit on both axes, plot y = 3x
the points A(–4, 2) and B(3, –5) on an x-y plane 6
and draw the line connecting the points.
(b) Find the gradient of the line. 4
(c) Write down the coordinates of the point where the
line cuts the x-axis. 2

Solution x
O 1 2 3 4
(a) The graph shows the line connecting
A(–4, 2) and B(3, –5).
12
y (c) Gradient
=
4
A
2
=3

1 11. There were 3 candidates, Bill, Nancy and Steve, running


for the election of chairman in a Sports Club. Bill
x
–4 –3 –2 –1 O 1 2 3 and Nancy received 32% and 29% of the votes cast
respectively. Steve received 468 votes and all votes are
–1
y=x–2 valid.
(a) Find the total number of votes cast.
–2
(b) Find the number of votes received by Bill.
–3
(c) By what percentage was the number of votes for
Steve more than that for Bill?
–4
Solution
–5 (a) Percentage of votes cast received by Steve
B
= 100% – 32% – 29%
= 39%

Review Exercise 3
128
Total number of votes cast 13. The following is a sequence of patterns formed by
= 468 ÷ 39% identical square tiles.
= 1200
(b) Number of votes received by Bill
= 1200 × 32%
= 384
(c) The required percentage n=1 n=2 n=3 n=4
468 – 384
= × 100% (a) Draw the pattern for n = 5.
384
7 (b) Let Tn be the number of tiles in the nth pattern.
= 21 %
8 (i) Write down the values of Tn for n = 1 to 5.
(ii) Express Tn in terms of n.
12. (a) When a piece of meat is taken out from a freezer,
Solution
its temperature is –15 °C. It is then placed in a
(a) The pattern for n = 5 is shown below.
microwave oven for defrosting. Its temperature
becomes –3 °C after heating for 90 seconds. If the
temperature of the meat rises at a constant rate, find
(i) the rate of increase of the temperature of the
meat in °C/s,
(ii) the temperature of the meat after heating for
2 minutes.
(b) In a 100-metre dash, the average speed of
John in the first 60 m was 8 m/s and he finished the n=5
last 40 m in 4.5 seconds. Find his average speed for (b) (i) T1 = 3
(i) the last 40 m, T2 = 8
(ii) the 100-metre dash. T3 = 15
T4 = 24
Solution T5 = 35
(a) (i) Rate of increase
(ii) If we add a square tile to the nth pattern, it
–3 – (–15)
= becomes a square with n + 1 tiles on a side.
90
2 \  Tn = (n + 1)2 – 1
= °C/s
15
(iii) If the top row of the nth pattern is cut and
(ii) Increase in temperature after attached to its right-hand side, the pattern will
Heating for 2 minutes become a rectangle of n by n + 2 tiles.
2
= × 2 × 60 \  Tn = n(n + 2)
15
= 16 °C
14. Jenny was 3 m from a lamppost. She began to run at a
Temperature after 2 minutes constant speed away from the lamppost. Her distance,
= –15 °C + 16 °C y m, from the lamppost after t seconds is given by the
= 1 °C function y = 2t + 3. The following diagram shows the
(b) (i) Average speed for the last 40 m graph of the function for 0 < t < 10.
40 y
=
4.5
= 8 m/s 25

(ii) Time taken for the first 60 m 20


= 60 ÷ 8
= 7.5 s 15 y = 2t + 3

\  Average speed for the 100 m dash


10
100
=
4.5 + 7.5
5
1
= 8 m/s
3
O t
2 4 6 8 10

129
(a) Find the time that she was 15 m from the lamppost.
(b) Find the gradient of the graph.
(c) What is the physical interpretation of the gradient
of the graph.

Solution
(a) When y = 15,
15 = 2t + 3
2t = 12
t = 6
(b) Gradient = 2
(d) The gradient is the speed of Jenny in m/s.

15. In an ant colony, the ratio of the number of red ants to


that of black ants is 5 : 2. The number of red ants is
decreased by 30%.
(a) Find the percentage increase in the number of black
ants so that there will be an equal number of red
ants and black ants in the colony.
(b) If the original number of ants in the colony is 420,
find the number of red ants after the change.

Solution
(a) Let 5x and 2x be the number of red ants and black
ants respectively.
Decreased number of red ants
= 5x × (1 – 30%)
= 3.5x
Percentage increase in black ants
3.5x – 2x
= × 100%
2x
= 75%
(b) 5x + 2x = 420
7x = 420
x = 60
Number of red ants after the change
= 3.5 × 60
= 210

Review Exercise 3
130
13 Simple Inequalities

Class Activity 1
Objective: To recognise the multiplication properties of inequalities.

Tasks
Fill in each blank with an inequality sign ‘.’ or ‘ ,’.

1. (a) 3 × 2 , 7 × 2 2. (a) –5 × 3 , –1 × 3

(b) 3 × 5 , 7 × 5 (b) –5 × 6 , –1 × 6

3. (a) 6 × (–2) . 12 × (–2) 4. (a) –2 × (–5) . 4 × (–5)

(b) 6 × (–8) . 12 × (–8) (b) –2 × (–9) . 4 × (–9)

5. (a) –8 × (–1) . –3 × (–1)

(b) –8 × (–7) . –3 × (–7)

Questions
1. Observe the results above, what do you realise?

If a , b and c . 0, then ac , bc. If a , b and c , 0, then ac . bc.

2. When both sides of an inequality are multiplied by a positive number, say 2, what would be the resulting inequality? Will
the inequality sign change?

a , b
a × 2 , b × 2
2a , 2b

When both sides of an inequality are multiplied by a positive number, the inequality sign will remain the same.

3. When both sides of an inequality are multiplied by a negative number, say , what would be the resulting inequality? Will
the inequality sign change?

a , b
a × (–3) . b × (–3)
–3a . – 3b

When both sides of an inequality are multiplied by a negative number, the inequality sign will change.

131
Try It (b) 14x  21
1 1
Section 13.1 × 14x  × 21
14 14
1. Use an inequality to represent a relationship involving 3
the given variable in each of following statements. x 
2
(a) The pH value, x, of a face cleanser is less than 7.
(b) The number of passengers, n, of a taxi should not The solution is represented on a number line as
exceed 4. follows:
(c) The area, y m2, of a flat is greater than 89 m2. 3
x<
(d) The volume, V cm3, of juice in a can is at least 2
375 cm3.
(e) The mass, m kg, of a car is more than 1100 kg. – 1 0 1 2 3
(f) The height, H cm, of a basketball player is at least
190 cm.
4. Solve each of the following inequalities.
Solution (a) –3x . 15 (b) –10x  – 4
(a) x , 7 (b) n  4
(c) y . 89 (d) V  375 Solution
(e) m . 1100 (f) H  190 (a) –3x . 15
(–1) × (–3x) , (–1) × 15
2. Represent each of the following inequalities using a 3x , –15
number line and name three solutions for x. 1 1
× 3x , × (–15)
4 3 3
(a) x , –2 (b) x  – x , –5
7

Solution (b) –10x  –4


(a) x , –2
(–1) × (–10x)  (–1) × (–4)
– 4 – 3 –2 –1 0 10x  4
1 1
Three possible solutions for x are –3, – 4.7 and × 10x  ×4
10 10
–50. 2
x 
(b) 4 5
x–
7
5. Find the smallest integer x that satisfies the inequality
–2 – 1 0 1 2 –3x  7.

Three possible solutions for x are –0.5, 0 and 12. Solution


–3x  7
(–1) × (–3x)  (–1) × 7
3. Solve each of the following inequalities and represent
3x  –7
the solution on a number line.
1 1
(a) 2x . –6 (b) 14x  21 × 3x  × (–7)
3 3
7
Solution x  –
3
(a) 2x . –6
1 1 7
× 2x . × (–6) x–
2 2 3
x . –3

The solution is represented on a number line as – 4 – 3 – 2 – 1 0
follows:
x,–3
As illustrated by the number line, the greatest integer x
7
bigger than or equal to – is –2.
3
–5 – 4 –3 –2 –1

Chapter 13  Simple Inequalities


132
Section 13.2
6. The maximum load that a small trolley can carry is 40 kg.
If the mass of each carton of goods is 5 kg, find the possible
numbers of cartons the trolley can carry at any one time.

Solution
Let x be the possible number of cartons.
5x  40
1 1
(5x)  × 40
5 5
x  8
The possible number of cartons is 1, 2, 3, 4, 5, 6, 7, 8.

7. A bowl of congee contains 70 g of carbohydrate. Suppose


the daily minimum carbohydrate requirement of a man is
130 g and he only eats congee on a day, find the minimum
whole number of bowls of congee that the man should
eat to meet the minimum daily carbohydrate requirement.

Solution
Let x be the number of bowls of congee that the man
should eat per day.
70x  130
1 1
(70x)  × 130
70 70
6
x  1
7

The man should eat at least 2 bowls of congee per day.

8. The operating cost of a gymnasium is $12 000 a month.


The monthly fee of each member of the gymnasium is
$70. Find the minimum number of members required
such that the gymnasium can make a profit each month.

Solution
Let n be the number of members required.
Total monthly fees = $70 × n
= $70n
We require 70n  12000
12000
n 
70
3
n  171
7
The minimum number of members is 172.

133
Exercise 13.1 Solution
Level 1 (a) 3  6
∴  x = 3 is a solution of x  6.
1. Use an inequality to represent a relationship involving
the given variable in each of the following statements. (b) 2  4
(a) The noon temperature, T  °C, is more than 30 °C. ∴  x = 2 is NOT a solution of x  4.
(b) The height, H m, of a basketball player is at least (c) –1  –5
1.8 m. ∴  x = –1 is a solution of x  –5.
(c) The combined length, width and height, C cm, of
an airline hand luggage should not exceed 115 cm. (d) –3 = –3
(d) The passing mark of a test is 50 marks. Larry got ∴  x = –3 is a solution of x  –3.
x marks and he failed in the test.
(e) The time taken, t minutes, to travel from the Changi 4. Solve each of the following inequalities and represent
Airport to Jurong is more than 45 minutes. the solution on a number line.
(f) The daily sugar intake, m g, of a patient should not (a) 2x , 10 (b) 3x . 18
be more than 36 grams. (c) 4x  0 (d) 5x  –20
(e) –6x . 9 (f) –8x , 12
Solution (g) –14x  –35 (h) –16x  –88
(a) T . 30
(b) H  1.8 Solution
(c) C  115 (a) 2x , 10
(d) x , 50 1 1
(e) t . 45 × 2x , × 10
2 2
(f) m  36 x , 5

2. Represent the solutions of each of the following (b) 3x . 18


inequalities on a number line. 1 1
× 3x . × 18
3 3
(a) x , 5 (b) x . –6
3 1
x . 6
(c) x  (d) x  –2
4 7
(c) 4x  0
Solution 1 1
(a) x,5
× 4x  × 10
4 4
x  0
2 3 4 5 6 7
(d) 5x  –20
(b) x . –6 1 1
× 5x  × –20
5 5
– 8 – 7 –6 –5 –4 –3 x  –4

(c) 3 (e) –6x . 6


x
4 1 1
– × (–6x) , – ×9
6 6
– 2 – 1 0 1 2 3 3
x , –
2
(d) x < –2
1
7 (f) –8x . 12
1 1
– × (–8x) . – × 12
0 – 1 – 2 – 3 – 4 –5 8 8
3
x , –
2
3. Determine whether the given value of x in each case is
a solution of the given inequality. (g) –14x  –35
1 1
(a) x  6; x=3 – × (–14x)  – × (–35)
14 14
(b) x  4; x=2
5
(c) x  –5; x = –1 x 
2
(d) x  –3; x = –3

Chapter 13  Simple Inequalities


134
(h) –16x  –88 Level 3
1 1 7. Find the greatest integer x that satisfies each inequality.
– × (–16x)  – × (–88)
16 16 (a) 11x  36 (b) 8x  72
11 (c) –3x  –8 (d) –5x  – 30
x  –
2
Solution
(a) 11x  36
Level 2 36
x 
5. Solve each of the following inequalities. 11
(a) 3x . 11 + 19 (b) 7x , 25 – 88 3
x  3
11
2 3
(c) –5x  –16 + 23 (d) –4x  –2 – 1 The greatest integer x is 3.
5 5
(b) 8x  72
Solution
1
(a) 3x . 11 + 19 x  × 72
8
3x . 30 x  9
x . 10 The greatest integer x is 9.
(b) 7x , 25 – 88 –3x  –8
(c)
7x , –63
x . –9
3x  8
(c) –5x  –16 + 23 8
x 
–5x  7 3
2
7 x  2
x  – 3
5
2 The greatest integer is 2.
x  –1
5 –5x . –30
(d)
2 3 x  30
(d) – 4x  –2 – 1
5 5 x  6
–4x  –4 The greatest integer is 5.
4x  –4
x  1
8. Find the smallest integer x that satisfies the given
inequality.
6. Solve each of the following inequalities. (a) 4x  12 (b) 5x . –64
(a) x + x  3 (b) 5x – 2x , –24 (c) – 6x  71 (d) –13x  –52
(c) –7x + 6x . –10 (d) – 4x – 8x  –30
Solution
Solution (a) 4x  12
(a)
x + x  3 1
x  × 12
2x  3 4
3 x  3
x 
2 The smallest integer x is 3.
1
x  1 (b) 5x . –64
2
64
(b) 5x – 2x , –24 x .
5
3x , –24 4
x , –8 x . –12
5
–7x + 6x . –10
(c) The smallest integer x is –12.
–x . –10 – 6x , 71
(c)
x , 10 6x . –71
71
– 4x – 8x  –30
(d) x . –
6
–12x  –30
5
5 x . –11
x  6
2
1
The smallest integer x is –11.
x  2
2

135
(d)
–13x  –52 Solution
13x  –52 (a) The amount Jinlan gets = $15 × x
x  4 = $15x
The smallest integer x is 4. (b) 15x  75
1
x  × 75
15
9. (a) Find the smallest prime number which satisfies the x  5
inequality 3x . 64. She works for less than 5 hours.
(b) Find the largest prime number which satisfies the
inequality – 4x  – 49.
2. The price of a concert ticket is $50.
Solution (a) Find the total price for x tickets in terms of x.
(a) 3x . 64 (b) Henry has $200 to spend on tickets. Find the
64 possible number of concert tickets he can buy.
x .
3
1 Solution
x . 21
3 (a) The total price for x tickets = $50 × x
The smallest prime number x is 23. = $50x
– 4x  – 49
(b) (b) 50x  200
–49 1
x < x
 × 200
–4 50
1
x < 12 x  4
4
The largest prime number x is 11. The possible numbers of concert tickets that he can
buy are 1, 2, 3 and 4.

10. Write down an inequality of the form ax  b, where 3. A pack of instant noodles contains 800 mg of sodium.
a ≠ 1, such that its solution is x  7. (a) Express the amount of sodium in x packs of instant
noodle in terms of x.
Solution (b) If the recommended maximum of daily intake of
One possible inequality is sodium is 2300 mg, find the maximum number of
5x  35. packs of instant noodles that you should eat on a
The solution is given by day so that the sodium intake is less than 2300 mg.
1
x  × 35. Solution
5
(a) The amount of sodium
i.e.  x  7.
= 800x mg
(b) 800x  2300
11. Write down an inequality of the form ax , b, where 23000
a ≠ 1, such that its solution is x . –3. x 
800
27
Solution x 
8
One possible inequality is –5x < 15. The maximum number of packs is 2.
The solution is given by
15
x > 4. The mass of a book is 400 g.
–5
i.e. x > –3. (a) Find the total mass of x books in terms of x.
(b) If the mass of a pile of such books is more than
7 kg, find the minimum number of books in the pile.

Exercise 13.2 Solution


Level 1 (a) Total mass = 400x g
1. Jinlan works in a school and is paid by the number of (b) 400x . 7000
hours she works. Her hourly rate of pay is $15. 7000
(a) Find her earnings for working x hours in terms of x .
400
x. 171
x .
(b) What are the possible numbers of hours she would 2
have worked if she earns less than $75? The minimum number of books is 18.

Chapter 13  Simple Inequalities


136
Level 2 Solution
5. Mr Lee wants to travel more than 100 km in 2 hours. Let $x be the marked price of a dress.
What should his average speed of driving be?
x(1 – 20%) < 250
0.8x < 250
Solution 250
x <
Let x km/h be the required speed. 0.8
2x  100 x < 312.5
x  50 Tthe range of the marked price is
His average speed should be greater than 50 km/h. $0 < marked price < $312.50.

6. The time taken by a carpenter to make a chair is 3 hours. 10. A school needs to raise $45 000 to equip a multimedia
Find the maximum number of chairs he can make if he room. The school has 800 students.
has 17 hours to make them. (a) If each student donates the same amount, how
much would each student need to donate to meet
Solution the target?
Let x be the number of chairs made. (b) If each set of computer costs $1700, find the possible
3x  17 numbers of sets of computers that can be bought
17
x  with $45 000.
3
52
x  Solution
3
(a) Let $x be the amount that each student needs to
The maximum number of chairs made is 5. donate.
800x = 45 000
7. A car park in the city area charges $4 per hour and part 45 000
x =
thereof. If you have $25, find the maximum number of 800
hours that you can park your car in this car park. x = 56.25
Each student needs to donate $56.25.
Solution
Let x be the number of hours of parking. (b) Let y be the possible numbers of sets of computers
4x < 25 that can be bought.
25 1700y  45 000
x <
4 1
61
y  × 45 000
1700
x <
4
450
The maximum number of hours of parking is 6. y 
17
8
y  26
8. The selling price of a bouquet of flowers is $25. Find the 17
minimum number of bouquets of flowers a florist must The possible numbers of sets of computers that can
sell if she wants her total sales to be more than $360. be bought are 1, 2, 3, ..., 26.

Solution
11. The length of a rectangle is twice its width. Let the width
Let x be the number of bouquets of flowers sold.
be x cm.
25x  360
(a) Express the perimeter of the rectangle in terms of
1
x  × 360 x.
25
72
(b) If the perimeter of the rectangle is 102 cm, find its
x  width.
5
2 (c) If the perimeter of the rectangle is less than
x  14 120 cm, find the range of possible values of the
5
The minimum number of bouquets of flowers she must width.
sell is 15.
Solution
(a) Length of the rectangle = 2x cm
Level 3 Perimeter of the rectangle = 2(2x + x)
= 6x cm
9. A fashion boutique offers 20% discount for its dresses.
If your budget for a dress is $250, find the range of the
marked prices of dresses that fits your budget.

137
(b) 6x = 102 Solution
102 Let x years old be the age of De Ming.
x =
6 His father’s age = 4x years.
x = 17 4x + x  50
5x  50
The width of the rectangle is 17 cm.
50
x 
(c) 6x  120 5

1 x  10
x  × 120
6 Since De Ming’s father should be at least 18 years old
x  20 when De Ming was born, the possible ages of De Ming
are 6, 7, 8, 9 or 10 years old.
The range of the width is 0  width  20.

12. The diagram shows a 3-tier bookshelf of height x cm Revision Exercise 13


1
and width y cm. The width is 1 times the height. 1. Solve the following inequalities and represent each
2
(a) Express y in terms of x. solution on a number line.
(b) Express the total length of wooden plank needed (a) 3x  24 (b) 5x > 75
to make the bookshelf in terms of x. (c) –7x < 21 (d) –9x . –8
(c) If there are 7.2 m of wooden plank available,
what is the maximum width of the bookshelf in
Solution
centimetres?
(a) 3x  24
1
x  × 24
3
x  8
x,8
x cm
5 6 7 8 9 10

(b) 5x  75
y cm
1
x  × 75
5
Solution x  15
3 x > 15
(a) y  x
2
(b) Total length 13 14 15 16 17 18
= 2x + 4y
3
= 2x + 4 x (c)
–7x < 21
2
7x < 21
= 8x cm x < –3
(c) 8x  720 x < –3
720
x 
8
– 6 –5 – 4 –3 –2 –1
x  90
Maximum value of x = 90 (d)
–9x . –8
Hence, maximum width 9x , –8
3 9
= × 90 x 
2 8
= 135 cm 1
x  1
8
1
13. The age of De Ming’s father is 4 times that of De Ming. x,1
8
The sum of the ages of De Ming and his father is not
more than 50 years. Find two possible ages for De Ming.
– 2 –1 0 1 2 3

Chapter 13  Simple Inequalities


138
2. Find the greatest integer x that satisfies each inequality. 5. (a) Simplify 3(5x – 4) + 2(x + 6).
(a) 8x < 32 (b) 2x + x  –31 (b) Hence solve the inequality
3(5x – 4) + 2(x + 6)  –51.
Solution
(a) 8x  32 Solution
1 (a) 3(5x – 4) + 2(x + 6) = 15x – 12 + 2x + 12
x  × 32
8 = 17x
x  4
(b) 3(5x – 4) + 2(x + 6)  –51
The greatest integer x is 4.
17x  –51
(b) 2x + x  –31 –51
x 
3x  –31 17
31 x  –3
x  –
3
1
x  –10 6. (a) Solve the inequalities
3
The greatest integer x is –11. (i) –8x  40, (ii) 6y < – 42.
(b) P(x, y) is a point on the Cartesian plane such that
–8x  40 and 6y < – 42. In which quadrant does
3. Find the smallest integer x that satisfies each inequality.
the point P lie?
(a) 2x  –9 (b) 3x – 7x < –17
Solution
Solution
(a) (i)
–8x  40
(a) 2x  –9
40
9  
x 
 x  – 8
2
  1  
x  5
 x  – 4
2
(ii) 6y  – 42
The smallest integer x is – 4.
1

y  × (– 42)
(b) 3x – 7x < –17 6
–4x < –17 y  –7

4x < –17
(b) Since x is positive and y is negative,
17
x  P(x, y) lies in the fourth quadrant.
4
1
x  4
4
7. When a stone is dropped from a cliff, its speed, v m/s,
The smallest integer x is 4. at time t seconds is given by v = 5t for 0 < t < 6.
(a) Sketch the graph of v = 5t.
4. (a) Find the prime factorisation of 5390. (b) When will the speed of the stone be more than
(b) Find the factors of 5390 that satisfy the inequality 20 m/s?
7x  105.
Solution
Solution (a) v = 5t
(a) When t = 0, v = 5 × 0 = 0.
5390 When t = 6, v = 5 × 6 = 30.
The following diagram is a sketch of the graph of
10 539 v = 5t.

2 5 7 77

7 11

∴ 5390 = 2 × 5 × 72 × 11
(b) 7x  105
105
 x 
7
 x  15
The required factors are 1, 2, 5, 7, 10, 11, 14.

139
V Solution
(a) 4x  60
30 60
 x 
4
 x  15
25 The maximum percentage discount is 15%.
V = 5t
(b) (i) Selling price of the lamp
20 = $200 × (100% – 15%)
= $170
15 (ii) Marked price of the rack
= $51 ÷ (100% – 15%)
10 = $60

10. For a maintenance job, a plumber offers his clients two


5 schemes as follows:
Scheme A: a charge of $200 regardless of the time
t needed for the job,
O 1 2 3 4 5 6
Scheme B: a charge of $70 per hour.
Let $y be the charge for a job that takes t hours to
complete.
(b) Speed . 20
(a) (i) Find y under scheme A.
5t . 20
20
(ii) Express y in terms of t under scheme B.
t . (b) On the same diagram, draw the graphs of
5
t . 4 the equations involving y obtained in (a) for
Since 0 < t < 6, 0 , t , 4.
4 , the required time < 6. (c) What would be the minimum duration of a job if
the charge under scheme A is not more than that
under scheme B?
8. Consider the sequence 3, 6, 9, 12, ... .
(a) Find the general term of the sequence. Solution
(b) Find the 11th term of the sequence. (a) (i) y = 200
(c) What is the largest term of the sequence that is less (ii) y = 70t
than 101? (b) The graph of y = 200 and y = 70t are shown below.
y
Solution
(a) The general term = nth term 300
= 3n
(b) The 11th term = 3 × 11
= 33
(c) 3n  101
y = 200
101 200
 n 
3
2
 n  33
3
The largest possible n is 33.
Since 3 × 33 = 99, 100 y = 70t
the largest term of the sequence that is less than
101 is 99.

9. A shop offers x% discount on goods such that 4x < 60. O t


1 2 3
(a) Find the maximum percentage discount.
(b) A desk lamp and a magazine rack are sold at the
maximum percentage discount. (c) 70t . 200
(i) If the marked price of the desk lamp is $200, 200
t .
find its selling price. 70
(ii) If the selling price of the magazine rack is 26
t .
$51, find its marked price. 7
6
The minimum duration is 2 hours.
7
Chapter 13  Simple Inequalities
140
14 Perimeter and Area of
Plane Figures

Class Activity 1
Objective: To make the connection between the area of a parallelogram and that of a rectangle.

D C D C D C

height

A B A B N M
N MN AB
base

A parallelogram is a quadrilateral with parallel opposite sides. The diagram on the left above shows a parallelogram ABCD
in which AB // DC and AD // BC. The perpendicular DN from D to AB is called the height of the parallelogram corresponding
to the base AB. Imagine that we cut the parallelogram along DN as shown in the middle diagram and then place ADM to the
other side to form the quadrilateral NMCD as shown on the right.

Questions
1. Name the quadrilateral NMCD.

NMCD is a rectangle.

2. Express the area of NMCD in terms of NM and DN.

Area of NMCD = NM × DN

3. Observe and suggest the relationship between AB and NM.

AB = NM

4. What is the relationship between the area of ABCD and the area of NMCD?

Area of ABCD = Area of NMCD

5. If you are tasked to suggest a formula for the area of the parallelogram ABCD, what would that be?

Area of ABCD = NM × DN = AB × DN = base × height

6. If a parallelogram PQRS is given as shown on the right, how could you cut and S R
assemble it into a rectangle with PQ as a side?

P Q
Draw a perpendicular PN from P to SR. Cut along PN and place ∆PSM to the other side to form quadrilateral PQMN.

141
Class Activity 2
Objective: To identify the height corresponding to any given side of a triangle or a parallelogram that is taken as the base.

Questions
1. Identify the height corresponding to the side AB in each of the following triangles.

(a) Height = CE (b)


Height = CE (c) Height = CE

C C A
D
D
F C
B
A B E
A B
D E E

2. Draw the height corresponding to the base PQ in each of the following triangles.

(a) (b) (c) Q
P P

R
P
Q R
R
Q

3. Identify the height corresponding to the side AB in each of the following parallelograms.

(a) Height = DE (b)


Height = DE (c)
Height = DE

D C F B C
F
C
A D B
F

D E
A B A
E
E

4. Draw the height corresponding to the base PQ in each of the following parallelograms.

(a) (b) Q
(c)
R Q Q

P
R P R

S P S
S

Chapter 14  Perimeter and Area of Plane Figures


142
Class Activity 3
Objective: To make connection between the area of a trapezium and that of a parallelogram.

A a B a a B, G b H
E F A

h h h h

D C H G D E
b b b C, F a
trapezium trapezium

In the diagram above, ABCD and EFGH are two identical trapeziums in which AB = EF = a, DC = HG = b and
the height of each trapezium is h. Let us place these two trapeziums together to form the quadrilateral AHED.

Questions
1. Can you name the quadrilateral AHED?

AHED is a parallelogram.

2. Express the length of AH in terms of a and b.

(a + b)

3. Write an expression for the area of AHED in terms of a, b, and h.

Area of AHED = (a + b)h

4. Hence deduce the area of the trapezium ABCD in terms of a, b and h.


1 1
Area of trapezium ABCD = × Area of AHED = (a + b)h
2 2

143
Class Activity 4
Objective: To identify the height corresponding to the parallel sides of a trapezium.

Questions
1. Identify the height corresponding to the side AB in each of the following trapeziums.

(a) Height = GE (b)


Height = BE (c)
Height = BE
C
D G C D

A E

D
A E F B
B E
B

A
C

2. Draw the height corresponding to the side PQ in each of the following trapeziums.

(a) (b) (c)


S R Q S

P
R R
P Q
Q
S P

Chapter 14  Perimeter and Area of Plane Figures


144
Class Activity 5
Objective: To explore how larger shapes can be formed using identical trapeziums.

Tasks
You are given 4 identical table tops, each in the shape of a trapezium as shown below. The dimensions are in centimetres. These
table tops are arranged to form a larger table for a class activity.

D 40 C

50 40

A 70 B

1. What shapes can be formed? Sketch the shapes that you can make.

2. Find the perimeter of each shape formed.

Suggested Answer:

Shape Perimeter

110

80 80 2(110) + 2(80) = 380

110

110

100 100 2(110) + 2(100) = 420

110

145
Try It! Solution
(a) Let r cm be the radius of the circle.
Section 14.1
2rr = 12r
1. The length and breadth of a rectangle are 40 cm and r = 6
10 cm respectively. A square of side y cm has the same The radius is 6 cm.
area as the rectangle. Find
(a) the area of the rectangle, (b) Area of the circle = r × 62
(b) the value of y, = 36r cm2
(c) the perimeter of the square.
4. In the figure, PQRS is a parallelogram, RS = 12 cm,
Solution PS = 6 cm and SM = 5 cm. It is given that RN is
(a) Area of the rectangle = 40 × 10 perpendicular to PS produced and PQ  SM.
= 400 cm2
(b) Area of the square = Area of the rectangle
y2 = 400
y = 400
= 20
(c) Perimeter of the square = 4y cm
= 4 × 20 cm
= 80 cm (a) Identify the height corresponding to the base PS
in the parallelogram PQRS.
(b) Find the area of the parallelogram.
2. In the figure, AB = 5 cm, BC = 4 cm and AC = 3 cm.
(c) Find the length of RN.

Solution
(a) Height = NR
(b) Area of PQRS = RS × SM
= 12 × 5
= 60 cm2
(c) PS × RN = Area of PQRS
Find 6 × RN = 60
(a) the area of ABC, RN = 10 cm
(b) the length of CD.
5. In the figure, ABCD is a square of side 24 cm. M and N
Solution are points on AB and DC respectively such that MBND
1
(a) Area of ABC = × BC × CA is a parallelogram and MB = 12 cm.
2
(a) Find the area of MBND.
1 (b) Find the ratio of the area of MBND to that of ABCD.
= ×4×3
2
(c) How is the ratio in (b) related to the ratio of MB
= 6 cm2 to AB?
1 D N C
(b) × AB × CD = 6
2
1
× 5 × CD = 6
2
12
CD = 24
5
= 2.4 cm

3. The circumference of a circle is 12r cm. Find A M 12 B


(a) the radius of the circle,
(b) the area of the circle (leave your answer in terms Solution
of r). (a) Area of MBND = 24 × 12
= 288 cm2

Chapter 14  Perimeter and Area of Plane Figures


146
(b) Area of MBND : Area of ABCD (a) the height from B to DC,
= 288 : 242 (b) the area of ABC as a percentage of the area of
= 288 : 576 ABCD.
=1:2
Solution
(c) AB = BC (sides of square)
(a) Let h be the height from B to DC.
= 24 cm
MB : AB = 12 : 24 1
(AB + DC)h = 10
=1:2 2
1
∴  MB : AB = Area of MBND : Area of ABCD (3 + 5)h = 10
2
4h = 10
6. In the figure, ABCD is a parallelogram and BC = 12 cm. h = cm
1
E is a point on BC such that the area of ABE is that The height from B to DC is 2.5 cm.
5
of parallelogram ABCD, find the length of BE. (b) Area of ABC
1
= × AB × h
2
1
= × 3 × 2.5
2
= 3.75 cm2
3.75
Required percentage = × 100%
10
Solution = 37.5%
Let h cm be the height from A to the base BC.

Area of ABE =
1
Area of ABCD 9. The side wall ABCD of a shed is in the shape of a
5 trapezium. Given that AB = 2.5 m, BC = 3 m and the

1
× BE × h =
1
× 12 × h area of ABCD is 5.5 m2, find the length of AD.
  2 5
24 E
BE = D C
5
= 4.8 cm 36

A B
50
Section 14.2
7. In the figure, PQRS is a trapezium in which PQ = 14 cm, Solution
SR = 8 cm and PS = 10 cm. Find the area of PQRS. Area of ABCD =
1
× (AD + 3) × 2.5
2
1
× (AD + 3) × 2.5 = 5.5
2
AD + 3 = 4.4
AD = 1.4 m

Solution
Section 14.3
10. In the figure, STP is a semicircle R
1
Area of PQRS = × (8 + 14) × 10 of diameter 30 cm, PQRS is a S
2 U
parallelogram, PQ = 16 cm and 15
= 110 cm2
SU = 15 cm. Take r to be 3.142
and giving your answers correct T 30
8. In the figure, the trapezium ABCD is a flowerbed in which to 3 significant figures, find
AB = 3 m and DC = 5 m. If the area of the flowerbed is (a) the perimeter of the figure, Q
10 m2, find B C (b) the area of the figure. 16
P

5
A

147
Solution 12. In the figure, ABCD and ABEF are two equal
(a) Perimeter of the figure parallelograms with common side. AB = 20 cm and the
= semicircle STP + PQ + QR + RS height of each parallelogram with respect to the base AB
1 is 30 cm. G, the intersection of AD and BE, is 16 cm
= × r × 30 + 16 + 30 + 16
2 from AB. Find
= 15(3.142) + 62
= 109.13 F E D C
= 109 cm  (correct to 3 s.f.)
G
(b) Area of the figure 30
= Area of semicircle STP 16
+ Area of parallelogram PQRS
A 20 B
1
= × r × 152 + 30 × 15
2
= 803.475
(a) the area of ABCD,
= 803 cm2  (correct to 3 s.f.)
(b) the area of ABG,
(c) the area of the figure.

11. The figure shows the shape of a cross-section of a stool.
AD = 40 cm, EH = 30 cm, BC = 24 cm, FG = 16 cm, Solution
the distances of BC and FG from the ground are 25 cm (a) Area of ABCD
and 15 cm respectively. Find the area of the figure. = 20 × 30
= 600 cm2
B 24 C
(b) Area of ∆ABG
F 16 G 1
25
= × 20 × 16
2
15 = 160 cm2
A E H D (c) Area of DCBG
30 = Area of ABCD – Area of ABG
40 = 600 – 160
= 440 cm2
Solution Area of figure
Area of ABCD =
1
× (24 + 40) × 25 = Area of DCBG + Area of FEGA + Area of ABG
2 = 440 + 440 + 160
= 800 cm2 = 1040 cm2
1
Area of EHGF = × (16 + 30) × 15
2
= 345 cm2
Area of the figure = (800 – 345)
= 455 cm2

Chapter 14  Perimeter and Area of Plane Figures


148
Exercise 14.1 Level 2
Level 1 3. In the figure, ABCD is a parallelogram with AB = 4 cm
1. Find the area of each parallelogram, where the unit of and EFG is a triangle with EF = 6 cm. ABEF and DCG
length is centimetre. are parallel straight lines.
(a) (b)
D C G
D C S R

4 9 6
N

P Q A 4 B E 6 F
A E B
5
(a) Find the ratio of the area of EFG to that of ABCD.
(b) If the area of EFG is 24 cm2, find the area of
(c) (d)
ABCD.
K H W 12 T

Solution
7 X (a) Let h cm be the height from D to AB.
Z
3
Area of EFG : Area of ABCD
F 7 G 1
Y = ×6×h:4×h
2
Solution =3:4
(a) Area of ABCD = 5 × 4 4
= 20 cm2 (b) Area of ABCD = 24 ×
3
(b) Area of PQRS = 6 × 9 = 32 cm2
= 54 cm2
(c) Area of FGHK = 7 × 7 4. In the figure, ABCD is a parallelogram and AD = 18 cm.
= 49 cm2 E is a point on AD produced such that the area of ABE
5
(d) Area of WXYZ = 3 × 12 is that of ABCD.
6
= 36 cm2
E

2. In the figure, ABCD is a parallelogram. E and F are D F C


points on AB and BC respectively such that AB  DE G
18
and BC  DF. Copy and complete the following table.
D C A B

(a) Find the length of DE.


F
(b) If the area of ABCD is 450 cm2, find the height
A B from B to AD.
E

Solution
AB DE BC DF Area of ABCD
(a) Let h cm be the height from B to AD.
(a) 10 6 8
5
(b) 18 9 12 Area of ABE = × Area of ABCD
6
(c) 15 8 10 1
× (18 + DE) × h =
5
× 18 × h
2 6
(d) 9 6 72
18 + DE = 30
DE = 12 cm
Solution
(b) 18h = 450
AB DE BC DF Area of ABCD h = 25
(a) 10 6 8 7.5 60 The height is 25 cm.
(b) 18 6 9 12 108
(c) 15 8 12 10 120
(d) 8 9 6 12 72

149
5. In the figure, ABCD is a rhombus, AD = 13 cm, 7. The picture below shows the national flag of the
AC = 24 cm and BD = 10 cm. Republic of the Congo. ABCD is a rectangle and APCQ
is a parallelogram. The ratio of AB to BC is 3 : 2 and
D
1
AP = AB. If AB = 48 cm, find
3
A C
E
D Q C
B

(a) State the size of AED.


(b) Find the area of ABCD.
(c) Find the height from B to AD.
A P B
Solution
(a) ∠AED = 90° (diagonals of rhombus) (a) the lengths of BC and AP,
(b) Area of ABCD (b) the area of APCQ,
= 2 × Area of ACD (c) the yellow area as a percentage of the area of the
1 flag.
=2× × 24 × 5
2

= 120 cm2 Solution
2
(c) Let h cm be the height from B to AD. (a) Length of BC = 48 ×
3
13h = 120 = 32 cm
120 1
h = Length of AP = AB
13 3
3 1
=9 = × 48 cm
13 3

The height is 9
3
cm. = 16 cm
13
(b) Area of APCQ = AP × BC
= 16 × 32
Level 3 = 512 cm2
512
6. In the figure, ABCD is a vertical wall 2 m high. It casts (c) The required percentage = × 100%
48 × 32
a shadow ABEF on the ground. ABEF is a parallelogram
1 1
in which EN is 1 times the height of the wall. = 33 %
2 3
D C
2 8. In the figure, ABCDEF is a signboard in which
A B ABCF and FCDE are parallelograms, FN = 10 cm and
N
DM = 6 cm.

F E E D
6
(a) Find the length of EN. F C
M
(b) If the area of the wall is 7.6 m2, find the area of
the shadow. 10

A N B
Solution
1
(a) The height EN = 2 × 1 (a) Find the ratio of the area of ABCF to that of FCDE.
2
=3m (b) If the area of the signboard is 320 cm2, find
(i) the area of ABCF,
7.6
(b) AB = (ii) the length of FC.
2
= 3.8 m Solution
Area of the shadow = 3.8 × 3 (a) Area of ABCF : Area of FCDE
= 11.4 m2 = FC × FN : FC × DM
= FN : DM
= 10 : 6
=5:3

Chapter 14  Perimeter and Area of Plane Figures


150
5 Solution
(b) (i) Area of ABCF = 320 ×
5+3 The following diagrams show some ways to divide a
= 200 cm2 parallelogram ABCD into two equal parts.
(ii) FC × FN = 200 D C D C
FC × 10 = 200
FC = 20 cm

A B A B
9. The figure shows a square grid in which the area of
each small square is 1 cm2. Copy the grid and draw
two parallelograms on it such that the area of each D C D C
parallelograms is 6 cm2.
M

A B A B

Note: M is the point of intersection of the diagonals.

Exercise 14.2
1 cm Level 1
1. Find the area of each trapezium.
1 cm (a) D (b)
10 C K 8 H
Solution

D F C E 9 6

E
A B F 5 G
14

R (c) (d)
A B R
S Z 5 Y
S

Q 3
7 9
4
P T
P
W 11 X T
Q
In the above diagram, ABCD, ABEF and PQRS are
parallelograms of areas 6 cm2 each. Solution
1
(a) Area of ABCD = (10 + 14) × 9
10. The figure shows one way to divide a parallelogram into 2
two identical parts by a line segment. Find as many ways = 108 cm2
as possible to divide the parallelogram into two identical (b) Area of FGHK = (5 + 8) × 6
1
parts. 2
= 39 cm2
D C 1
(c) Area of PQRS = (3 + 7) × 4
2
= 20 cm2
1
(d) Area of WXYZ = (5 + 11) × 9
2
A B = 72 cm2

2. In the figure, ABCD is a trapezium. Copy and complete


the table using the given measurements.
D a C

A E B
b

151
a b h Area of ABCD (b) Let h cm be the height from A to DC.
10h = 60
(a) 7 cm 10 cm 8 cm h = 6
(b) 5 cm 9 cm 42 cm2 1
Area of AMCD = (5 + 10) × h
(c) 13 cm 14 cm 231 cm2 2
1
(d) 11 cm 10 cm 85 m2 = (5 + 10) × 6
2
= 45 cm2
Solution

a b h Area of ABCD 5. In the figure, ABCD is a D M C


(a) 7 cm 10 cm 8 cm 68 cm2 parallelogram. N is a point
(b) 5 cm 9 cm 6 cm 42 cm2 on AB such that AN = 6 cm,
NB = 12 cm and M is the
(c) 13 cm 20 cm 14 cm 231 cm2 A 6 N B
midpoint of CD. 12
(d) 6 cm 11 cm 10 cm 85 m2 (a) Find the length of CM.
(b) Find the ratio of the area of ANMD to that of NBCM.
(c) If the area of ABCD is 200 cm2, find the area of
Level 2 ANMD.
3. In the figure, ABCD is a rectangle. E and F are points
on AB and CD respectively. It is given that AE = 3 cm, Solution
EB = 5 cm, BC = 5 cm and DF = 6 cm. Find (a) CD = BA  (opp. sides of //gram)
(a) the length of CF, 2CM = 6 + 12
(b) the area of AECF, CM = 9 cm
(c) the shaded area as a percentage of the area of (b) Let h cm be the height from D to AB.
ABCD. Area of ANMD : Area of NBCM
D 6 F C 1 1
= (6 + 9) × h : (12 + 9) × h
2 2

5
= 15 : 21
=5:7
5
A B
(c) Area of ANMD = 200 ×
3 E 5 5+7
1
Solution = 83 cm2
3
(a) DC = AB  (opp. sides of rectangle)
6 + CF = 3 + 5
∴  CF = 2 cm 6. In the figure, ABCD is a trapezium, D 10 C
1
(b) Area of AECF = (3 + 2) × 5 AB = 18 cm and CD = 10 cm. The
2
area of ACD = 75 cm2.
= 12.5 cm2
Find the area of ABCD.
A B
4. In the figure, ABCD is a parallelogram, M is the midpoint 18
of AB and CD = 10 cm. Solution
D 10
C Area of nACD :
DC : AB = Area of nABC
75 10
=
Area of ABC 18
A M B
18
(a) Find the length of AM.
∴  Area of ABC = 75 ×
10
(b) If the area of ABCD is 60 cm2, find the area of = 135 cm2
AMCD.
Area of ABCD = Area of ACD + Area of ABC
Solution = 75 + 135
(a) AB = DC  (opp. sides of //gram) = 210 cm2
2AM = 10
AM = 5 cm

Chapter 14  Perimeter and Area of Plane Figures


152
Level 3 (b) BM =
1
BC
2
7. The figure shows a window frame ABCD.
1
C
= × 36
2
0.5 m = 18

D 1
Area of ABMG = × (54 + 27) × 18
0.75 m 2
= 729 cm2
0.45 m
(c) Red area = Area of ABMG
= 729 cm2
A 0.4 m B 729
The required percentage = 54 × 36
× 100%
(a) Find the perimeter of the frame. = 37.5%
(b) Find the area enclosed by the frame.
(c) If the cost of the frame is $30 per metre and the
cost of the glass panel used to fit the window is 9. In the figure, AB, CD and 30
$150 per m2, find the cost of making the window. EF are three parallel rungs E F
of a ladder which are of
h
Solution equal distance, h cm, apart.
(a) Perimeter of the frame AB = 50 cm, CD = 40 cm and 40
C D
= 0.4 + 0.75 + 0.5 + 0.45 EF = 30 cm. The area of ABDC
= 2.1 m is 1260 cm2. Find h
(a) the value of h, 50
(b) Area enclosed by the frame (b) the area of CDFE, A B
1
= (0.45 + 0.75) × 0.4 (c) the ratio of the area of
2
CDFE to that of ABDC.
= 0.24 m2
(c) The cost of making the window
= $(30 × 2.1 + 150 × 0.24) Solution
= $99 (a) Area of ABDC = 1260
1
× (40 + 50) × h = 1260
2
8. The figure shows the national flag of the Czech Republic. 45h = 1260
G is the point of intersection of the diagonals of the h = 28
rectangle ABCD and M is the midpoint of BC.
1
The ratio of the length of AB to that of BC is 3 : 2. If (b) Area of CDFE = × (40 + 30) × 28
2
BC = 36 cm, find = 980 cm2
A B (c) Area of CDFE : Area of ABDC
= 980 : 1260
G = 7 : 9
M

10. In the figure, ABCD is a rectangular board that measures


D C 20 cm by 15 cm, and AE = BF = x cm. A trapezium
CDEF is cut out to form a side of a litter bin.
(a) the lengths of AB and GM,

(b) the area of the trapezium ABMG, D 20 C
(c) the red area as a percentage of the area of
the flag.
15
Solution
3
(a) Length of AB = 36 ×
2
A x E F x B
= 54 cm
Length of GM =
1
× AB (a) Express EF in terms of x.
2 3
1 (b) If the area of CDEF is of the area of ABCD, find
= × 54 4
2 (i) the area of CDEF,
= 27 cm (ii) the value of x.

153
Solution (b) Join C and E.
(a) EF = 20 – x – x
D
= (20 – 2x) cm
3 15 8
(b) (i) Area of CDEF = × Area of ABCD
4

=
3
× 20 × 15 E C
4
= 225 cm2 6 6

1 A 17 B
(ii) (20 + 20 – 2x) × 15 = 225
2
40 – 2x = 30 Perimeter of the figure
2x = 10 = 17 + 6 + 8 + 15 + 6
x = 5 = 52 cm
Area of the figure
= Area of ABCE + Area of CDE
Exercise 14.3 1
= 17 × 6 + × 8 × 15
Level 1 2

1. Find the perimeter and area of each figure, where the = 162 cm2
unit of length is centimetre. (c) Perimeter of the figure
(a) (b) = 27 + 23 + 5 + 23 + 15 + 13
1 D = 106 cm
D C
1 15 8
Area of the figure
F 2
= Area of ABEF + Area of BCDE
E C
E 1
1 = (15 + 27) × 5 + 5 × 20 cm2
6 6 2
A 3 B
A 17 B = 205 cm2

(d) Perimeter of the figure


(c) (d) 
Leave your answers in = 30 + 10 + r × 15 + 10
terms of r. = 50 + 15r
= 50 + 15(3.142)
C = 97.1 cm  (correct to 3 s.f.)
D
5 Area of the figure
23 C
= Area of AEDB + Area of semicircle BDC
F E B D 1
13
15
5
= 30 × 10 + × r × 152
10 2
A 27 B 20 G = 300 + 112.5r
A E
30
= 300 + 112.5(3.142)
= 653 cm2  (correct to 3 s.f.)
Solution
(a) Produce DE to meet AB at G.
2. Find the perimeter and area of each figure, where the
D 1 C unit of length is centimetre.
1 (a) A 2 H E 2 D
(b) D 4 C
F E 2
3 3 4
1 5 5
5
G F
E
A 3 G B 5
B C A 8 B
8

FE = 3 – 1
= 2 cm (c) (d) 
Leave your answers in
Perimeter of the figure terms of r.
= 3 + 2 + 1 + 1 + 2 + 1 F1 E G
G 15 F
= 10 cm 3 2.8
C
Area of the figure 2
6 D C 13 12 13
= Area of AGEF + Area of GBCD
3 2.8
= 2 × 1 + 2 × 1
A B
A 5 B D 5 E
= 4 cm2 5

Chapter 14  Perimeter and Area of Plane Figures


154
Solution E D
(a) GF = 8 – 2 – 2
= 4 cm 3
F 2 C
Perimeter of the figure
= 8 + 5 + 2 + 3 + 4 + 3 + 2 + 5
A B
= 32 cm 3
Area of the figure
= Area of ABCD – Area of EFGH Solution
= 8 × 5 – 4 × 3 (a) Perimeter of the shape
= 28 cm2 =3+2+2+3+r×1
(b) Perimeter of the figure = 10 + r
=8+5+4+4+5 = 13.1 cm  (correct to 3 s.f.)
= 26 cm (b) Area of the shape
Area of the figure 1 1
= Area of trapezium ABCD – Area of AED = × r × 12 + 3 × 2 + ×2× 3
2 2
1 1
= (8 + 4) × 5 – × 5 × 4 = 0.5r + 6 + 3
2 2
= 20 cm 2
= 9.30 cm2  (correct to 3 s.f.)
(c) Perimeter of the figure
= 5 + 3 + 2 + 3 + 1 + 6 4. In the figure, ABH is a triangle, BCGH is a trapezium
= 20 cm and DEF is a semicircle. The unit of length is centimetre.
Area of the figure Taking r to be 3.142, find the area of the figure, giving
= Area of parallelogram ABGF your answer correct to 3 significant figures.
– Area of trapezium CGED
A
1
= 5 × 5.6 – (2 + 4) × 2.8
2
24
= 19.6 cm2
(d) Perimeter of the figure 28 H
B
= 5 + r × 7.5 + 5 + 13 + 15 + 13 E
18
= 51 + 7.5r
C D F
= 74.6 cm  (correct to 3 s.f.) G
10 20 10
AE = 5 + 15 + 5
= 25 cm Solution
Area of the figure Area of the figure
= Area of trapezium AEFG = Area of ABH + Area of trapezium BCGH – Area of
– Area of semicircle BCD semicircle DEF
1 1 1 1 1
= (25 + 15) × 12 – × r × 7.52 = × 28 × 24 + × (28 + 40) × 18 – r × 102
2 2 2 2 2
= 240 – 28.125r = 791 cm2 (correct to 3 s.f.)
= 152 cm2  (correct to 3 s.f.)
5. In the figure, four identical D C
triangles such as GEF are
Level 2 cut off from a square ABCD to
3. In the figure, the shape is made up of a semicircle AFE, form a symmetrical shape. If G
a rectangle ABDE and an equilateral triangle BCD. It is AE = FB = 7 cm, EF = 10 cm
13 13
given that AE = 2 cm, AB = 3 cm and the perpendicular and GE = GF = 13 cm, find
distance from C to BD is 3 cm. Taking r to be 3.142, (a) the perimeter, A B
7 E 10 F 7
find (b) the area of the shape formed.
(a) the perimeter,
(b) the area of the shape. Solution
Give your answers correct to 3 significant figures. (a) Perimeter of the shape
= 4 × (7 + 13 + 13 + 7)
= 160 cm

155
(b) Height from G to EF Solution
1 Area of the wall
= BC
2 = Area of square ABFG + Area of parallelogram BCEF
1 + Area of semicircle CDE
= (7 + 10 + 7) 2
2 1 1
= 1 × 1 + 1 × 4 + ×r×
= 12 cm 2 2
1 = 5.39 m2  (correct to 3 s.f.)
Area of GEF = × 10 × 12
2
= 60 cm2 8. The figure shows a cross-section of an iron rail. The top
Area of the shape part consists of a rectangle and two semicircles at the
= Area of ABCD – 4 × Area of GEF ends. The bottom part is a trapezium. These two parts
= 24 × 24 – 4 × 60 are joined by a rectangle in the middle. Taking r to be
= 336 cm2 3.142, find
(a) the perimeter,
6. In the figure, ABC is a right-angled triangle and ABEF (b) the area of the cross-section.
is a parallelogram. AC and BE intersect at D. The unit 20
of length is centimetre. Find
10 10
(a) the area of ABC,
(b) the area of ABD,
(c) the area of the parallelogram ABEF, 15
(d) the area of the figure. 10 10 10
C
13 13
F E
12

D 40
18
14
10
Solution
(a) Perimeter of the cross-section
A
15
B = 40 + 13 + 10 + 15 + 5 + 2r × 5 + 20 + 5 + 15
+ 10 + 13
= (146 + 10r) cm
Solution
= 177 cm  (correct to 3 s.f.)
1
(a) Area of ABC = × 18 × 15
2 (b) Area of the cross-section
= 135 cm2 1
= (40 + 30) × 12 + 10 × 15 + 20 × 10 + r × 52
1 2
(b) Area of ABD = × 15 × 10
2 = 770 + 25r
2
= 75 cm = 849 cm2  (correct to 3 s.f.)
(c) Area of parallelogram ABEF
= 15 × 14 9. The figure shows the cross-section of a funnel. ABFG
= 210 cm2 is a trapezium of height 5 cm. BCEF is a rectangle with
CE = 10 cm and EF = 6 cm. CDE is a triangle such that
(d) Area of the figure
the perpendicular distance from D to CE is 4 cm. Find
= Area of ABC + Area of ABEF – Area of ∆ABD
(a) the area of BCDEF,
= 135 + 210 – 75
(b) the length of AG if the area of ABFG is equal to
= 270 cm2
the area of BCDEF.
A G

Level 3 5
B F
7. The figure shows the wall of a staircase. ABFG is a
6
square of side 1 m. BCEF is a parallelogram in which
10
the perpendicular distance from C to BF is 4 m. CDE is C E
4
a semicircle. Taking r to be 3.142, find the area of the
D
wall, giving your answer correct to 3 significant figures.
Solution
E
(a) Area of BCDEF
G F D = Area of BCEF + Area of ∆CDE
1 1
C = 10 × 6 + × 10 × 4
2
A 1 B 2
4 = 80 cm

Chapter 14  Perimeter and Area of Plane Figures


156
(b) Let x cm be the length of AG. 12. The diagram shows a closed plane figure with right-angled
1 vertices formed by using 12 toothpicks. Each toothpick
(x + 10) × 5 = 80
2 is 5 cm long.
2 (a) Form another two such closed plane figures with
x + 10 = 80 ×
5
12 toothpicks.
x + 10 = 32 (b) Find the perimeter and the area of the given figure,
x = 22 and that of each of the figure you formed in (a).
\  The length of AG is 22 cm. (c) Do you think that the perimeters and areas of all
such plane figures with 12 toothpicks are the same?
10. The figure is formed by a rectangular board ABCD
and a parallelogram board SREF. AD = 5 cm,
DF = FE = EC = 6 cm and the height corresponding to
the side PQ of PQRS is h cm. Find
(a) the area of ABCD.
(b) the area of PQEF.
(c) the value of h if the area of the figure is 150 cm2.
Solution
S R (a) (i)

D 6 6 6 Ch
F E
5

A P Q B

Solution
(a) Area of ABCD
= AD × DC (ii)
= AD × (DF + FE + EC)
= 5 × (6 + 6 + 6)
= 5 × 18
= 90 cm2
(b) Area of PQEF
= DA × FE
=5×6 (b) The perimeter of each figure = 5 × 12
= 30 cm2 = 60 cm
(c) Area of FERS = Area of PQRS – Area of PQEF Area of figure (i) = 15 × 15
= 6 × h – 30 = 225 cm2
= 6h – 30 Area of figure (ii) = 20 × 5 + 10 × 5
= 150 cm2
Area of figure = Area of ABCD + Area of FERS
90 + 6h – 30 = 150 (c) From (b), the perimeter will remain the same but
6h = 90 the area will differ.
h = 15 cm

11. Describe two items used in our daily life which are
composite plane figures.

Solution
• An oval sports field is composed of a rectangle and
two semicircles.
• A direction arrow on a lane of a road is composed of
a triangle and a rectangle.

157
Revision Exercise 14 (c) Length of a side of the square
1. In the figure, ABCD is a D F 15 C = 5
rectangle, AB = 32 cm, 8
BE = 13 cm, EC = 8 cm and = 2.24 cm  (correct to 3 s.f.)
E
CF = 15 cm. Find 13
(a) the perimeter of ABCD, 3. In the figure, the shaded region is formed by three
A B
(b) the area of ABCD, 32 semicircles, AB = 12 cm and BC = 6 cm. Find, in terms
(c) the area of AEF. of r,
(a) the perimeter of the shaded region,
Solution (b) the area of the shaded region.
(a) AD = 8 + 13
= 21 cm
Perimeter of ABCD = 2(32 + 21)
= 106 cm
A 12 B 6 C
(b) Area of ABCD = 32 × 21
= 672 cm2 Solution
(c) DF = 32 – 15 (a) The radii of the semicircles are 9 cm, 6 cm and
= 17 cm 3 cm.
Area of AEF Perimeter of the shaded region
= Area of ABCD – Area of ABE =r×9+r×6+r×3
– Area of CEF – Area of ADF = 18r cm
1 1
= 672 – × 32 × 13 – × 8 × 15 (b) Area of the shaded region
2 2
1 1 1

1
× 21 × 17 = r × 92 – r × 62 – r × 32
2 2 2
2
= 672 – 208 – 60 – 178.5 = 18r cm2
= 225.5 cm2
4. The figure shows the national flag of Kuwait. The ratio
of its width to its length is 1 : 2. The height of the black
2. The figure shows a ‘T’ shape with right-angled 1
vertices. CD = DE = FG = GH = 1 cm, AH = 2 cm and trapezium is of the length of the national flag. The
4
EF = 3 cm. horizontal bands, coloured in green, white and red, are
(a) Find the perimeter of the figure. of equal width. It is given that the width of the flag is
(b) Find the area of the figure. 36 cm. Find
(c) If a square is equal in area to the figure, find the (a) the area of the black trapezium,
length of a side of the square. Give your answer (b) the area of the green trapezium,
correct to 3 significant figures. (c) the area of the green trapezium as a percentage of
the area of the flag.
F 3 E
1 1
H C
G 1 1 D

A B
Solution
Solution (a) Length of the flag = 36 × 2
(a) AB = 3 – 1 – 1 = 72 cm
= 1 cm Height of the black trapezium
Perimeter of the figure 1
= 1 + 2 + 1 + 1 + 3 + 1 + 1 + 2 = × 72
4
= 12 cm = 18 cm
(b) Area of the figure Width of each horizontal band
= Area of ABCH + Area of DEFG 1
=2×1+1×3 = 36 ×
3
= 5 cm2 = 12 cm

Chapter 14  Perimeter and Area of Plane Figures


158
Area of black trapezium Solution
1 (a) Perimeter of the figure
= × (36 + 12) × 18
2 = 24 × 2 + 18 × 4
= 432 cm2 = 120 cm
(b) Area of green trapezium (b) AC = 30 cm
1 3
= × 72 + 72 × × 12 1
2 4 Height from C to BE = × 30
2
2
= 756 cm
= 15 cm
(c) The required percentage Area of the figure = 2 × 24 × 15
=
756
× 100% = 720 cm2
72 × 36
(c) Let h cm be the perpendicular distance from F
1
= 29 % to AB.
2
18 × h = 24 × 15
h = 20
5. In the figure, a rectangular picture PQRS is mounted The perpendicular distance from F to AB is
inside a picture frame of uniform width of 4 cm, 20 cm.
PQ = 25 cm and QR = 20 cm. The joints AP, BQ, CR and
DS make an angle of 45° with the sides of ABCD. Find
(a) the lengths of AB and BC, 7. The figure shown is made up of A 10 G
(b) the area of ABQP, a trapezium, a rectangle and a 12 13
13
(c) the ratio of the area of PQRS to that of ABCD. semicircle. CE = 20 cm, BC = 25 cm,
B F
AB = GF = 13 cm, AG = 10 cm and N
D C AN = 12 cm. Taking r to be 3.142, find
S R (a) the perimeter, 25
(b) the total area of the figure.
Give your answers correct to 3 C
20
E
4 20
significant figures.
25 D
P 4 Q
Solution
A B
(a) Perimeter of the cross-section
= 13 × 2 + 25 × 2 + 10 + r × 10
Solution = 86 + 10r
(a) Length of AB = 25 + 4 + 4 = 117 cm  (correct to 3 s.f.)
= 33 cm (b) Area of the cross-section
Length of BC = 20 + 4 + 4
1 1
= (10 + 20) × 12 + 25 × 20 + × r × 102
= 28 cm 2 2
= 680 + 50r
(b) Area of ABQP = 837 cm2  (correct to 3 s.f.)
1
= × (25 + 33) × 4
2
= 116 cm2 8. In the figure, ABCD is a parallelogram. E is a point
on CD and CN is perpendicular to AD produced. It is
(c) Area of PQRS = 25 × 20 given that AB = 30 cm, AD = 20 cm, DE = 10 cm and
= 500 cm2 CN = 27 cm. Find
Area of ABCD = 33 × 28 (a) the perimeter of ABCD,
= 924 cm2 (b) the area of ABCD,
Area of PQRS : Area of ABCD (c) the area of ABCE,
= 500 : 924 (d) the ratio of the area of ADE to the area of ABCE.
= 125 : 231
N
27
6. In the figure, ABEF and BCDE are two parallelograms. 10 E
D C
CD = 24 cm, DE = EF = 18 cm and AC = 30 cm. Find
(a) the perimeter of the figure, 20
(b) the area of the figure,
(c) the perpendicular distance from F to AB. A B
30

159
Solution (b) Let h cm be the height of the trapezium AMNC
(a) Perimeter of ABCD = 2(30 + 20) 1
× (17 + 34) × h = 180
= 100 cm 2
51
h = 180
(b) Area of ABCD = 20 × 27 2
2
= 540 cm2 h = 180 ×
51
(c) Let h be the height from A to CD. 1
h = 7
17
Area of ADE + Area of ABCE = 540 cm2
1
1
× 10 × h + (30 + 20) × h = 540
1 The height is 7 cm.
17
2 2
5h + 25h = 540
540 10. The figure shows a logo which is formed by two identical
h =
30 parallelograms ABCG and AEFG. The parallelograms
= 18 have a common side AG of 20 cm and a common height
30 cm. CG and AE intersect at D. The area of the logo
1
Area of ABCE = (30 + 20) × 18 is 1020 cm2. Find the perpendicular distance from D to
2
AG.
= 450 cm2
A 20 G
(d) Let h cm be the height from A to CD.
Area of ADE : Area of ABCE
1 1
= × 10 × h : (30 + 20) × h 30
2 2
D
= 10 : 50
=1:5 B C E F

Alternatively,
Area of ADE : Area of ABCE Solution
= 90 : 450 Let h cm be the perpendicular distance from D to AG.
=1:5 Area of AEFG = 20 × 30
1 = 600 cm2
Note: Area of ADE = × 10 × h
2
2 × Area of AEFG – Area of ADG = Area of the logo
1
= × 10 × 18 1
2 2 × 600 – × 20 × h = 1020
2
2
= 90 cm 1200 – 10h = 1020
10h = 180
9. The figure shows a right-angled ABC, M and N h = 18
are points on the sides AB and BC respectively. The required distance is 18 cm.
AM = MB = 8 cm, BN = NC = 15 cm, MN = 17 cm,
AC = 34 cm and MN // AC. Find
(a) the area of the trapezium AMNC,
(b) the height of the trapezium AMNC.
A

8
34
M
17
8

B N C
15 15

Solution
(a) Area of trapezium AMNC
= Area of ∆ABC – Area of ∆MBN
1 1
= × (8 + 8) × (15 + 15) – × 8 × 15
2 2
= 180 cm2

Chapter 14  Perimeter and Area of Plane Figures


160
15 Volume and Surface Area
of Solids

Class Activity 1
Objective: To visualise and draw sketches of 3D shapes from different views.

Task
Your teachers will place some large 3D objects, such as cuboids and prisms, in front of the class.

1. Look at the objects from your seat and sketch a view of each object. You may use a camera device to take pictures of the
objects and help you sketch the views.

Cube

Top view Front view Side view

Cuboid

Top view Front view Side view

Triangular prism

Top view Front view Side view

Cylinder

Top view Front view Side view

161
2. Identify those right angles on your own sketches.

Right angles are indicated in the previous sketches.

3. What do you notice when you compare your sketches with a classmate who is seated at a different place in the classroom?

The sketches are different. The view from another orientation is different.

Class Activity 2
Objective: To visualise and draw the nets of cubes, cuboids, prisms and cylinders.

Tasks
2. Cut up a container so that you can lay it flat. Draw the net of the container.

Cube

3. How else would you cut up the same container to get a different net? Draw this net.

Cube

           

           

4. What do you observe when you compare your nets with those of your classmates for the containers of the same shape?

A container can have more than one net.

5. How is the surface area of a container related to its net?



The surface area of a container is the area of its net.

Chapter 15  Volume and Surface Area of Solids


162
Try It! Section 15.2
Section 15.1 3. The figure shows a triangular prism in which
1. The diagram on the right shows an awards stand. AB = 14 cm, CN = 6 cm and BE = 8 cm. Find the volume
(a) When viewed from the top, how would the object of the prism.
look like? Draw the shape. F
(b) When viewed from the front, how would the object
look like? Draw the shape.

top D E
C
8
6
N
A B
14

Solution
1
Area of ABC = × 14 × 6
2
front
= 42 cm2
Solution
Volume of the prism = 42 × 8
(a) = 336 cm3


4. The figure shows a swimming pool with a rectangular
(b) surface ABCD (not drawn to scale). It is 25 m long, 18
m wide, 3 m deep at the deep end and 2 m deep at the
shallow end. Find the volume of the swimming pool.
D C
18

A 25 B
2. The figure shows a prism whose end faces are H
G
2
parallelograms, where the unit of length is centimetre. 3
Draw a net of the prism.
F
E

2.5 Solution
Base area = Area of ABFE
1
2 = × (3 + 2) × 25
2
= 62.5 m2
1
Volume of the prism = 62.5 × 18
= 1125 m3
Solution

2.5 5. The surface area of a cube is 294 cm2. Find


(a) the length of a side of the cube,
1 1
70° 110° (b) the volume of the cube.
2.5 1 1
Solution
2 2
(a) Let the length of a side of the cube be x cm.
6x 2 = 294
2
x  = 49
1 2.5 1 x = 7
1 1 The length of a side of the cube is 7 cm.
2.5 (b) Volume of the cube = 73
= 343 cm3

163
6. A rectangular block of copper is 10 cm by 9 cm by (b) Perimeter of the base = Perimeter of ABC
4 cm. = 5 + 12 + 13
(a) Find the volume of the copper block. = 30 cm
(b) Find the total surface area of the copper block. Total surface area of the prism
(c) If the copper block is melted and recast into a cube, = 30 × 14 + 2 × 30
find the length of a side of the cube. Give your = 480 cm2
answer correct to 3 significant figures.

Solution 9. The figure shows a sauce dish shaped like a trapezoidal



(a) Volume of the block = 10 × 9 × 4 prism in which AB = 10 cm, BC = AD = 5 cm,
= 360 cm3 CD = 16 cm, BF = 18 cm and the depth of the dish is 4 cm.

(b) Total surface area of the block
= 2 × (10 × 9 + 9 × 4 + 10 × 4) H G
= 332 cm2 4
E F
(c) Let the length of a side of the cube be x cm.
x3 = 360 16 C 18
D
3 5 5
x = 360 A 10 B
= 7.11  (correct to 3 s.f.)
The length of a side of the cube is 7.11 cm.
(a) Find the volume of the dish.
(b) Find the external surface area of the dish.
7. A cuboid is 13 cm long and 6 cm high. Its volume is (c) If the material for the dish costs $0.02 per cm2,
624 cm3. Find find the cost of material used in making the dish.
(a) its breadth,
(b) its total surface area. Solution
(a) Base area = Area of ABCD
Solution
1
(a) Let the breadth of the cuboid be y cm. = × (10 + 16) × 4
2
13 × y × 6 = 624 = 52 cm2
y = 8 Volume of the dish = 52 × 18
The breadth of the cuboid is 8 cm. = 936 cm3
(b) Total surface area of the cuboid (b) DA + AB + BC = 5 + 10 + 5
= 2 × (13 × 8 + 8 × 6 + 13 × 6) = 20 cm
= 460 cm2 External surface area of the dish
= 20 × 18 + 2 × 52
8. The figure shows a triangular prism in which AB = 5 cm, = 464 cm2
BC = 13 cm, AC = 12 cm, BE = 14 cm and ∠BAC = 90°. Find (c) The required cost of material = $0.02 × 464
(a) the volume of the prism, = $9.28
(b) the total surface area of
the prism.
F
Section 15.3
10. The base radius of a solid cylinder is 2 cm and its height
is 7 cm. Find
C
(a) the volume of the cylinder,
D (b) the total surface area of the cylinder,
E
leaving your answers in terms of r.
12 13
14
Solution
A 5 B (a) Volume of the cylinder = r × 22 × 7 cm3
= 28r cm3
Solution (b) Total surface area of the cylinder
(a) Base area = Area of ABC = 2r × 2 × 7 + 2 × r × 22
1 = 36r cm2
= × 5 × 12
2
= 30 cm2
Volume of the prism = 30 × 14
= 420 cm3

Chapter 15  Volume and Surface Area of Solids


164
11. The volume of a cylindrical disc is 150 cm 3. If the Solution
thickness of the disc is 2 cm, find the base radius of the (a) Cross-sectional area of the pipe
disc. Give your answer correct to 3 significant figures. = r(22 – 1.62)
= 1.44r cm2
Solution Volume of material used in making the pipe
Let the base radius of the disc be r cm. = 1.44r × 5
rr 2 × 2 = 150 = 7.2r
75
= 22.6 cm3  (correct to 3 s.f.)
r =
r (b) External curved surface area = 2r × 2 × 5
= 4.89  (correct to 3 s.f.) = 20r cm2
The base radius of the disc is 4.89 cm. Internal curved surface area = 2r × 1.6 × 5
= 16r cm2
Total surface area of the pipe
12. A metal cylindrical bar of length 18 cm and base radius
= 20r + 16r + 2 × 1.44r
2 cm is melted and recast into a solid cylinder of base
= 38.88r
radius 3 cm. Find
= 122 cm2  (correct to 3 s.f.)
(a) the height of the new cylinder,
(b) the ratio of the total surface area of the bar to that (c) Volume of water delivered in 1 second
of the new cylinder. = r × 1.62 × 10
= 25.6r cm3
Solution Volume of water delivered in 30 seconds
(a) Let the height of the new cylinder be h cm. = 25.6r × 30
r × 32 × h = r × 22 × 18 = 768r
9h = 72 = 2410 cm3  (correct to 3 s.f.)
h = 8
The height of the new cylinder is 8 cm.
(b) Total surface area of the bar Section 15.4
= 2r × 2 × 18 + 2 × r × 22 14. The internal dimensions of a cargo container are 5.90 m
= 80r cm2 by 2.35 m by 2.39 m.
Total surface area of the new cylinder (a) Find its internal total surface area
= 2r × 3 × 8 + 2 × r × 32 (i) in m2, (ii) in cm2.
= 66r cm2 (b) Find its internal volume
The required ratio = 80r : 66r (i) in m3, (ii) in cm3.
= 40 : 33 Give your answers correct to 3 significant figures.

Solution
13. The external and internal diameters of a cylindrical water
pipe are 4 cm and 3.2 cm respectively and the length of (a) (i) Internal total surface area
the pipe is 5 cm. = 2(5.90 × 2.35 + 5.90 × 2.39
(a) Find the volume of material used in making the   + 2.35 × 2.39)
pipe. = 67.165
(b) Find the total surface area of the pipe. = 67.2 m2  (correct to 3 s.f.)
(c) If water flows through the pipe at 10 cm/s, find the (ii) Internal total surface area
volume of water delivered in 30 seconds. = 67.165 × 10 000 cm2
Give your answers correct to 3 significant figures. = 671 650 cm2
= 672 000 cm2  (correct to 3 s.f.)
4 cm (b) (i) Internal volume
= 5.90 × 2.35 × 2.39
= 33.137 35
3.2 cm = 33.1 m3  (correct to 3 s.f.)
5 cm (ii) Internal volume
= 33.137 35 × 1 000 000 cm3
= 33 137 350 cm3
= 33 100 000 cm3  (correct to 3 s.f.)

165
15. A cylindrical drum 25 cm high has a base radius of (b) Area of the cross-section
30 cm. = 13 × 18 +
1
× r × 132
(a) Find its curved surface area 4
(i) in cm2, = 366.73
(ii) in m2. = 367 cm2  (correct to 3 s.f.)
(b) Find its volume
(c) Total surface area of the bread
(i) in cm3,
= 82.42 × 40 + 2 × 366.73
(ii) in m3.
= 4030 cm2  (correct to 3 s.f.)
Give your answers correct to 3 significant figures.
(d) Volume of the bread
Solution = 366.73 × 40
(a) (i) Its curved surface area = 14 700 cm3  (correct to 3 s.f.)
= 2r × 30 × 25
= 4712.39
17. The figure shows a prism whose cross-section is formed
= 4710 cm2  (correct to 3 s.f.)
by cutting out CDF from the parallelogram ABFE.
(ii) Its curved surface area AB = 25 cm, DE = 15 cm and BH = 30 cm. The heights
1 from C to EF and AB are 8 cm and 10 cm respectively.
= 4712.39 × m2
10 000 Find the volume of the prism.
= 0.471 239 m2
K J L
= 0.471 m2  (correct to 3 s.f.)
8
(b) (i) Volume of the drum I
= r × 302 × 25 10
= 70 685.8 D F H
E
15
= 70 700 cm3  (correct to 3 s.f.)
C 30
(ii) Volume of the drum
1
= 70 685.8 × m3 A B
1000 000 25
3
= 0.070 7 m   (correct to 3 s.f.)
Solution
Area of the cross-section of the prism
16. The figure shows a loaf of bread with a uniform cross- 1
section. ABDE is a rectangle and BCD is a quarter of = 25 × 18 – × 10 × 8
2
a circle with centre B. It is given that AB = 18 cm, = 410 cm 2

AE = 13 cm and
CH = 40 cm. Find Volume of the prism
(a) the perimeter of the cross-section, = 410 × 30
(b) the area of the cross-section, = 12 300 cm3
(c) the total surface area of the bread,
(d) the volume of the bread.
Give your answers correct to 3 significant figures.
J I

H
F G

D 40
E
13
A C
18 B

Solution
(a) Perimeter of the cross-section
1
= 13 + 18 × 2 + 13 + × 2r × 13
4
= 82.42
= 82.4 cm  (correct to 3 s.f.)

Chapter 15  Volume and Surface Area of Solids


166
Exercise 15.1 3. Draw the views from the front and from the left-hand
Level 1 side of each of the following shapes.
(a) (b)
1. Draw a sketch of each of the following shapes.
(a) (b)

Solution
(a) (b) (a)

Front view Left view

Solution (b)
(a) (b)

Front view Left view

(a) (b)
4. Draw the views from the top and from the right-hand
side of each of the following shapes.
(a) (b)

2. Draw the views from the front and from the top of each
of the following shapes. Solution
(a) (b) (a)

Top view Right view


(b)
Solution
(a)

Top view Right view


Front view Top view

(b) 5. Determine whether each of the following is a net of a cube.
(a) (b)

Front view Top view

Solution
(a) It is not the net of cube.
(b) It is a net of a cube.

167
6. Draw a sketch of the shape that can be formed by each (a) (b)
of the following nets.
(a)

Solution
(a) Yes, the figure is the top view of the solid.
(b) Yes, the figure is the right-hand side view of the
solid.

9. Sketch the views from the front and from the top of the
solid below.

Solution
(a)

Solution

(b)

Top view Front view

10. Draw a net of each of the following solids, where the


unit of length is centimetre.
Level 2 (a) (b)
7. Determine whether the given figure can be a view of
(a) a cube,
1.5 1.5
(b) a cuboid,
(c) a cylinder. 1 1 5
1 2
2

Solution
Solution
(a) Yes, the figure can be the top view of a cube.
(a)
(b) Yes, the figure can be the side view of a cuboid.
1.5 1.5
(c) Yes, the figure can be the side view of a cylinder.

1 1
8. Determine whether the given figure can be a view of the 1 2 1
following solids.
1 1 1 1
2
1 1
1 1

1 1
1.5 1.5

1.5 1.5
1 1

Chapter 15  Volume and Surface Area of Solids


168
(b) 13. The figure is an incomplete net of a candy box. Complete
the net and sketch the box.
1

2 r

5 5 5

Solution

2 r

Level 3
11. Sketch two different views of the given bottle.

Solution

xercise 15.2
E
Level 1
Give your answers correct to 3 significant figures where
applicable.
1. (a) Draw a net of a cube whose sides are 3 cm.
Top view Front view
(b) Find the volume and total surface area of a cube
whose sides are
(i) 3 cm,
12. The figures are two different views of a solid. Sketch (ii) 11 cm.
the shape of a possible solid.
Solution
(a) Diagram is not drawn to exact scale.


Solution

169
(b) (i) Volume of the cube = 33 Solution
= 27 cm3 (a)
Total surface area of the cube = 6 × 32
3.5
= 54 cm2
(ii) Volume of the cube = 113 5 3.5
= 1331 cm3
Total surface area of the cube = 6 × 112
= 726 cm2
5 4 4
2. A cuboid has dimensions 4 cm by 3 cm by 2 cm.
(a) Draw a net of the cuboid. 3.5
(b) Find the volume of the cuboid. 3 3.5
(c) Find the total surface area of the cuboid.

Solution
(a) 3.5 3.5

5 4

(b) Volume of the cuboid = 4 × 3 × 2 (b) Volume


= Base area × Height
= 24 cm3 = Area of DEF × FC
1
(c) Surface area of the cuboid = × 3 × 4 × 3.5
2
= 2(4 × 2 + 4 × 3 + 3 × 2)
= 21 cm3
= 52 cm2
(c) Surface area
= 2 × Area of DEF + Area of DEAB +
3. The diagram shows a triangular prism, DE = 3 cm,
Area of EFCB + Area of DFCA
EF = 4 cm, FD = 5 cm and CF = 3.5 cm. 1
(a) Draw a net of the prism. = 2 × × 3 × 4 + 3 × 3.5 + 4 × 3.5 + 5 × 3.5
2
(b) Find the volume of the prism. = 54 cm2
(c) Find the total surface area of the prism.
4. Find the volume and total surface area of each prism,
F where the unit of length is centimetre.
5
D (a) (b)
E 4 3.5 D
3 H G
E
C 13
A E F
E F
5 10
B A
25
D C
12 13
2
A 3 B
B N C

(c) H G

E F

6 C 10
D
4 5

A B
9

Chapter 15  Volume and Surface Area of Solids
170
Solution 7. Copy and complete the following table for cuboids.
(a) Volume of the prism = 3 × 2 × 5
= 30 cm3 Total surface
Length Breadth Height Volume
area
Total surface area of the prism
= 2 × (3 × 2 + 3 × 5 + 2 × 5) (a) 9 cm 6 cm 270 cm3
= 62 cm2 (b) 8 cm 5 cm 440 cm3
1 (c) 14 cm 6 cm 608 cm2
(b) Area of ABC = × 10 × 12
2
= 60 cm2 Solution
Volume of the prism = 60 × 25
= 1500 cm3 Total surface
Length Breadth Height Volume
area
Total surface area of the prism
= (13 + 13 + 10) × 25 + 2 × 60 (a) 9 cm 6 cm 5 cm 270 cm3 258 cm2

= 1020 cm2 (b) 11 cm 8 cm 5 cm 440 cm3


366 cm2

1 (c) 14 cm 11 cm 6 cm 924 cm3 608 cm2


(c) Area of ABCD = × (6 + 9) × 4
2
270
= 30 cm2 (a) Height
=
9×6
Volume of the prism = 30 × 10
= 5 cm
= 300 cm3
Total surface area
Total surface area of the prism
= 2 × (9 × 6 + 9 × 5 + 6 × 5)
= (4 + 6 + 5 + 9) × 10 + 2 × 30
= 258 cm2
= 300 cm2
440
(b) Length
=
8×5
5. The figure shows a pentagonal prism I = 11 cm
in which the area of ABCDE is Total surface area
150 cm2, AB = 10 cm, BC = 6 cm, J H
= 2 × (11 × 8 + 11 × 5 + 8 × 5)
CD = 11 cm, DE = 13 cm, EA = 8 cm F G = 366 cm2
and CH = 21 cm. Find
21 (c) Let the breadth be y cm.
(a) the volume of the prism,
D 2 × (14y + 14 × 6 + 6y) = 608
(b) the total surface area of the 13 11
20y + 84 = 304
prism. E C
6
20y = 220
8
A
y = 11
10 B
Volume = 14 × 11 × 6
Solution
= 924 cm3
(a) Volume of the prism = 150 × 21
= 3150 cm3
8. A rectangular copper block A measuring 10 cm by 6 cm
(b) Total surface area of the prism
by 5 cm, is melted and recast into a rectangular bar B,
= (10 + 6 + 11 + 13 + 8) × 21 + 2 × 150
3 cm wide and 2 cm high. Find
= 1308 cm2
(a) the length of copper bar B.
(b) the ratio of the total surface area of block A to that
of bar B.
Level 2
6. The total surface area of a cube is 486 cm2. Find
Solution
(a) the length of a side of the cube,
(a) Let the length of the copper bar B be y cm.
(b) the volume of the cube.
3 × 2 × y = 10 × 6 × 5
y = 50
Solution
The length of copper bar B is 50 cm.
(a) Let the length of a side of the cube be x cm.
6x2 = 486 (b) Total surface area of block A
 x2 = 81 = 2 × (10 × 6 + 10 × 5 + 6 × 5)
 x = 9 = 280 cm2
The length of a side of the cube is 9 cm. Total surface area of bar B
= 2 × (50 × 3 + 50 × 2 + 3 × 2)
(b) Volume of the cube = 93
= 512 cm2
= 729 cm3
The required ratio = 280 : 512
= 35 : 64

171
9. Find the volume and total surface area of each prism, (c) Let the drop in water level be h cm.
where the unit of length is centimetre. 50 × 30 × h = 6000
(a) (b) 6000
h =
1500
I H
L K
D 8 =4
C The drop in water level is 4 cm.
J 12 I
6
F
G 5
J
E H G
B 11. A rectangular tin plate measures 40 cm by 30 cm. A
E 5
15
C
25 small square of side 8 cm is cut out from each corner as
3
D 5 F shown in the figure. The plate is then folded along the
A 20 B 7
A dotted lines to form an open tray. Find
(a) the external surface area of the tray,
Solution
(b) the volume of the tray.
(a) Area of the cross-section ABCDEF
= 20 × 5 + 10 × 8 8
= 180 cm2 8

Volume of the prism = 180 × 25


30
= 4500 cm3
Surface area of the prism
= (20 + 5 + 12 + 10 + 8 + 15) × 25 + 2 × 180
= 2110 cm2 40

(b) Area of the cross-section ABCDE


1 Solution
= ×8×3+6×8 (a) External surface area of the tray
2
= 40 × 30 – 4 × 82
= 60 cm2
= 944 cm2
Volume of the prism = 60 × 7
= 420 cm3 (b) Length of the tray = 40 – 8 – 8
Surface area of the prism = 24 cm
= (5 + 5 + 6 + 8 + 6) × 7 + 2 × 60] Breadth of the tray = 30 – 8 – 8
= 330 cm2 = 14 cm
Volume of the tray = 24 × 14 × 8
= 2688 cm3
10. A rectangular aquarium tank is 50 cm long, 30 cm wide
and 36 cm high.
12. The external dimensions of a wooden box are 15 cm by
10 cm by 8 cm. The thickness of the wood on each side
is 1.5 cm. Find
36
(a) the internal dimensions of the box.
(b) the volume of wood used to make the box.
30
Solution
50
(a) Internal length = 15 – 2 × 1.5
(a) Find the volume of water required to fill up the = 12 cm
empty tank to its brim. Internal breadth = 10 – 2 × 1.5
(b) The tank is open at the top. Find the external surface = 7 cm
area of glass required to make the tank. Internal height = 8 – 2 × 1.5
(c) If 6000 cm3 of water is drained from the tank, find = 5 cm
the drop in water level. (b) External volume of the box = 15 × 10 × 8
= 1200 cm3
Solution Internal volume of the box = 12 × 7 × 5
(a) The required volume of water = 420 cm3
= 50 × 30 × 36 Volume of wood used = 1200 – 420
= 54 000 cm3 = 780 cm3
(b) External surface area of glass
= 50 × 30 + 2 × 30 × 36 + 2 × 50 × 36
= 7260 cm2

Chapter 15  Volume and Surface Area of Solids


172
13. The figure shows a rectangular living room which is Space inside the tent = 1.2 × 2.1
5 m long, 4 m wide and 3 m high. It has a rectangular = 2.52 m3
door of width 1 m and height 2 m, and two rectangular (b) Surface area of the tent
windows, each 1 m by 1.5 m. = (1.7 + 1.7 + 1.6) × 2.1 + 2 × 1.2
= 12.9 m2
1.5
(c) Total cost of material = $30 × 12.9
3 = $387
1 1

2
15. The figure shows a swimming pool with a rectangular
4 surface ABCD (not drawn to scale). CD = 20 m,
5 AE = 1.6 m, BF = 1 m and the area of ABCD is
240 m2. Find
(a) Find the volume of the space inside the room.
(a) the length of AD,
(b) Find the total surface area of the walls of the room.
(b) the volume of the pool.
(c) The owner is going to put wallpaper onto the walls.
Each roll of wallpaper is 10 m long and 0.53 m wide. D 20 C
How many rolls of wallpaper should be ordered?

Solution G
A
(a) Volume of space = 5 × 4 × 3 H
B
1
= 60 m3
1.6
F
(b) Surface area of walls E
= 2(5 × 3 + 4 × 3) – 1 × 2 – 2(1 × 1.5)
= 49 m2
Solution
(c) Area covered by each roll = 10 × 0.53 240
(a) Length of AD =
20
= 5.3 m2
49 = 12 m
Number of rolls = (b) Area of the cross-section ABFE
5.3
1
= 9.245 = × (1.6 + 1) × 20
2
10 rolls should be ordered. = 26 m2
Volume of the pool = 26 × 12
= 312 m3
Level 3
14. The figure shows a camping tent. The tent is 16. The figure shows a tray whose end faces are trapeziums.
closed at both ends and at the bottom. Given that If AB = 15 cm, BC = AD = 12.5 cm, CD = 22 cm,
AB = AC = 1.7 m, BC = 1.6 m, AN = 1.5 m and BF = 45 cm and AN = 12 cm, find
CF = 2.1 m, find (a) the area of ABCD,
(a) the space inside the tent, (b) the volume of the tray,
(b) the surface area of the tent, (c) the external surface area of the tray.
(c) the total cost of the material used in making the
H 22 G
tent if the material of the tent costs $30 per m2.
12.5
D E F

N C 45
D
12.5 12
A
A 15 B
E F
1.7 Solution
1.5 1.7
2.1 1
(a) Area of ABCD = × (15 + 22) × 12
N
2
B 1.6 C = 222 cm2
(b) Volume of the tray = 222 × 45
Solution = 9990 cm3
1
(a) Area of ABC = × 1.6 × 1.5 (c) External surface area of the tray
2
= 1.2 m2 = (12.5 + 15 + 12.5) × 45 + 2 × 222
= 2244 cm2

173
17. The diagram shows a piece of chocolate which is in 19. A manager wants to design a rectangular box of capacity
the form of a prism with a parallelogram base ABCD. 900 cm3. The box should have a square base of side a cm
AB = 4 cm, AD = 3 cm, DN = 2.5 cm and the volume and a height of h cm, where a and h are integers greater
of the piece is 15 cm3. Find than 1.
(a) the area of ABCD,
(b) the length of BF,
(c) the total surface area of the piece.
H G h
D
C

3 2.5
a
E F a
A B
N
4
(a) Find two possible sets of dimensions for the box.
Solution
(b) Suggest the dimensions for a design that uses the
(a) Area of ABCD = 4 × 2.5
least material to make the box.
= 10 cm2
(b) Volume = 15
10 × BF = 15 Solution
BF = 1.5 cm (a) a2h = 900
When a = 5, 52 × h = 900
(c) Total surface area = (3 + 4 + 3 + 4) × 1.5 + 2 × 10
h = 36
= 41 cm2
When a = 10, 102 × h = 900
h = 9
18. The diagram shows a marble chess box which is Two possible sets of dimensions are:
in the form of a hexagonal prism. The length of 5 cm × 5 cm × 36 cm, and
each side of the hexagon is x cm and the height of 10 cm × 10 cm × 9 cm.
the box is 2.5 cm. It is known that the base area of the
3
3 (b) Total surface area of the box = 2a2 + 4ah
box is x2 cm2 and the volume of the box is 375 3 cm3.
2 900
Find = 2a2 + 4a ×
a2
(a) the value of x, = 2a2 +
3600
(b) the total surface area of the box. a
Setting up a table or using a spreadsheet program,
we have

2.5
a Total surface area
5 770
6 672
x
7 612.3
Solution 8 578
(a) Base Area × Height = Volume
9 562
3
3 10 560
x2 × 2.5 = 375 3
2
2 11 569.3
x2 = 375 3 × 3 12 588
7.5
x2 = 100 13 614.9
x = 100
Hence the material used is minimum when
x = 10 a = 10. The optimal dimensions are
10 cm × 10 cm × 9 cm.
3
3
(b) Total surface area = 6x × 2.5 + 2 × x2
2
3
= 6 × 10 × 2.5 + 3 × 102

= 670 cm2  (correct to 3 s.f.)

Chapter 15  Volume and Surface Area of Solids


174
Exercise 15.3 3. A cylindrical metal bar is 11 cm long and has a base
Level 1 radius of 1 cm. Find its volume and total surface area.
In this exercise, give your answers in terms of r or correct to Solution
3 significant figures where applicable. Volume of the sausage = r × 12 × 11
1. A cylinder has a base radius of 2 cm and a height of 3 cm. = 11r cm3
(a) Draw a net of the cylinder. Total surface area of the sausage
(b) Find the volume of the cylinder. = 2r × 1 × 11 + 2 × r × 12
(c) Find the total surface area of the cylinder. = 24r cm2
Solution
(a) Diagram is not drawn to exact scale. 4. A metal cylindrical disc is 3 cm thick and its diameter
is 14 cm. Find its volume and total surface area.

1 Solution
Volume of the disc = r × 72 × 3
= 147r cm3

6.28 Total surface area of the disc


= 2r × 7 × 3 + 2 × r × 72
1.5
= 140r cm2

5. A cylindrical glass has an external base radius of 4 cm


and a height of 9 cm. Find
(a) its volume,
1
(b) its external surface area.

Hint: A glass is open at the top.
(b) Volume
= Base area × height
= 2r × 2 × 3
Solution
= 12r
(a) Volume of the glass = r × 42 × 9
= 37.7 cm3
= 144r cm3
(c) Total surface area = r(2)2 × 2 + 2r × 2 × 3
(b) External surface area of the glass
= 20r
= 2r × 4 × 9 + r × 42
= 62.8 cm2
= 88r cm2

2. Find the volume and the total surface area of each cylinder
below, where the unit of measurement is in centimetre. Level 2
(a) (b) 6. Find the height of a cylinder if its
(a) volume = 63r cm3, base radius = 3 cm,
(b) volume = 100 cm3, base radius = 2 cm.

13 18
Solution
(a) Let the height of the cylinder be h cm.
6 30 r × 32 × h = 63r
h = 7
The height of the cylinder is 7 cm.
Solution
(b) Let H cm be the height of the cylinder.
(a) Volume of the cylinder = r × 62 × 13
r × 22 × H = 100
= 468r cm3
25
Total surface area of the cylinder H =
r
= 2r × 6 × 13 + 2 × r × 62
= 7.96  (correct to 3 s.f.)
= 228r cm2
The height of the cylinder is 7.96 cm.
(b) Volume of the cylinder
= r × 92 × 30
= 2430r cm3
Total surface area of the cylinder
= 2r × 9 × 30 + 2 × r × 92
= 702r cm2

175
7. Find the base radius of a cylinder if its Solution
(a) volume = 150r cm3, height = 6 cm, Let the depth of water in the jar be h cm.
(b) volume = 400 cm3, height = 8 cm. r × 52 × h = 15 × 10 × 4
24
h =
Solution r
(a) Let the base radius of the cylinder be r cm. = 7.64  (correct to 3 s.f.)
r × r2 × 6 = 150r The depth of water in the jar is 7.64 cm.
r 2 = 25
r = 5
11. The figure shows a solid half cylinder of base diameter
The base radius of the cylinder is 5 cm.
2 cm and height 2.5 cm.
(b) Let the base radius of the cylinder be R cm. (a) Find its volume. D
r × R2 × 8 = 400 (b) Draw a net of the solid. D
C
50 (c) Find its total surface area.
R = 2.5
r
= 3.99  (correct to 3 s.f.)
A 2 B
The base radius of the cylinder is 3.99 cm.

Solution
8. Find the circumference of a solid cylinder if its 1
(a) Volume of the solid = × r × 12 × 2.5
(a) curved surface area = 660 cm2, height = 10 cm, 2
(b) curved surface area = 1200 cm2, height = 15 cm. = 1.25r cm3
(b) The net of the solid is shown below.
Solution
(a) Circumference × height = curved surface area
660
Circumference of the cylinder =
10 D C D'
= 66 cm
1200
(b) Circumference of the cylinder = 2.5
15
= 80 cm
A 2 B A'
3.14
9. A metal cylinder of base radius 6 cm and height 5 cm
is melted and recast into a cylindrical metal bar of base

radius 2 cm. Taking r to be 3.124, find
(a) the length of the bar formed, (c) Total surface area of the solid
(b) the ratio of the total surface area of the cylinder to = (2 + r) × 2.5 + r × 12
that of the bar. = 16.0 cm2  (correct to 3 s.f.)

Solution
(a) Let y cm be the length of the bar formed. Level 3
r × 22 × y = r × 62 × 62 12. A cylindrical can of curry chicken 3.5
4y = 180 has a base radius of 3.5 cm and a
The length of the bar is 45 cm. height of 10 cm. Find
(b)
Total surface area of the original cylinder (a) the volume of the can,
(b) the area of the wrapper on the 10
= 2r × 6 × 5 + 2 × r × 62
= 132r cm2 curved surface of the can.
Total surface area of the bar
= 2r × 2 × 45 + 2 × r × 22
= 188r cm2
The required ratio = 132r : 188r Solution
= 33 : 47 (a) Volume of can = r × (3.5)2 × 10
= 385 cm3  (correct to 3 s.f.)
10. A rectangular tray of dimensions 15 cm by 10 cm by (b) Area pressed by roller after 3 revolutions
4 cm is full of water. The water is poured into an empty = 2 × r × (3.5) × 10
cylindrical jar of internal radius 5 cm. Taking r to be = 220 cm2  (correct to 3 s.f.)
3.124, find the depth of water in the jar.

Chapter 15  Volume and Surface Area of Solids


176
13. A dough roller has a cylindrical body of diameter 5 cm (c) Increase in total surface area
and length 30 cm. Find the area pressed by the roller if 6
= 2 × r × 1.12 × 2.4 ×
it rotates 3 revolutions. 10
= 10.1 cm2  (correct to 3 s.f.)

16. A cylindrical bucket of base radius 10 cm and height 18


cm is completely filled with water each time before the
Solution
water is poured into a rectangular tank. The tank is 60
(a) Area pressed by roller after 3 revolutions
cm long, 45 cm wide and 36 cm high. Taking r to be
= 3 × 2 × r × 2.5 × 30
3.142, find
= 1410 cm2  (correct to 3 s.f.)
(a) the volume of the bucket,
(b) the volume of the tank,
14. A measuring cylinder of internal diameter 5 cm is partially (c) the rise in the water level of the tank when 7 full
filled with water. When a stone is placed in the cylinder buckets of water are poured into the tank,
as shown, the water level rises by 3 cm. (d) the number of full buckets of water required to fill
the tank completely.

Solution
(a) Volume of the bucket = r × 102 × 18
3 = 1800r cm3
(b) Volume of the tank = 60 × 45 × 36
= 97 200 cm3
5 5 (c) Let the rise in the water level be h cm.
Find 60 × 45 × h = 1800r × 7
(a) the volume of the stone, 14r
h =
(b) the increase in the contact area between the water 3
and the measuring cylinder. = 14.7  (correct to 3 s.f.)
The rise in the water level is 14.7 cm.
Solution
(d) Let n be the number of full buckets required.
(a) Volume of the stone
1800r × n  97 200
= r × 2.52 × 3
97 200
= 18.75r cm3 n 
1800r
(b) Increase in water contact area n  17.19
= 2r × 2.5 × 3 The required number of full buckets is 18.
= 15r cm2
17. A piece of cylindrical pipe is 8 cm long. Its internal base
15. A stack of 10 $1 coins forms a cylinder of base diameter diameter is 4 cm and its wall is 0.5 cm thick.
2.24 cm and height 2.4 cm. (a) Find the external base radius of the pipe.
(a) Find the volume of the stack. (b) Find the volume of material used in making the
(b) Find the total surface area of the stack. pipe.
(c) If six more $1 coins are added to the stack, find (c) Find the total surface area of the pipe.
the increase in the total surface area of the stack. (d) Water flows through the pipe at a rate of 15 cm/s.
(Take r to be 3.142) Find the volume of water delivered in 20 seconds.

Solution
(a) External base radius of the pipe
2.4
= 4 ÷ 2 + 0.5
= 2.5 cm
(b) Cross-sectional area of the pipe
2.24
= r × 2.52 – r × 22
Solution
= 2.25r cm2
(a) Volume of stack = r × (1.12)2 × 2.4
Volume of material used in making the pipe
= 9.46 cm3  (correct to 3 s.f.)
= 2.25r × 8
(b) Total surface area = 18r cm3
= 2 × r × 1.12 × 2.4 + 2 × r × (1.12)2
= 24.8 cm2  (correct to 3 s.f.)

177
(c) Internal lateral surface area of the pipe A sheet of rectangular paper measures 20 cm by 15 cm. It
= 2r × 2 × 8 can be rolled up along either side to form two cylinders,
= 32r cm2 A and B as shown.
External lateral surface area of the pipe (a) Which cylinder has a greater volume?
= 2r × 2.5 × 8 cm2 (b) Suppose the sheet is divided into two equal parts,
= 40r cm2 X and Y, as shown and then rolled up to form two
Total surface area of the pipe cylinders of height 15 cm each. Would the sum of
= 32r + 40r + 2 × 2.25r the volumes of these two small cylinders be greater
= 76.5r cm2 than the volume of cylinder B? Explain how you
arrived at your conclusion.
(d) Volume of water delivered in 1 second
= r × 22 × 15 Solution
= 60r cm3 (a) Let a cm and b cm be the base radii of cylinder A
Volume of water delivered in 20 seconds and cylinder B respectively.
= 60r × 20 2ra = 15
= 1200r cm3 15
a =
2r 2
15
18. A cylindrical cake has a base radius of 12 cm and height Volume of cylinder A = r ×   × 20
2r
8 cm. It is cut into two equal halves as shown. Taking r 1125
= cm3
to be 3.142, find r
(a) the volume of each half, 2rb = 20
10
(b) the total surface area of each half, b =
r
(c) the percentage increase in the total surface area of 2
the cake due to the cutting. Volume of cylinder B = r ×  
10
× 15
r
1500
= cm3
12
r
Cylinder B has a greater volume.

(b) Let c cm be the base radius of the cylinder formed


8
by sheet X.
Then 2rc = 10
5
Solution c =
r
1
(a) Volume of each half = × r × 122 × 8 Sum of the volumes of two small cylinders
2 2
= 1810 cm3  (correct to 3 s.f.) 5
=2×r× × 15
r
(b) Total surface area of each half 750
= 24 × 8 + r(12)2 + r × 12 × 8 = cm3
r
= 946 cm2  (correct to 3 s.f.) The sum of the volumes of these two small cylinders
is smaller than the volume of cylinder B.
(c) Perecntage increase
946 × 2
= × 100% – 100%
2 × r × (12)2 + 2 × r × 12 × 8
Exercise 15.4
= 25.5%
Level 1
19. Give your answers correct to 3 significant figures where
applicable.
X
1. Express the following areas in cm2.
20 A 20 or B 15 (a) 3 m2
(b) 13.6 m2
Y

Solution

15 (a) 3 m2 = 3 × 10 000 cm2
= 30 000 cm2
(b) 13.6 m2 = 13.6 × 10 000 cm2
= 136 000 cm2

Chapter 15  Volume and Surface Area of Solids


178
2. Express the following areas in m2. Total surface area
(a) 4000 cm2 = 66 × 6 + 2 × 150
(b) 25 600 cm2 = 696 cm2
(b) Area of cross-section
Solution
= Area of A + Area of B
4000
(a) 4000 cm2 = m2 1
10 000 = 10 × 15 + × (20 + 28) × 15
2
= 0.4 m2 = 510 cm 2

25 600 Perimeter of cross-section


(b) 25 600 cm2 = m2
10 000 = 10 + 15 + 18 + 17 + 20 + 30
= 2.56 m2 = 110 cm
Volume
= 510 × 14
3. Express the following volumes in cm3. = 7140 cm3
(a) 2 m3 (b) 39.7 m3 Total surface area
= 110 × 14 + 2 × 510
Solution
= 2560 cm2
(a) 2 m3 = 2 × 1 000 000 cm3
= 2 000 000 cm3
6. Find the total surface area and volume of each prism,
(b) 39.7 m3 = 39.7 × 1 000 000 cm3
where the unit of length is centimetre.
= 39 700 000 cm3
(a) (b)

4. Express the following volumes in m3.


(a) 63 000 cm3 (b) 9 280 000 cm3 3
5 A B C
2
6 A B 4
Solution 3 10 2 2
5
3 63 000 3 7 2 2
(a) 63 000 cm = m
1 000 000
= 0.063 m3 Solution
(a) Area of cross-section
9 280 000
(b) 9 280 000 cm3 = m3 = Area of A + Area of B
1 000 000
1
= 9.28 m3 =3×6+ × (6 + 3) × 4
2
2
= 36 cm
5. Find the total surface area and volume of each prism Perimeter of cross-section
below, where the unit of length is centimetre. =3+5+3+7+6
(a) (b) = 24 cm
Volume
10 = 36 × 10
13 15
A
A = 360 cm3
18
20 30 Total surface area
10
B 17 = 24 × 10 + 2 × 36
B 8
14 = 312 cm2
6 20
15
(b) Area of cross-section
Solution = Area of A + Area of B + Area of C
=4×2+2×2+2×4
(a) Area of cross-section = 20 cm2
= Area of A + Area of B Perimeter of cross-section
1 =6+4+2+2+2+2+2+4
= × 12 × 5 + 15 × 8
2 = 24 cm
= 150 cm2 Volume
Perimeter of cross-section = 20 × 5
= 13 + 10 + 8 + 15 + 20 = 100 cm3
= 66 cm Total surface area
Volume = 24 × 5 + 2 × 20
= 150 × 6 = 160 cm2
= 900 cm3

179
Level 2 Solution
7. A playground is built on a rectangular plot of land 35 m (b) Area of the cross-section ABCDEFGH
long and 24 m wide. Find the area of the land 1
= × (4 + 16) × 8 – 4 × 4
2
(a) in m2, (b) in cm2.
= 64 cm2
Solution Volume of the prism = 64 × 20
(a) Area of the playground = 35 × 24 = 1280 cm3
= 840 m2 Total surface area of the prism
(b) Area of the playground = 840 × 10 000 cm2 = (10 + 4 + 10 + 6 + 4 + 4 + 4 + 6) × 20
= 8 400 000 cm2 + 2 × 64
= 1088 cm2

8. A person drinks 8 cups of water a day. If each cup


11. In the figure, AB = 10 cm, BC = 7 cm and AE = 12 cm. A
contains 250 cm3 of water, find the total amount of water
hole of radius 2 cm is drilled through the cuboid. Taking
the patient drinks per day in
r to be 3.142, find
(a) cm3, (b) m3.
(a) the volume of the solid,
Solution (b) the total surface area of the solid.
(a) Amount of water = 250 × 8 cm3 H G
= 2000 cm3
2000
(b) Amount of water = m3 E F
1 000 000
3
= 0.002 m
12 D C

9. Find the total surface area and volume of the each prism, 7

where the unit of length is centimetre. A 10 B

H
Solution
D (a) Volume of the solid
17 17
8 = 10 × 7 × 12 – r × 22 × 12
E C G = 840 – 48r
12 12 = 689 cm3  (correct to 3 s.f.)
27
A 30 B (b) Total surface area of the solid
Solution = 2 × (10 × 7 + 10 × 12 + 12 × 7) – 2 × r × 22
1
Volume = × 30 × 8 × 27 + 12 × 30 × 27 + 2r × 2 × 12
2 = 548 + 40r
= 12 960 cm3 = 674 cm2  (correct to 3 s.f.)
Total surface area
1 12. The figure shows a solid formed by stacking two cubes
= × 30 × 8 × 2 + 12 × 30 × 2 + 12 × 27 × 2 + 17 ×
2 of sides 3 cm and 5 cm respectively. Find
27 × 2 + 30 × 27 (a) the volume of the solid,
= 3336 cm2 (b) the total surface area of the solid.

10. Find the total surface area and volume of the prism,
where the unit of length is centimetre. 3
I P M L

A 6 6
H E D 5
4 4
K
10 G 4 F 10
4 20

B C Solution
(a) Volume = 33 + 53
= 152 cm3

(b) Total surface area = 5 × 32 + 5 × 52 + 52 – 32
= 186 cm2

Chapter 15  Volume and Surface Area of Solids


180
13. The figure shows a solid formed by two cylinders. The Level 3
base radius of the upper cylinder is 3 cm and its height 15. A cylindrical petrol tank has a base radius of 0.8 m and
is 9 cm. The base radius of the lower cylinder is 9 cm a height of 2.0 m. The petrol in it is used to fill up small
and its height is 6 cm. Taking r to be 3.142, find cylindrical cans of base radius 10 cm and height 25 cm
(a) the volume of the solid, each.
(b) the total surface area of the solid. (a) Find the volume of the tank
3 (i) in m3, (ii) in cm3.
(b) Find the volume of a can
(i) in cm3, (ii) in m3.
9
(c) If the tank is full of petrol, how many cans can it
fill?
(Take r to be 3.142)
6

Solution
9 (a) (i) Volume of the tank
Solution = r × 0.82 × 2
(a) Volume of the solid = 1.28r
= r × 32 × 9 + r × 92 × 6 = 4.02 m3  (correct to 3 s.f.)
= 567r (ii) Volume of the tank
= 1780 cm3  (correct to 3 s.f.) = 4.02 × 1 000 000 cm3
(b) Total surface area of the solid = 4 020 000 cm3  (correct to 3 s.f.)
= 2r × 3 × 9 + 2r × 9 × 6 + 2 × r × 92 (b) (i) Volume of a can
= 324r = r × 102 × 25
= 1020 cm2  (correct to 3 s.f.) = 2500r
= 7850 cm3  (correct to 3 s.f.)
14. F 40 E (ii) Volume of a can
G 7850
13
= m3
12 12 13 1 000 000
= 0.007 850 m3
A D
M B C N
20 (c) Let n be the number of cans that can be filled with
5 5
petrol.
The figure shows the uniform cross-section of a 2500r × n  1.28r × 1 000 000
solid. ADEF is a trapezium and BGC is a semicircle. n  512
AM = DN = 5 cm, AF = DE = 13 cm, EN = FM = 12 cm, It can fill 512 cans with petrol.
BC = 20 cm, EF = 40 cm and the thickness of the solid
is 8 cm. Taking r to be 3.142, find 16. The figure shows a bolt which has a hexagonal head
(a) the volume of the solid, and a cylindrical body. The hexagonal head is a prism
(b) the total surface area of the solid. 3
3
of base area cm2 and height 0.5 cm.
2
Solution The length of each side of the hexagon is 1 cm. The
(a) AD = 5 + 40 + 5 cylindrical body has a diameter of 1 cm and a length of
= 50 cm 4 cm. Taking r to be 3.142, find
Area of the cross-section (a) the volume of the bolt,
1 1 (b) the total surface area of the bolt.
= × (40 + 50) × 12 – × r × 102
2 2
= (540 – 50r) cm2 0.5
Volume of the solid
= (540 – 50r) × 8
= 3060 cm3  (correct to 3 s.f.) 1
(b) Perimeter of the cross-section 1
= 40 + 13 + 30 + 13 + r × 10
= 96 + 10 r 4
Total surface area of the solid
= (96 + 10r) × 8 + 2 × (540 – 50r)
= 1848 – 20r
= 1790 cm2  (correct to 3 s.f.)

181
Solution (d) Volume of the sofa
(a) Volume of the bolt = 3218.12 × 180 cm3
3
3 1 2 = 579 261.6 cm3
= × 0.5 + ×r×4 579 261.6
2 2 = m3
3 1 000 000
= 4.44 cm (correct to 3 s.f.)
= 0.579 m3  (correct to 3 s.f.)
(b) Total surface are of the bolt
3
1 2 3 1 2
= 2 × r × 0.5 × 4 + r × + ×2–r× 18. In the figure, 6 cylindrical cans of soft drink are packed
2 2 2
+ 6 × 0.5 × 1 tightly in a rectangular carton which is open at the top.
= 20.8 cm2 (correct to 3 s.f.) Each can is of diameter 6.4 cm and height 12 cm. Taking
r to be 3.142, find
(a) the external surface area of the carton,
17. The figure shows the uniform cross-section of a (b) the total volume of the cans,
sofa. EFG is a semicircle, AB = 72 cm, BC = 20 cm, (c) the percentage of the volume of the carton which
CD = 40 cm, AG = 63 cm and the length of the sofa is is empty space.
1.8 m. Taking r to be 3.142, find
(a) the perimeter of the cross-section in centimetre,
(b) the area of the cross-section in cm2,
(c) the total surface area of the sofa in m2,
(d) the volume of the sofa in m3.
F

G E Solution
(a) External surface area of carton
= 6.4 × 2 × 12 × 2 + 6.4 × 3 × 12 × 2 + 6.4 × 2 ×
63 6.4 × 3
40 C = 12.8 × 12 × 2 + 19.2 × 12 × 2 + 12.8 × 19.2
D
20 = 1010 cm2  (correct to 3 s.f.)
A 72 B (b) Total volume of the cans
6.4 2
=r × 12 × 6
Solution 2
(a) GE = 72 – 40 = 2316.53376
= 32 cm = 2320  (correct to 3 s.f.)
Perimeter of the cross-section (c) Percentage of empty space
1
= 72 + 20 + 40 + 43 + 63 + × 2r × 16 12.8 × 19.2 × 12 – 2316.53376
2 = × 100%
12.8 × 19.2 × 12
= 238 + 16r
= 288.27 = 21.5%  (correct to 3 s.f.)
= 288 cm  (correct to 3 s.f.)
19. (a) State and draw two items in your home that are
(b) Area of the cross-section
1 made up of basic solids.
= 20 × 40 + 63 × 32 + × r × 162 (b) Measure their dimensions and calculate their
2
= 2816 + 128r volumes and total surface areas.
= 3218.12
= 3220 cm2  (correct to 3 s.f.) Solution
For (a) and (b), the answers vary.
(c) Total surface area of the sofa Examples of items that are made up of basic solids are
= (20 + 40 + 43 + 63 + 72 + 16r) × 180 as follows:
+ 2 × 3218.12 A table may be in the form of a cuboid with 4 cylindrical
= 288.27 × 180 + 2 × 3218.12 legs.
= 58 324.84 cm2 A candleholder may be a cube with a cylindrical hole.
58 324.84
= m2
10 000
= 5.832 484 m2
= 5.83 m2  (correct to 3 s.f.)

Chapter 15  Volume and Surface Area of Solids


182
Revision Exercise 15 4. The length and breadth of a cuboid are 18 cm and
Give your answers correct to 3 significant figures where 13 cm respectively. If the volume of the cuboid is
applicable. 2574 cm3, find
(a) the height of the cuboid,
1. The dimensions of a $50 note are 15.6 cm by 7.4 cm. (b) the total surface area of the cuboid.
Express the area of the note in
(a) cm2, (b) m2. Solution
2574
Solution (a) Height =
18 × 13
= 15.6 × 7.4 cm2
(a) Area = 11 cm
= 115.44 cm2
= 115 cm2  (correct to 3 s.f.) (b) Total surface area
= 2(18 × 13 + 18 × 11 + 13 × 11)
= 0.156 × 0.074 cm2
(b) Area = 1150 cm2
= 0.011544 cm2
= 0.0115 cm2  (correct to 3 s.f.) 5. The figure shows the net of a prism.
BC = PB = PC = 2 cm, DE = 2.5 cm and
2. The volume of a cube is 729 cm3. Find PN = 3 cm.
(a) the length of a side of the cube, (a) Name the prism.
(b) the total surface area of the cube. (b) Find the volume of the prism.
(c) Find its total surface area.
Solution P
3
(a) Length of a side of the cube = 729
= 9 cm 2
3
2

A B N C D
(b) Total surface area of the cube = 6 × 92 2
= 486 cm2
2.5

3. A cuboid is 3.5 cm long, 2 cm wide and 1.5 cm high.


(a) Draw a net of the cuboid. H G F E

(b) Find the volume of the cuboid.


(c) Find its total surface area.
Q
Solution
Solution (a) The prism is a triangular prism.
(a) The following is a net of the cuboid.
(b) Area of BCP
1
= ×2× 3
2
1.5
= 3 cm2
Volume of the prism
= 3 × 2.5
= 4.33 cm3  (correct to 3 s.f.)

3.5 (c) Total surface area of the prism


= 2.5 × 6 + 2 × 3
= 18.5 cm2  (correct to 3 s.f.)

6. The figure shows the cross-section ABCD of a prism of


height 2.5 cm. AB = 1.8 cm, BC = 1.6 cm, BD = 3.0 cm
1.5
and ∠BAD = ∠CBD = 90°.
D
2

(b) Volume of the cuboid = 3.5 × 2 × 1.5


= 10.5 cm3 3.0
(c) Total surface area of the cuboid C
= 2 × (3.5 × 2 + 3.5 × 1.5 + 2 × 1.5)
1.6
= 30.5 cm2
A 1.8 B

183
(a) Construct the quadrilateral ABCD using a protractor, (f) Volume of the prism = 4.56 × 2.5 = 11.4 cm3
a pair of compasses and a ruler. (g) Total surface area of the prism
(b) Measure the lengths of AD and CD. = 9.2 × 2.5 + 2 × 4.56
(c) Find the perimeter of ABCD. = 32.12 cm2
(d) Find the area of ABCD.
(e) Draw a net of the prism.
(f ) Find the volume of the prism. 7. The base radius and height of a cylinder are 6 cm and
(g) Find its total surface area. 10 cm respectively.
(a) Find, in terms of r, the volume of the cylinder.
Solution (b) If the base radius increases by 20% and the height
(a) D decreases by 10%, taking r to be 3.142, find
(i) the new base radius,
(ii) the new height,
(iii) the percentage change in the volume of the
3.0 C cylinder.
1.6
Solution
A 1.8 B
(a) Volume of cylinder
Construction Steps: = r × 62 × 10
1. Draw a line segment AB 1.8 cm long. = 360 cm3
2. Draw a ray from A and perpendicular to AB. (b) (i) New base radius = 120% × 6
3. With B as centre and radius 3.0 cm, draw an = 7.2 cm
arc to cut the ray at D. (ii) New height = 90% × 10
4. Draw the line segment BD. = 9 cm
5. Draw BC 1.6 cm long and perpendicular to BD.
(iii) New volume of cylinder
6. Draw the line segment CD.
= r × (7.2)2 × 9
7. Then ABCD is the required quadrilateral.
= 466.56r cm3
(b) Length of AD = 2.4 cm
Length of CD = 3.4 cm Percentage change in volume
(c) Perimeter of ABCD = 1.8 + 1.6 + 3.4 + 2.4 466.56r – 360r
= × 100%
= 9.2 cm 360r
(d) Area of ABCD = 29.6%
1 1 There is a 29.6% increase in volume.
= × 1.8 × 2.4 + × 3.0 × 1.6
2 2
2
= 4.56 cm
8. The figure shows an arrow sign which is in the form of a
(e) 2.4 prism. It is given that DK = 2.5 cm, AB = AG = 10 cm,
AN = 6 cm, BG = 16 cm, CD = 15 cm, DE = 8
cm and BC = GF.
1.8 (a) Sketch the view of the sign when viewing from the
3.4 top.
(b) Find the total surface area of the sign.
1.6 (c) Find the volume of the sign.
1.6

N
G
M
L
10 E
F
3.4 N
16 A 6 8
K
10 C 2.5
15 D
I
3.4
B

2.4

1.6
1.8 2.5

Chapter 15  Volume and Surface Area of Solids


184
Solution (b) Area of the cross-section
(a) 1
= 60 × 90 + × r × 302
2
= 5400 + 450r
2.5
= 6813.7
= 6810 cm2  (correct to 3 s.f.)
(c) (i) Surface area of the cabinet
Diagram is not drawn to exact scale. = 334.25 × 40 + 2 × 6813.7
= 26 997.4
3.75 = 27 000 cm2  (correct to 3 s.f.)
(ii) Surface area of the cabinet
27 000
= m2
10 000
0.6 = 2.70 m2

(b) Total surface area (d) (i) Volume of the cabinet
= 2 × 6 × 16 ×
1
+ 15 × 8 × 2 + 2 × 4 × 2.5 + = 6813.7 × 40
2 = 272 548
2 × 2.5 × 15 + 8 × 2.5 + 2 × 10 × 2.5 = 273 000 cm3  (correct to 3 s.f.)
= 501 cm2 (ii) Volume of the cabinet
273 000
(c) Volume = m3
1 000 000
1 = 0.273 m3
=  × 6 × 16 × 2.5 + 15 × 2.5 × 8
2
= 420 cm3
10. The figure shows the uniform cross-section of
a tray. ABF and CDE are quarters of a circle.
9. The figure shows a cabinet whose uniform cross- AF = DE = 20 cm, EF = 30 cm and the length of the
section is a semicircle CDE on a rectangle ABCE. tray is 30 cm.
AB = 60 cm, AE = 90 cm and BG = 40 cm. (a) Find the external surface area of the tray.
I (b) Find the volume of the tray.
D (c) A cylindrical bin of height 40 cm has a volume
H equal to that of the tray. Find its base radius.
(Take r to be 3.142)
E C

G H
90
30

G 20 F 30 E 20
A D
40
A 60 B
Find B C
(a) the perimeter of the cross-section,
(b) the area of the cross-section, Solution
(c) the surface area of the cabinet (a) Area of the cross-section
(i) in cm2, 1
= 30 × 20 + × r × 202
(ii) in m2. 2
(d) the volume of the cabinet = 600 + 200r
(i) in cm3, = 1228 cm2
(ii) in m3. Length of the curve ABCD
Give your answers correct to 3 significant figures.
1
= 30 + × 2r × 20
2
Solution = 30 + 20r
(a) Perimeter of the cross-section = 92.83 cm
1
= 60 + 90 + 90 + × 2r × 30 External surface area of the tray
2
= 240 + 30r = 92.83 × 30 + 2 × 1228
= 334.25 = 5240.9
= 334 cm  (correct to 3 s.f.) = 5240 cm2  (correct to 3 s.f.)

185
(b) Volume of the tray 12. A circular pipe is 30 cm long. Its internal and external
= 1228 × 30 diameters are 3 cm and 4 cm respectively.
= 36 840 (a) Find the volume of the material used in making
= 36 800 cm3  (correct to 3 s.f.) the pipe.
(b) Find the total surface area of the pipe.
(c) Let the base radius of the bin be r cm.
(c) If water flows through the pipe at a rate of 5 cm/s
rr 2 × 40 = 36 840
into an empty rectangular tank, find the volume of
r = 921 water delivered in 20 seconds.
r (d) If the rectangular tank is 28 cm long and 20 cm
= 17.1  (correct to 3 s.f.) wide, find the depth of water in the tank after 20
The required base radius is 17.1 cm. seconds.
(Take r to be 3.142)

11. The figure shows an open wooden box. Its external Solution
length, breadth and height are 25 cm, 22 cm and 20 cm (a) Area of the cross-section of the pipe
respectively. The thickness of the wood is 2 cm. Find = r × 22 – r × 1.52
(a) the external volume of the box, = 1.75r cm2
(b) the internal volume of the box, Volume of material used
(c) the volume of wood used in making the box, = 1.75r × 30
(d) the total surface area of the box. = 52.5r
= 165 cm3  (correct to 3 s.f.)
(b) Total surface area of the pipe
2 = 2r × 2 × 30 + 2r × 1.5 × 30 + 2 × 1.75r
20 = 213.5r
= 671 cm2  (correct to 3 s.f.)
(c) Volume of water delivered in 20 s
22 = r × 1.52 × 5 × 20
= 225r
25 = 707 cm3  (correct to 3 s.f.)

Solution (d) Depth of water


(a)
External volume of the box 225r
=
= 25 × 22 × 20 28 × 20
= 11 000 cm3 = 1.26 cm  (correct to 3 s.f.)
(b)
Internal volume of the box
= 21 × 18 × 18
= 6804 cm3
(c)
Volume of wood used
= 11 000 – 6804
= 4196 cm3
(d)
Total surface area of the box
= Total surface area of a closed box with external
dimensions + sum of the areas of the 4 internal
side walls
= 2 × (25 × 22 + 25 × 20 + 20 × 22) +
2 × (21 + 18) × 18
= 4384 cm2

Chapter 15  Volume and Surface Area of Solids


186
16 Data Handling

Class Activity 1
Objective: To select an appropriate data collection method.

Task
Discuss and suggest an appropriate way (taking measurements, observing outcomes, conducting surveys, reading publications)
to collect data to inform the following statements.

1. Favourite sports of Secondary One students in your school

conducting a survey

2. Body temperatures of a group of patients

taking measurements

3. Popularity of a pop singer

conducting surveys

4. Opinions of customers about a bank’s services

conducting surveys

5. Effectiveness of a new medicine

taking measurements

6. Population of Thailand, Malaysia and Indonesia in the year 2010

reading publications

187
Class Activity 2
Objective: To discuss the advantages and disadvantages of using pictogram and bar graph.

1. The following table shows the distribution of different types of trees in a farm.

Tree Apple Peach Pear Plum


Number of trees 32 16 20 9

You are the owner of the farm and would like to present the data by a graph to the visitors.
If the data is to be represented by a pictogram or bar graph, which graph

(a) is easier to draw?

The bar graph is easier to draw.                                                                                                                                   

(b) is more appealing to the public?

The pictogram is more appealing to the public.                                                                                                                

(c) is easier to read the number of trees?

It is easier to read the number of trees from the bar graph.                                                                                                

2. The following table shows the ages of students in a tuition centre.

Age (years) 12 13 14 15 16
Number of male students 16 23 16 19 8
Number of female students 12 17 20 15 13

Between pictogram and bar graph, which graph would you think is more appropriate to represent the data? Explain your
answer.

The data cannot be represented by a pictogram. An icon cannot represent both male and female students.                                                   

Chapter 16  Data Handling


188
Class Activity 3
Objective: To discuss the advantages and disadvantages of using line graph and pie chart.

Questions
1. The table shows the revenue of a new company in its first 4 quarters.

Quarter First Second Third Fourth


Revenue ($’000) 362 409 498 531

You are helping the chairman of this company to draw either a line graph or a pie chart representing the data for this
presentation.

(a) Which diagram would you choose so that it is easier to draw?

The line graph is easier to draw.           

(b) Which diagram would meet each of the following objectives of the chairman’s presentation?

(i) more appealing to the public

The pie chart is more appealing to the public. This is because each sector in a pie chart can be represented by different colours.

(ii) easier for the audience to read the difference between the revenues in the third quarter and the
second quarter

The line graph would be preferred. This is because the reading of the revenue in each quarter can be read from the scales on the vertical

axis.

(iii) can show the proportion of the first quarter revenue in the whole year revenue

The pie chart can show the proportion of the first quarter revenue in the whole year revenue. This proportion is the proportion of the area of

the sector representing the first quarter revenue in the whole circle.

(iv) can show the trend of the revenue in these 4 quarters

The line graph can show the trend of the revenue in these 4 quarters. The trend can be observed as the general movement of the line graph.

2. The table shows the time that Eric spent on four home assignments in one day.

Assignment 1 2 3 4
Time (minutes) 45 30 60 45

Between line graph and pie chart, which graph is more appropriate to represent the data? Explain your answer.

A pie chart is more appropriate as the data does not involve a time series.                                                                                          

189
3. The table shows the price index of the private residential property in 6 consecutive quarters in Singapore.

Year/Quarter 2010 Q1 2010 Q2 2010 Q3 2010 Q4 2011 Q1 2011 Q2


Index 175.0 184.2 189.6 194.8 199.1 203.0

Suppose the aim of drawing a graph is to predict the index in the third quarter of 2011. Given a choice between the line
graph and pie chart, which graph is more appropriate? Explain your answer.

It is meaningless to draw a pie chart to represent the data. There is no meaning for both the sum of the given figures and each figure as a portion of

the sum. To predict the index in the third quarter of 2011, extend the line graph to 2011 Q3 to get an expected index of 206.                 

Class Activity 4
Objective: To work collaboratively on a survey to collect and classify data, present data using appropriate graphs, and analyse
data. Use the data to make some decisions.

Notes to teacher
This activity wants students to apply the statistical techniques they have learnt to conduct a survey. They should be guided at
various stages. Teachers may check and discuss their questionnaire before their data collection. Students should be reminded
to use appropriate graphs to represent their data. When analysing data, students should first set the facts they want to reflect
in the survey.

Tasks
1. Before collecting data, students must design a questionnaire. They should be aware the following points:
• The questionnaire should have a brief description of the objective of the survey.
• The questionnaire should not be too long.
• Questions should be related to the objective.
• Questions should be in simple and clear language, and appropriate format.
• They should first ask a few classmates to attempt the draft questionnaire to examine whether there are any problems
in the questionnaire design.

Students have to decide the target interviewees, the sample size, the way, the place and the time to collect data. They
should make sure that they are in a safe environment when collecting data.

2. The data collected in each question can be organised into tables. Each set of data may have different way of presenting
them.

3. Students may enter their data into a spreadsheet program and generate graphs using the Graph Wizard of the program.

4. In analyzing data, students may use percentage and ratio. For instance, calculate the percentage of students who are satisfied
with the cleanliness of the canteen or the ratio of rubbish that is recyclable. Teachers may need to set some guidelines for
the presentation of the report to students.

Chapter 16  Data Handling


190
Class Activity 5
Objective: To compare various statistical graphs and justify why a particular graph is more suitable than others for a given
situation.

Questions
1. The table shows the number of residential broadband subscriptions in Singapore in recent years.

Year 2005 2006 2007 2008 2009 2010


Number of subscriptions (’000) 576 697 879 1154 1655 2231

If you are going to present this set of data and show the trend of the data to your classmates, which type of statistical
graph is most appropriate? Give your reasons.

The line graph is the most appropriate to represent the data. The data forms a time series and we can visualize the trend of movement of the data from the

line graph.                                                                                                                                                                                                                      

2. The table shows the average monthly incomes of employees from four different industries in Singapore in the year 2010.

Industry Average monthly income ($)


Accommodation & Food 1493
Information & Communications 5350
Financial & Insurance 7663
Real Estate 3094

If you are going to present this set of data to some school leavers, which type of graph is most appropriate? Give your
reasons.

The bar graph is most appropriate to represent the data. It is easy to read the difference between the average monthly incomes of two industries

from the bar graph.                                                                                                                                                                                       

3. The table shows the percentage of resident non-students in Singapore aged 25 – 39 years having various education
attainments in the year 2010.

Educational level Percentage


Below secondary 8.3%
Secondary 13.9%
Post-secondary (Non-tertiary) 11.2%
Diploma and Professional Qualification 22.5%
University 44.1%

If this set of data is going to be published in a newspaper, which type of graph is most appropriate to represent the data?
Give your reasons.

The pie chart is most appropriate to represent the data as it is visually appealing to the public. The public can easily observe the proportion of residents in

each education level from the pie chart.                                                                                                

191
4. The table shows the number of rainy days in each of the first 6 months in the year 2011 in Singapore.

Month January February March April May June


Number of rainy days 17 7 22 12 15 15

If you are going to present this set of data and show the trend of the data to your classmates, which type of statistical
graph is most appropriate? Give your reasons.

The pictogram is most appropriate to represent the data. This is because a pictogram is visually interesting and appealing to the public. At a glance, the

public would know which mouth is the wettest month.

5. Work in groups with your classmates and write down the advantages and disadvantages of the following types of statistical
graphs.
(a) Pictogram (b) Bar graph
(c) Line graph (d) Pie chart

Refer students to page 197 of the textbook 1B.

Statistical Graph Advantages Disadvantages


Pictogram • easy to read • hard to quantify a partial symbol
• visually appealing • degree of accuracy of figures is low
• inconvenient to draw by hand
• not good for too many classes
Bar graph • easy to draw • cannot show the percentage of each
• easy to read class
• easy to compare differences
between two classes
• can show subgroups of each class
Line graph • easy to draw • only applicable to time-related data
• easy to read
• can show trend of the data
• can be used for prediction
Pie chart • visually appealing • cannot read the frequency of each class
• can compare a part to the whole unless specified
• cannot read the difference between two
classes
• not good for too many classes
• need to work out proportions before
drawing
• can only provide an estimate for each
proportion

Chapter 16  Data Handling


192
Class Activity 6
Objective: To compare two statistical diagrams and make inferences from them.

Questions
Two branch managers present the following line graphs of annual sales of their stores to the board of directors.

Annual sales of branch A Annual sales of branch B

54 100
Sales ($ million)

Sales ($ million)
52 80

50 60

48 40
2010 2011 2012 2010 2011 2012
Year Year

he manager of branch A claimed that his branch had a higher percentage increase if compared with branch B from 2010 to
T
2012. Do you think his claim is correct? Explain your answer.

For branch A,
sales in 2010 = $50 million
sales in 2012 = $54 million
54 – 50
\  percentage increase in sales from 2010 to 2012 = 50 × 100%
= 8%

For branch B,
sales in 2010 = $60 million
sales in 2012 = $80 million
80 – 60
\  percentage increase in sales from 2010 to 2012 = 60 × 100%
1
= 33 3 %

Manager A’s claim is incorrect. Both graphs use different scales to represent their sales. The scale used for branch A misleads readers into thinking branch A

had a higher percentage increase.    


 

193
Try It! Section 16.3
Section 16.2 3. The pictogram below shows the number of plants in a
1. The following list shows the fitness grades ranging from flowerbed.
Number of plants in a flowerbed
A to E of 36 students.
Daffodil
B B A C D C A B E        
C A D B C E D C B Marigold
D E E A D B C B A      
E C B C E C D C D Orchid
             
Represent the data using a frequency table. Tulip
           
Solution
represents 5 plants
Fitness Grade Tally Frequency
A //// 5 (a) Which type of plant is the most available in the
flowerbed?
B //// /// 8
(b) Find the difference in the numbers of marigold and
C ////  //// 10 tulip plants in the flowerbed.
D //// // 7
E //// / 6 Solution
(a) The row of orchid has the most number of symbols.
    Total 36
Therefore, orchid is most available in the flowerbed.
(b) Difference = 32 – 18
2. The following list shows the prices (in dollars) of = 14
24 books. number of tulips – number of marigold

23 45 19 18 33 36 25 37 4. The following table shows the numbers of reported


47 30 29 40 34 25 38 12 dengue cases in 4 weeks of July 2011.
32 13 41 20 26 34 28 22

03-09 10-16 17-23 24-30
Week
(a) Represent the data in a frequency table with class July July July July
intervals 10  x  20, 20  x  30 and so on.
(b) Find the percentage of books whose prices are Number of
208 215 226 263
higher than $30. dengue cases
Source: http://www.nea.gov.sg
Solution
(a) Represent the data by a pictogram.
Price ($) Tally Frequency (b) Read from the pictogram the week in which the
10 x  20 ////  5 number of reported dengue cases was the least.
20  x  30 //// /// 8
Solution
30  x  40 //// /// 8 (a)
Number of reported dengue cases in 4 weeks of
40  x  50 /// 3 July 2011
    Total 24
03-39 July
       

(b) Percentage of books with prices higher than $30 10-16 July
       
3+8
= × 100%
24 17-23 July
11
       
= × 100%
24
24-30 July
= 45.8%  (correct to 3 s.f.)          
represents 5 plants

(b) The number of cases is the least in the week of


03 – 09 July.
Chapter 16  Data Handling
194
5. The stock (in kg) of grades A, B, C and D tea in shop Solution
XYZ is shown in the following table. (a) The sales of coffee (30 cups) were highest at
8 a.m. The sales of coffee (18 cups) were lowest
Grade of tea A B C D at 10 a.m.
Mass (kg) 5 10 13 8 (b) The sales of tea were more than those of coffee at
10 a.m.
(a) Find the total mass of tea in the stock.
(b) Represent the data by a bar graph. (c) Total number of cups of coffee sold
(c) Which grade of tea has the lowest stock? = 30 + 26 + 18 + 20
= 94
Solution
(a) Total mass of tea = 5 + 10 + 13 + 8
= 36 kg Section 16.4
7. The following table shows the number of childbriths per
(b)
Stock of tea in shop XYZ 1000 population of Singapore per year for every decade
from 1970 to 2010.

15 Year 1970 1980 1990 2000 2010


Crude birth
22.1 17.6 18.2 13.7 9.3
Mass (kg)

10 rate

5 (a) Draw a line graph to represent the data.


(b) Describe the trend of the birth rate.
(c) Based on the graph, estimate the birth rate of
0
A B C D Singapore in the year 2020.
Grade
Solution
(c) Grade A has the lowest stock. (a)
Crude birth rate per 1000
population of Singapore

6. The bar graph shows the sales of coffee and tea at a stall
in 4 hours on a certain morning. 20

Sales of coffee and tea at a stall

coffee
tea
40
Crude birth rate (thousands)
Number of cups

30

20

10 10

O
08 00 – 08 59 09 00 – 09 59 10 00 –10 59 11 00 –11 59
Time

(a) Compare the sales of coffee over the four hours.


(b) During which period were the sales of tea more O 1970 1980 1990 2000 2010 2020
than those of coffee?
Year
(c) How many cups of coffee were sold during the
four hours? (b) The trend of the crude birth rate is decreasing from
1970 to 2010, with the exception of a slight rise in
1990.
(c) From the graph, the estimated crude birth rate in
2020 is 5 babies per 1000 population.

195
8. The line graph below shows the Consumer Price Index Solution
of Country X from 2004 to 2010, where the index in Expenses
2009 is set to 100. Category Angle at Centre
($ million)
420
Consumer Price Index of Country X Transport 420 × 360° = 79.0°
1914

673
110 Trade & Industry 673 × 360° = 126.6°
1914

746
Consumer price index

Manpower 746 × 360° = 140.3°


1914

100 Info-communications 75
technology and 75 × 360° = 14.1°
1914
media development
Total 1914 360.0°

90

Trade &
O Manpower Industry
2004 2005 2006 2007 2008 2009 2010
Year 140.3° 126.6°

(a) Describe the trend of the Consumer Price Index Transport


between 2004 and 2010. 79.0°

(b) Find the lowest Consumer Price Index between 14.1°


2004 and 2010. Info-communications
technology and
(c) Which year had the greatest rise in the index? What media development
was the greatest rise?

Solution 10. In a survey, each interviewee was asked about his/her


(a) The trend of the CPI between 2004 and 2010 is favourite fruit juice. The results are shown on the given
increasing. It rises steadily in the first 3 years and pie chart.
has a steep rise in 2008.
(b) The lowest CPI is at the year 2004. The index is
Mango Orange
90.
135°
(c) The greatest rise occurred in the year 2008. x°
75°
Others
The greatest rise = 99 – 93
Apple
=6
Favourite fruit juice

9. In the Singapore government’s expenditure in the year


(a) Find the value of x.
2010, the economic development expenses can be
(b) Find the percentage of interviewees who chose
subdivided into 4 categories as given in the table below.
mango juice.
Info- (c) If 40 interviewees selected apple juice, find
Category
Trade & communications (i) the total number of interviewees,
Transport Manpower
Industry technology and (ii) the number of interviewees who chose orange
media development juice.
Expenses
420 673 746 75
($ million) Solution
Construct a pie chart to represent the data. (a) 90° + 135° + x° + 75° = 360° (∠s at a point)
∴  x = 60
(b) Percentage of interviewees who chose mango juice
135
= × 100%
360
= 37.5%

Chapter 16  Data Handling


196
(c) (i) Total number of interviewees Line graph:
60
= 40 ÷
360 Sales of cars from January to April
360
= 40 ×
60
= 240 20
(ii) Number of interviewees who chose orange
juice 15
90
= 240 ×
360 10
= 60
5

11. The following table shows the sales of cars of a company in


four months. O Jan Feb Mar Apr
Number of cars sold
Month Number of cars sold
January 23 (b) The pictogram is more appealing to the public.
February 15 (c) The line graph can reveal the trend of sales.
March 11 (d) Both diagrams can indicate the month with the
April 8 lowest sales.

(a) Present the data by a pictogram and a line graph 12. The bar graph shows the times taken by 3 students to
respectively. complete a mathematics assignment. By observing the
(b) Which of the diagrams drawn in (a) is more bar graph, can we claim that Ling took three times as
appealing to the public? long as Majid to complete the mathematics assignment?
(c) Which of the diagrams drawn in (a) can reveal the Explain your answer.
trend of the sales?
(d) Which of the diagrams drawn in (a) can indicate Time taken for a mathematics assignment
the month with the lowest sales?

Solution 45
Time taken (min)

(a)
Sales of cars of a company in four months
40
January
       
35
February
     
30
Ali Ling Majid
March
    Student

April

Solution
represents 5 cases Ratio of time taken by Majid to Ling
 = 35 : 45
= 7 : 9
Therefore, we cannot claim that Ling took three times
as long as Majid.

197
13. The following diagram is released by Singapore Police 14. The line graph shows the energy level of a person after
Force. It shows the fatal and injury rate of traffic accidents drinking Bull tea.
in Singapore from 2007 to 2011. (a) What is the message that the graph wants to convey
to readers?
Fatal and Injury Accident Rate (2007 – 2011) (b) The advertisement claims that ‘You will be full of
1,000,000 120.00 energy 30 minutes after drinking Bull tea.’ Would
950,000
you believe it? Explain your answer.
110.00

900,000 100.98
98.17
Energy level of a person after drinking Bull Tea
100.00
850,000 93.41
91.96 90.00
800,000
83.31
80.00

Energy level
750,000

700,000 70.00
2007 2008 2009 2010 2011

Vehicle Population Fatal and Injury Accident Rate per 10,000 Vehicle Population

Source: Singapore Police Force


http://driving-in-singapore.spf.gov.sg/services/Driving_in_
Singapore/information_traffic_statistics.htm O 15 30 45 60
Time (minutes)
In 2010, the total number of fatal and injury accidents
was 8625. In 2011, the total number of cases decreased
to 7925. Solution
(a) The graph wants to lead the readers to think that
Alan observes the diagram and the total numbers of fatal
their energy level will become very high in 30
and injury accident in these two years. He claims that
minutes after drinking BULL tea.
the total number of fatal and injury accidents in 2011
decreases by 8.65% when compared with the figure in (b) We should not believe the advertising claim.
2010. Could you justify whether his claim is correct?
The reasons are as follows:
(i) The ‘energy level’ is not a common scientific
Solution
unit. There is no indication of how the energy
Actual percentage decrease
level of a person is measured.
8625 – 7925
 = × 100%
8625 (ii) The energy level axis has no scale. We have
= 8.12% no idea how much energy is replenished after
drinking BULL tea.
Alan’s calculation of percentage decrease
 = (91.96 – 83.31%)
= 8.65%
From the diagram, there is a decrease in the total number
of fatal and injury accidents from 2010 to 2011. However,
Alan’s claim is incorrect as the vehicle population in 2010
and 2011 are different. The actual percentage decrease
is 8.12%.

Chapter 16  Data Handling


198
Exercise 16.1 (b) The data are masses of chickens at regular time
Level 1 intervals.
1. Huili wanted to determine whether a die was biased. She
tossed the die 120 times and recorded the results. Level 3
5. Consider the following question in a questionnaire of a
hotel.

How do you rate our services?
(a) What kind of data-collection method did she use? 1 2 3 4 5 6 7
(b) What are the possible results in each toss?
Point out two areas for improvement in the design of
Solution this question.
(a) She used the method of taking measurements in an
experiment. Solution
(b) The possible scores in each toss are 1, 2, 3, 4, 5 or (1) The question is better if it asks about the rating of
6. a specific service of the hotel.
(2) We do not know whether option 1 or 7 is the best.
2. A teacher wants to find out the number of his Secondary
One students who have birthdays falling in each of the
12 months. He collects the data in the following ways. 6. A restaurant manager requests her customers to fill in
(a) Checking the date of birth of each student in the the following questionnaire.
class registry.
(b) Asking the students to raise their hands if their ABC Restaurant
birthdays fall within a certain month. Customer Satisfaction Survey
(c) Designing a form and get the students to fill in
their birth dates and return the forms to him. 1. How would you rate our food?
Name the data-collection method used in each case. very good  good  OK
Solution bad  very bad
(a) reading publications
(b) observing outcomes
2. Is the service of our staff up to your
expectation?
(c) conducting surveys
(a) What type of data-collection method does the
3. A hospital conducts a study on the causes of sports manager use?
injuries. (b) Which question in the survey form is better? Why?
(a) Which data collection method would be appropriate (c) Is it a good idea to ask customers to fill in the date
for conducting this study? of visit in the form? Why?
(b) Suggest a purpose for the study.
Solution
Solution (a) The method used is conducting surveys.
(a) We can collect the data by conducting a survey on
patients with sports injuries. (b) Question 1 is better. It is simple and clear, easy to
complete and can convey useful information to the
(b) The purpose of study may be to educate the public manager.
on the prevention of sports injuries.
(c) It is good to fill in the date of visit. This is because
the restaurant may have different teams of staff on
4. A zoologist studies the growth rates of chickens on different dates. The quality of service of each team
different feeds. may be different.
(a) Which data collection method should be used?
(b) What data should be collected?
7. Collect a survey form and answer the following questions
Solution based on it.
(a) The data can be collected by taking measurements (a) State the objective of the survey.
of the masses of chickens which have different (b) Describe the styles of the questions in it.
feeds.

199
(c) Describe some of its good features. Solution
(d) Suggest some ways to improve it.
Number of pets Tally Frequency
Solution 0 //// // 7
(a) Advise students that each survey must have its 1 ////  //// 10
objective or goal. 2 //// // 7
(b) Questions may be 3 ////  4
(i) two-state, 4 // 2
(ii) multiple choice, and     Total 30

(iii) open-ended types.


3. Ali, Ben and Cili are candidates for the post of councillor
(c) Good questions should of a class. The following list shows the votes received
(i) be simple and easy to understand, by the three candidates from the 40 students in the class.
(ii) not be ambiguous,
Ben Ben Ali Ben Cili Cili Ali Ben Cili Ali
(iii) not be biased. Ali Cili Cili Ben Ben Ali Cili Ben Ali Cili
(d) Observe the design and ordering of the questions. Cili Ben Ben Ali Cili Ben Ali Cili Ben Ben
Ali Ben Cili Cili Ben Ben Ali Cili Cili Ben

(a) Construct a frequency table for the data.


Exercise 16.2 (b) Who will be the class councillor according to the
Level 1 votes?
1. The following list shows the ages, measured to the nearest
year, of students in a tuition class. Solution

   (a) Candidate Tally Frequency
13 14 13 12 15 14 13 13
Ali //// //// 10
12 13 13 14 14 15 12 13
13 12 14 14 13 14 13 12 Ben //// //// //// / 16
14 15 13 13 12 13 14 13 Cili //// //// //// 14

Construct a frequency table for the data.    Total 40

(b) Ben will be the class councillor.


Solution

Age (Year) Tally Frequency


Level 2
12 //// / 6
4. The following list is a record of the grades of 40 students
13 ////  //// //// 14
in a Mathematics examination.
14 //// //// 9
15 /// 3 B C A D E A B C C B
    Total 32 D C C A B D E C D C
B B D E A D B D E E
C E C B D C C A D C
2. The following list shows the number of pets in each of
30 families.
(a) Construct a frequency table for the data.
3 2 2 1 1 0 1 4 2 1 (b) What was the grade obtained by the most number
1 0 0 1 2 3 0 1 1 0 of students?
3 2 4 1 0 0 2 1 3 2 (c) Find the percentage of students who obtain grade
A or grade B.
Construct a frequency table for the data.

Chapter 16  Data Handling


200
Solution (a) Construct a frequency table of uniform intervals
for the data, using the intervals 29, x < 39, 39,
  (a) Grade Tally Frequency
x < 49 and so on.
A //// 5 (b) The passing mark for the test is 50. Find the
B //// /// 8 percentage of students who passed the test.
C //// //// // 12
Solution
D //// //// 9
  (a) Test scores (x) Tally Frequency
E //// / 6
   Total 40 29  x  39 /// 3
39  x  49 //// / 5
(b) Grade C is obtained by most students. //// //// 4
49  x  59
(c) Percentage of students who obtain grade A or 59  x  69 //// // 8
5+8
grade B = × 100% //// 6
40 69  x  79
= 32.5% 79  x  89 //// 4
89  x  99 // 2
5. The following list shows the distances (in km) covered
by 30 passengers who took taxis.    Total 32

(b) Number of students whose scores are greater than
63 56 32 41 66 79 45 91 or equal to the passing mark
33 68 75 47 78 61 83 49  =4+8+6+4+2
85 42 77 86 63 60 95 54 = 24
62 88 57 38 64 73 72 55
Percentage of students who passed
(a) Construct a frequency table of uniform intervals 24
= × 100%
for the data, using the intervals 0  x  2, 32
2  x  4, and so on. = 75%
(b) Find the percentage of passengers who covered
more than 6 km. 7. A quality control officer inspected 30 cartons of vases
produced by a factory. The list below shows the number
Solution
of defective vases found in each carton of 10 vases.
  (a) Distance (km) Tally Frequency
0x2 /// 3 1 3 1 4 1 2 0 0 4 0
2x4 //// / 6 4 2 0 1 3 0 1 3 1 2
0 1 3 0 2 1 0 1 2 1
4x6 //// //// 9
6x8 //// // 7 (a) Construct a frequency table for the data.
8  x  10 //// 5 (b) Find the percentage of cartons that contain at most
   Total 30 one defective vase.

Solution
(b) Percentage of passengers who covered more than
7+5   (a) Number of
6 km = × 100% Tally Frequency
30 defective vases
= 40% 0 //// /// 8
1 //// //// 10
Level 3 2 //// 5
6. The following are the scores of 32 students in a science 3 //// 4
test.
4 /// 3
7.2 2.8 4.2 6.0 6.5 3.3 5.3 9.0 7.1 0.8    Total 30
4.1 3.7 8.1 3.6 4.7 6.4 8.4 1.5 5.5 7.9
9.3 5.8 6.9 5.7 1.6 2.3 4.8 4.0 9.2 6.5 (b) Percentage of cartons that contain at most one
8 + 10
defective vase = × 100%
30
= 60%

201
8. The scores in Round 1 of the first 30 golfers in the 2011 Solution
US Masters Tournament were as follows:
  (a) Time (x h) Tally Frequency
0  x  3 //// //// /// 13
69 72 72 71 69 72 71 73 67 71
73 72 72 74 69 69 71 69 65 71 3  x  6 //// // 7
68 67 74 69 72 72 72 74 74 71 //// 4
6  x  9

9  x  12 /// 3
(a) Construct a frequency table for the data.
   Total 27
(b) What is the winning score in Round 1 if the game
was won by the one with lowest score?
(b) Percentage of people who surfed the Internet
(c) If the even-par score is 72, find the percentage of
in hours in the range 3  x  9
golfers who completed the round with an even-par
7+4
score. = × 100%
27

Solution
= 40.7%  (correct to 3 s.f.)
  (a) scores Tally Frequency
10. The following data are the monthly utilities bills (in $)
65 / 1 of 35 families in August 2011.
67 // 2
68 / 1 236.49 315.37 256.88 104.56 418.30 176.40 198.95
69 //// / 6 367.90 187.80 54.73 467.32 238.89 356.60 245.71
287.59 314.90 267.80 378.24 480.43 69.78 135.60
71 //// / 6
78.40 178.10 245.67 314.90 83.20 472.10 174.30
72 //// /// 8 267.80 69.30 134.70 314.60 180.90 245.60 430.15
73 // 2
74 //// 4 (a) Construct a frequency table of uniform intervals
   Total 32 for the data, using the intervals 0 , x  100,
100 , x  200 and so on.
(b) The winning score is 65. (b) Find the percentage of the number of families whose
(c) Percentage of golfers who completed the round monthly utilities bills are more than $200 but less
with an even-par score than or equal to $400.
8
= × 100% Solution
30
= 26.7%  (correct to 3 s.f.)   (a) Test scores (x) Tally Frequency
0  x  100 //// 5
9. A survey investigates the number of hours that a person //// //// 9
100  x  200
surfs the Internet per day. The following list shows the
survey results of 27 people who surfed the Internet. 200  x  300 //// //// 9
300  x  400 //// // 7
1.5 3.2 2.1 3.0 0.8 4.2 11.6 2.7 6.6 400  x  500 //// 5
4.1 1.6 6.3 2.5 2.0 5.8 2.4 8.8 5.4    Total 35
0.4 9.2 2.9 5.7 7.8 1.0 3.9 10.3 2.9

(b) Number of families whose monthly electricity bills
(a) Construct a frequency table of uniform intervals, are more than $200 but less than or equal to $400
using the intervals 0  x  3, 3  x  6 and so = 9 + 7
on. = 16
(b) Find the percentage of people in the survey who Required percentage
spend more than 3 hours but fewer than or equal 16
to 9 hours per day surfing the Internet. = × 100%
35
= 45.7%  (correct to 3 s.f.)

Chapter 16  Data Handling


202
11. In a factory, the lifetime of each of 30 dry batteries was Exercise 16.3
tested. The results (in hours) are shown below. Level 1
1. The pictogram below shows the number of books read
9.2 3.2 7.7 8.0 5.2 8.8 6.9 7.6 3.8 4.1 by 4 students in a month.
7.6 8.4 9.9 3.6 8.4 9.0 5.1 7.5 4.9 9.5
8.1 4.5 7.2 9.5 6.4 6.9 7.4 5.7 7.9 6.3
Number of books read in a month

(a) Construct a frequency table of 4 or 5 uniform Ali


intervals for the data.
(b) Compare your table with those of your classmates.
Brian
Solution
  (a) Lifetime (x h) Tally Frequency Cathy
3  x  5 //// / 6
5  x  7 //// // 7
Donna
7  x  9 //// //// /// 13
9  x  11 //// 4 represents 1 books
   Total 30
(a) Who read the least number of books in the month?
(b) If a classmate uses a different set of class intervals,
(b) How many more books did Ali read than Cathy?
the frequencies may not be the same.
Solution
12. After a survey on the rating of the quality of food in (a) Brian read the least numbers of books.
a school canteen, the results are summarised as shown (b) Number of books read by Ali 
below. Number of books read by Cathy 
Difference = 11 – 8
Rating Frequency
= 3
Very good 22 Ali read 3 more books than Cathy.
Good 57
Fair 61
2. The pictogram below shows the goals scored by four
Poor 35
teams in a football season.
Very poor 25
Write a brief report about the survey based on the Number of goals scored in a football season
information in the table.
Dragon
Solution
Total number of respondents in the survey Horse
= 22 + 57 + 61 + 35 + 25
= 200
Lion
Most respondents selected ‘fair’ for the rating on the
quality of food.
Percentage of respondents with ‘very good’ rating Tiger
22
= × 100%
200 represents 4 goals
= 11%
It is a grave concern that the percentage of respondents
who suggested ‘poor’ and ‘very poor’ ratings is very (a) Which team scored the most number of goals?
high (30%). (b) Find the ratio of the number of goals scored by
35 + 25 Dragon to that scored by Horse.
× 100% = 30%
200
Solution
(a) Lion team scored the most number of goals.
(b) Ration of number of goals scored by Dragon to
that scored by those = 18 : 11

203
3. 5. The bar graph below shows the number of in-coming
Evergreen telephone calls received by an office in a working week.

Number of calls received by an office in a working week


Good Harvest

Reddish
Fri
Sunny

Day of a working week


Thurs
represents 200 kg of apples
Wed
The above pictogram shows the yield of apples from
four farms.
(a) Which farm had the greatest yield? Tues
(b) Find the total yield from the four farms.
Mon
Solution
(a) Farm Evergreen has the greatest yield. O 5 10 15 20 25
Number of calls received
(b) Total yield from the 4 farms
1
= 200 × 6 + 200 × 5 + 200 × 3 + 200 × (a) On which day was the least number of calls
2
received?
1
+ 200 × 4 + 200 × 2 (b) Find the ratio of the number of calls received
on Monday to that on Tuesday.
= 3800 kg
(c) Find the average number of calls received in
4. The bar graph shows the number of members in each the working week.
leisure club at a community centre.
Solution
(a) Which club has the greatest number of members?
(b) What is the total number of members in the four (a) The least number of calls was received on
clubs? Tuesday.
(b) Ratio of the number of calls received on
Number of members of leisure clubs
Monday to that on Tuesday = 24 : 15
= 8 : 5
40 (c) Average number of calls received
24 + 15 + 19 + 16 + 21
=
Number of members

5
30
= 19
The average number of calls received in a week
20
is 19.

10
6. The bar graph shows the numbers of rainy days in city
XYZ over a period of four months.
0
Badminton Cooking Dancing Sewing (a) Which month had the least number of rainy days?
Club (b) How many more rainy days were there in July than
in June?
(c) Find the total number of rainy days in these four
Solution months.
(a) The dancing club has the greatest number of
members.
(b) Total number of members = 29 + 19 + 35 + 24
= 107

Chapter 16  Data Handling


204
Number of rainy days in city XYZ (a) Compare Ahmad’s monthly income over the
3 months.

Number of rainy days


(b) In which month was his income less than his
10 expenditure and by how much?
(c) Determine whether his total income in the three
5 months was greater than his total expenditure for
the same period.

0 Solution
May Jun Jul Aug
Month (a) Ahmad’s monthly income increased steadily from
January to March.
Solution (b) In January, Ahmad’s income was less than his
(a) May had the least number of rainy days. expenditure.
The difference = $2800 – $2300
(b) Number of rainy days in July = 8
= $500
Number of rainy days in June = 5
Difference = 8 – 5 (c) Total income in 3 months
=3 = $(2300 + 2500 + 3000)
There are 3 more rainy days in July than in June. = $7800
Total expenditure in 3 months
= $(2800 + 2500 + 2000)
Level 2
= $7300
7. The table shows the number of different types Hence his total income was greater than his total
of shops in a shopping mall. expenditure.
Rating Frequency
Appliances 8 Level 3
Clothing 19 9. The following table shows the number of medals was
Food 20 won by the top 3 countries in the 2008 Beijing Summer
Shoes 10 Olympics.
Others 15 Country Gold Silver Bronze Total
A pictogram is used to represent the data. It is given that China 51 21 28 100
the number of symbols in each row should not exceed 10.
What is the least number of shops that can be represented USA 36 38 36 110
by each symbol?
Russia 23 21 28 72

Solution
(a) Represent the data using a bar graph.
As the highest frequency is 20, the least number of shops
20
(b) Express the total number of medals won by USA
is = 2. as a percentage of the total number of medals won
10
by China.
8. The bar graph shows Ahmad’s income and expenditure (c) Express the number of gold medals won by USA
for each month from January to March. as a percentage of the number of gold medals won
by China, correct to 3 significant figures.
Income and expenditure of Ahmad

Income
Expenditure

3000
Amount ($)

2000

1000

0
Jan Feb Mar
Month

205
Solution Solution
(a) Represent the data using a bar graph. It is not a fair comparison chart.
1. The scale on the vertical axis does not start from
Gold zero.
2. The DD Brand bar is highlighted (it is different in
Silver
55 colour).
Bronze 3. The width of the DD Brand bar is double the widths
50 of the other brands.
All of the above tricks give an illusion that the annual
45 sales of DD Brand cars is a lot more than the other brands.

40
11. Visit a shopping mall near your home. Count the number
Number of medals

35 of different shops and tabulate your result similar to


Question 7. Draw an appropriate diagram to represent
30 your result.
The answer depends on the data collected.
25

20
Exercise 16.4
15 Level 1
1. The line graph shows the changes in the body temperature
10 (°C) of a patient from 6 a.m. to 12 a.m. on a certain
day.
5 Body temperature of a patient

0 China USA Russia
Countries 41
Temperature (°C)

39
110
(b) Required percentage = × 100%
100
37
= 110%
36 35
(c) Required percentage = × 100%
51
= 70.6% (correct to 3 s.f.) 0
6 a.m. 9 a.m. 12 p.m. 3 p.m. 6 p.m. 9 p.m. 12 a.m.
Time

10. The bar graph shows the annual sales of four models
of cars. It was used in an advertisement by DD Brand (a) What was the highest body temperature of the
Company. Is it a fair comparison chart? Explain your patient? When did the patient have it?
answer. (b) Find the number of readings for temperatures above
37 °C.

Annual sales of cars Solution


(a) The highest body temperature of the patient was
40 °C.
Number of cars sold (thousands)

24
The patient had it at 9 p.m.
23 (b) The number of readings for temperatures above
37 °C is 4.
22

21

20
AA BB CC DD
Brand

Chapter 16  Data Handling


206
2. The line graph shows the closing prices ($) of a stock 4. The pie chart shows the favourite subjects of
in a week. 180 students.

Closing price of a stock Mathematics



20% English
22 25%
Others
Science 10%
21
Price ($)

35% 10%
Geography
20
Favourite subjects of 180 students
19
(a) Which subject do most students like?
O (b) Find the angle at the centre of the sector representing
Mon Tues Wed Thurs Fri Science.
Day (c) Find the number of students whose favourite subject
is English.
(a) On which day is the closing price the lowest?
(b) Describe the trend of the price of the stock. Solution
(a) Most students like Science.
Solution (b) Angle representing Science
(a) The closing price is the lowest on Friday. = 35° × 360°
(b) The price of the stock rose slightly on Tuesday and = 126°
then it dropped on the next three days. (c) Number of students whose favourite subject is
English
3. The pie chart shows how some = 25° × 180
students travel from home to school. = 45
On foot
(a) By which mode do most of Bus
the students travel from home 105°
135°
to school? Others Level 2
(b) Find the percentage of students MRT 5. The line graph below shows the mass (kg) of a baby in
who go to school on foot. the first 12 months.
(c) Find the angle of the sector Modes of travel Mass of a baby
representing ‘Others’. of students from
home to school
Solution 10
(a) Most of the students travel by bus.
8
(b) Percentage of students who go to school on foot
105
Mass (kg)

= × 100% 6
360
= 29.2%  (correct to 3 s.f.)
4
(c) Angle representing 'others'
= 360° – 135° – 105° – 90° 2
= 30°
O 2 4 6 8 10 12
Month

(a) What was the mass of the baby at birth?
(b) How many months after birth was the mass of the
baby exceeding 8 kg?

207
(c) Find the average rate of increase in the mass of the 7. A shop sells three grades of coffee
baby, in kg/month, which are Platinum, Gold and Platinum Silver
(i) during the first two months, Silver. The pie chart represents
5x°
(ii) between the 10th and 12th months, the mass (in kg) of each grade
4x°
(iii) in the first year. of coffee sold on a particular day.
Gold
The total mass of coffee sold was
Solution 14.4 kg.
(a) The mass of the baby at birth is 3.6 kg. Sales of coffee

(b) The mass of the baby was exceeding 8 kg after Find
6.6 months. (a) the value of x,
(b) the percentage of Platinum coffee in the total mass,
(c) (i) The required rate
4.8 – 3.6 (c) the mass of Gold coffee sold.
= 2
= 0.6 kg/month Solution
(a) 4x° + 5x° + 90° = 360°  (∠s at a point)
(ii) The required rate 9x = 270
10 – 9.6 x = 30
= 2
= 0.2 kg/month 90
(b) Percentage of platinum coffee = × 100%
360
(iii) The required rate = 25%
10 – 3.6
= 12 4 × 30
(c) Mass of Gold coffee sold = 14.4 × kg
= 0.533 kg/month (correct to 3 s.f.) 360
= 4.8 kg

6. The line graph shows the balance of Ali’s savings account


8. The pie chart shows the proportions of students by age
at the end of each month from February to June in 2011.
in a class. There are 23 students who are 12 years old.
Savings account balance of Ali Find

12 years

800 230°

30°
600 x° 14 years
Balance ($)

13 years
400
Ages of students
200

(a) the value of x,
(b) the total number of students in the class,
O
Feb Mar Apr May Jun Jul (c) the number of students who are 13 years old.
Month

Solution
(a) In which months was the balance below $500? (a) x° + 30° + 230° = 360°  (∠s at a point)
(b) Can you predict the balance in July? Explain briefly. x = 100
230
Solution (b) Total number of students = 23 ÷
360
(a) The balance was below $500 in the months of = 36
February and March.
(c) Number of students who are 13 years old
100
(b) The balance in July cannot be predicted. This is = 36 ×
360
because the balances in the previous months were = 10
fluctuating and there was no trend.

Chapter 16  Data Handling


208
Level 3 (c) A person should not drive a car until the BAC
9. The line graph below shows the total number (in million) drops below 0.05%. After having the drink, how
of mobile phone subscribers in Singapore from 2006 to long should Jack wait before driving?
2010.
Solution
Number of mobile phone subscribers in Singapore (a) The BAC reached the highest level after 1 hour
with a value of 0.08%.
8 (b) After 4 hours, the BAC would drop to 0.02%.
Number of subscribers (million)

(c) Jack should wait at least 2.5 hours before driving.


7

6 11. The following table shows the profits of a company in


the first four years of its operation.
5 Year 1 2 3 4
Profits ($’000) 100 101.5 102 103
4


O
2006 2007 2008 2009 2010 2011
Year 103

Profits ($’000)
102
(a) Describe the trend of mobile phone subscriptions
between 2006 and 2010. 101
(b) Based on the graph, estimate the number of 100
mobile phone subscribers in Singapore in the
year 2011. 0
1 2 3 4
Year
Solution
Fig. 1
(a) The line graph indicates that the mobile subsciptions
increased steadily from 2006 to 2010.
(b) The estimated number of mobile phone subscriptions 115

in the year 2011 is 7.8 million.
Profits ($’000)

110

10. The line graph below shows the blood alcohol 105
concentration (BAC) of Jack over time after having a
glass of wine. 100

Blood Alchohol Concentration after drinking 0


1 2 3 4
Year
Fig. 2
0.08
(a) Compare the two line graphs above.
0.06 (b) If you were a manager in the company, which line
graph would you use to show the profits? Explain
BAC (%)

0.04 your answer.

Solution
0.02
(a) The profits can be read more accurately in Fig. 1
than in Fig. 2. The variation of the profits seems
O 1 2 3 4 to be greater in Fig. 1.
Time (hours)
(b) A manager would most likely use Fig. 1 to show
the profits. This is because it gives an impression
(a) How many hours after drinking did the BAC reach that the profits grow very fast.
the highest level? What was its value?
(b) What would be the expected BAC 4 hours after
drinking a glass of wine?

209
12. The pie charts below show the proportions of education Solution
expenses in the government operating expenses of Hong (a) No, we cannot conclude that Singapore spent
Kong in 2009/10 and Singapore in 2009/10 respectively. more money on education than Hong Kong. This
is because the area on each chart just indicates the
proportion of expenses on education, not the actual
Education amount spent on education.
x 58.2
(b) (i) x = × 360°
289.0
= 72.5° (correct to 1 d.p.)
8.5174
y = × 360°
32.7584
= 93.6° (correct to 1 d.p.)
Government operating expenses
of Hong Kong in 2009/10 (ii) Education expenses of Hong Kong
= 58.2 billion HKD
58.2
= billion SGD
6.40
Education = 9.09375 billion SGD
y Education expenses of Singapore
= 8.5174 billion SGD
Hong Kong spent more money on education.
(ii) Amount per student in Hong Kong
9.09735 billion SGD
Government operating expenses =
1294500
of Singapore in 2009/10
= 7024.91 billion SGD (correct to 2 d.p.)
(a) Based on the pie charts above, can you conclude Amount per student in Singapore
that Singapore spent more money on education 8.5174 billion SGD
=
than Hong Kong? Explain briefly. 698544
(b) The following table shows some relevant figures. = 12193.08 billion SGD (correct to 2 d.p.)

Hence Singapore spent more money per student on
Hong Kong in Singapore in
education.
2009/10 2009/10
Government
289.0 billion 32.7584 billion 13. The line graph shows the numbers of Singapore residents
operating who were at least 70 years old in the years 1995, 2000,
HKD SGD
expenses 2005 and 2010.

Education 58.2 billion 8.5174 billion
Number of Singapore residents who are at least 70 years old
Expenses HKD SGD
Student
1 294 500 698 544
Enrolment
300
Assume that the exchange rate between Hong Kong
dollars (HKD) and Singapore dollars (SGD) is 1
250
SGD = $6.40 HKD.
Number (thousand)

(i) Find the angles x and y in the pie charts. Give


your answers correct to 1 decimal place. 200
(ii) Which government spent more money on
education? 150
(iii) Which government spent more money per
student on education?
100
Note: The financial years for the governments of Hong
Kong and Singapore are both from 1 April of one O
year to 31 March of the following year. 1995 2000 2005 2010 2015
Year

(a) Estimate the percentage growth in the number of


Singapore residents who were at least 70 years old
from 1995 to 2010.

Chapter 16  Data Handling


210
(b) Estimate the number of Singapore residents who (d) Number of private residential units completed in
are at least 70 years old in 2015 from the graph. the year 2010
(c) Describe the trend of the line graph.
Quarter Number of units
Solution
(a) Percentage growth Q1 1407
225 – 120
= × 100%
120 Q2 4379
= 87.5%
Q3 2581
(b) The estimate is 270 000.
(c) The line graph has an increasing trend. Q4 2032

Solution
Exercise 16.5 (a) line graph
Level 1 (b) pie chart
1. Determine what type of statistical diagram is most (c) bar graph
appropriate for each of the following data sets.
(a) Average number of MRT passengers during (d) bar graph
weekdays in Singapore from 2005 to 2010
2. A hospital is going to draw a statistical graph to show
Year Number of passengers the number of newborn babies in the hospital in the last
three years to the public. Which statistical graph should
2005 1 404 000 it use? Explain briefly.
2006 1 498 000
Solution
2007 1 622 000 A pictogram should be used. As the target audience is the
2008 1 799 000 public, the graph should be easy to read and understand,
as well as usually appealing. Hence the pictogram is most
2009 1 879 000 suitable in this case.
2010 2 124 081
3. David is going to draw a statistical graph to show the
(b) Population of Singapore by age groups at the end mean monthly temperature in each month in the year
of June 2010 2011. Which statistical graph should he use? Explain
briefly.
Age group Population (thousands)

Solution
0 – 19 918.2 David should use a line graph. It can show the variation
20 – 39 1138.5 of temperature in different months, the hottest and the
coldest months.
40 – 59 1184.6
60 – 79 461.3 4. The pictogram shows the number of votes obtained by
3 candidates in an election.
80 and over 69,0
Alfred
(c) Points scored by the top teams in S.League in 2009

Population Betty
Age group
(thousands)
Singapore Armed Forces 67 Carmen

Tampines Rovers 56 represents 500 votes


Gombak United 53
(a) Who was the winner of the election?
Home United 53
(b) In which way does the pictogram mislead the
Super Reds 50 readers?

211
Solution (b) Name a suitable type of statistical graph which can
(a) Betty was the winner of the election. show the proportion of the company staff in each
department.
(b) The symbol used to represent Betty's votes is bigger
(c) Amy wants to represent the data by a line graph.
than those of Alfred and Carmen. It seems that
Do you think it is appropriate to do so? Why?
Betty had much more votes than the others.
Solution
5. The bar graph shows the average monthly incomes of
(a) bar graph
employees in 4 competitive companies.
(b) pie chart
Average monthly incomes of employees
(c) The data does not relate to time. Therefore, it cannot
be represented by a line graph.
5000
Monthly income ($)

4000 7. The bar graph shows the distribution of comments on


the services of a hotel in a survey.
3000
Comments on the services of the hotel

2000
A B C D

Number of responds
Company 30

20
(a) Find the ratio of the average monthly income in
company A to that in company B.
10
(b) Find the ratio of the length of the bar representing
the average monthly income in company A to that
in company B. O Excellent Very Satisfactory Not Poor
(c) Ajib said the average monthly income of employees Good Satisfactory
Comments
in Company D is twice that in Company A. Do you
agree with his statement? Explain your answer.
(a) Find the percentage of comments which are
‘Satisfactory’ or better.
Solution
(b) The manager of the hotel said that it is more
(a) The required ratio = $ 3400 : $ 4200
appropriate to represent the data by a pie chart.
= 17: 21
Do you agree with him? Explain your answer.
(b) The required ratio = 7 : 11
Solution
(c) The vertical scale does not start from zero. The
(a) Total number of responses
readers may use the heights of the bars to compare
= 16 + 30 + 20 + 10 + 4
the actual average monthly incomes of different
= 80
companies.
Number of comments which are ‘satisfactory’ or
better
Level 2 = 16 + 30 + 20
6. The table shows the number of staff in different = 66
departments of a company. = 82.5%
(b) Yes, it is better to represent the data by a pie chart.
Department Number of staff We will then be able to compare the proportion of
each comment to the whole survey.
Accounting 4
General 10
Marketing 5
Production 17

(a) Name a suitable type of statistical graph which can


show the number of staff in each department.

Chapter 16  Data Handling


212
8. The line graph shows the profits of a company from (a) What was the maximum temperature in the year
2007 to 2011. 2010? When did it occur?
(b) Which was the wettest month in the year 2010?
Profits of the company
How much was the rainfall in that month?
(c) Suggest some ways to improve the readability of
44
the diagram.
(d) What are the advantages of this graph?
Profits ($’000)

42 Solution
(a) The maximum temperature was 33.5 °C and it
40 occurred in February.
(b) The wettest month was July. The rainfall in July
38 was 300 mm.
2007 2008 2009 2010 2011
Year
(c) The graph should have units of measurements on
the two vertical scales. The scales can be read more
(a) Find the percentage increase in the profits from accurately if graph paper is used.
2007 to 2011.
(d) It shows 3 sets of data on one graph. We can
(b) By what percentage is the height of the point
see whether there is any relationship between
representing the profits in 2011 higher than that
temperature and rainfall.
in 2007?
(c) By observing the graphs, Laura said that the profits
in 2011 was three times that in 2007. Do you agree 10. Design a simple survey on the Internet usage of your
with her? Explain your answer. classmates. Present your survey in a report which should
include your questionnaire, the methods used for collecting
Solution and organising data, the statistical diagrams to illustrate
(a) Percentage increase your results, the summary of your findings and the
44 – 40 conclusion.
= × 100%
40
= 10% Solution
Advise students that each survey must have its objective
(b) Required percentage
or goal. Questions may be
3–1
= × 100%
1 (i) two-state
= 200%
(ii) multiple choice, and
(c) Yes, the line graph misleads the readers. The vertical
scale does not start from zero. It seems that the (iii) open-ended types.
profit in 2011 is three times of that in 2007.

Level 3
9. The diagram shows monthly statistics of the maximum
and minimum temperatures, and rainfall recorded at the
climate station located in Changi in the year 2010.

2010 Monthly Statistics (Climate Station)
40 350
38
300
36
34 250

32
200
30
150
28
26 100
24
50
22
20 0
Jan Feb Mar Apr May Jun Jul Aug Sept Oct Nov Dec

Legend: Rainfall Max Temp Min Temp

Source: h ttp://app2.nea.gov.sg/data/cmsresource/
20110524444301448687.pdf

213
Revision Exercise 16 2. Students in a class were asked about their favourite games.
1. The table shows the number of hybrid vehicles in The results of the survey are shown in the bar graph.
Singapore from 2008 to 2010.
Favourite games of students

Year 2008 2009 2010


10
Number of hybrid
1999 2641 3335

Number of students
vehicles

(a) Round off the number of hybrid vehicles in each


5
year correct to the nearest hundred.
The pictogram below represents the data.

Number of hybrid vehicles in Singapore O


Basketball Football Golf Badminton Others

2008 Game

2009 (a) How many students are there in the class?


(b) Which game is the most popular among the
2010 students?
(c) If a pie chart is used to represent the data, what is
represents n vehicles the angle of the sector representing Badminton?

(b) What is the number of vehicles represented by each Solution


car symbol in the pictogram? (a) Number of students in the class
(c) Can we represent the data by a line graph? Explain = 6 + 10 + 2 + 6 + 5
briefly. = 29
(b) Football is the most popular game.
Solution
(a) Number of cars in 2008 (c) Angle of sector representing badminton
6
= 1999 = × 360°
29
= 2000  (correct to the nearest hundred)
= 74.5° (correct to 3 s.f.)
Number of cars in 2009
= 2641
= 2600  (correct to the nearest hundred) 3. The bar graph shows the number of students who
participated in the club activities of a school.
Number of cars in 2010
= 3335 Participation in extracurricular activities
= 3300  (correct to the nearest hundred) Female
Male
Number of students

(b) Number of vehicles represented by each car 30


2000
=
5
20
= 400
(b) Yes, the data can be represented by a line graph. 10
It is because the date is related to time.
0
Chess Drama Dance Swimming
Club

(a) Find the ratio of the number of male students to


that of female students in the chess club.
(b) Which club is the most popular among all the
students?
(c) Which club has equal numbers of male and female
students?
(d) Find the total number of students in the four clubs.

Chapter 16  Data Handling


214
Solution (a) Find the percentage of interviewees who do not
(a) In the chess club, favour the issue.
(b) If a pie chart is drawn for the data, find the angle
number of number of
at centre of the sector representing the opinion “No
male students : female students
idea”.
= 12 : 4
(c) Explain why a pie chart should be preferred to a
= 3 : 1
bar graph to represent this data.
(b) The swimming club was the most popular.
Solution
(c) The dance club had equal numbers of male and
(a) Number of responses which are not favourable
female students.
150
= × 100%
(d) Total number of students = 16 + 22 + 20 + 27 250
= 85 = 60%
(b) Angle representing "No idea"
4. The pie chart shows the amount of fat, carbohydrate and 20
= × 360°
protein contained in per 100 g of egg. 250
= 28.8°
Fat (c) A pie chart can show the proportion of each type
of opinion.

Carbohydrate
6. The pie chart shows the nature of jobs of the staff in
Protein a large company. There are 105 technical staff in the
company.
Amount of fat, carbohydrate
and protein contained in per 100 g of egg Clerical
Administrative
(a) Find the percentage of carbohydrate in per 100 g
Professional
of egg.
(b) If there are 10 g of fat, find the mass of protein.
(c) Is it better to represent the data by a pie chart or a Technical
bar graph? Explain briefly. Public relation

Solution Nature of jobs in a company


(a) Percentage of carbohydrate in per 100 g of egg
360 – 150 – 195 (a) Find the value of x.
= × 100%
360 (b) Find the total number of staff.
= 4.17% (correct to 3 s.f.) (c) Find the number of staff who are professionals.
(b) Mass of protein (d) If 20% of the clerical staff are laid off, find the
10 number of clerical staff remaining.
= 150
× 195
= 13 g Solution
(a) 2x° + x° + 36° + 126° + 36° = 360°
(c) It is better to represent by a pie chart. It can show 3x = 162
the proportion of each constituent more clearly. x = 54
126°
(b) Total number of staff = 105 ÷ = 300
5. The following table shows the results of a survey of the 360°
opinions of 250 people on an issue. (c) Number of staff who are professionals
54°
Opinion Number of responses = 300 × = 45
360°

Strongly agree 38 (d) Original number of clercial staff = 45 × 2


= 90
Agree 42 Number of clercial staff remaining
No idea 20 = 90 × (100% – 20%)
= 72
Disagree 79
Strongly disagree 71

215
7. The line graph shows the exchange rate between (a) The girl’s height was 143 cm when she was
Singapore dollars (SGD) and the US dollars (USD) in 11 years old. Her height increased rapidly in the
SGD/USD at the end of each year from 2002 to 2010. next two years. Then from 13 to 14 years old, she
grew by another 2 cm. At 15 years old, her height
Exchange rate between Singapore and US dollars
was 157 cm and it remained the same for the next
2.0 two years.

1.8 (b) The girl’s height at age 18 is estimated to be


157 cm.
SGD/USD

1.6
(c) m = 45 + 0.9(h – 150)
1.4
When h = 157,
1.2 m = 45 + 0.9 × (157 – 150)
1 = 51.3

O 2002 2003 2004 2005 2006 2007 2008 2009 2010 2011
Year 9. The bar graph shows the number of friends of each of 4
students in a social networking website.
(a) Describe the general trend of the exchange rate.
Number of friends in a social networking website
(b) Find, correct to the nearest integer, the percentage
change in the exchange rate from the end of 2002
to the end of 2010. 120
(c) Based on the graph, estimate the exchange rate at
the end of the year 2011.

Number of friends
100

Solution
80
(a) The general trend is decreasing. This means the US
dollar is weakening with respect to the Singapore
60
dollar.

(b) Percentage change 40
Alan Bowie Carol Doris
1.3 – 1.75
= × 100% Student
1.75
= –25.7% (correct to 3 s.f.)
(a) Find the ratio of the number of friends of Alan to
(c) The estimated exchange rate at the end of 2011 is that of Doris.
1.24 SGD/USD. (b) Find the ratio of the height of the bar representing
the number of friends of Alan to that of Doris.
8. The line graph shows the heights of a girl at different (c) By observing the graph, Doris said that her number
ages. of friends in the social networking website was
(a) Describe the growth in height of the girl. twice as many as Alan. Do you agree with her?
(b) Based on the graph, estimate the girl’s height when Explain your answer.
she is 18 years old.
(c) A doctor proposed that the ideal mass, m kg, of a Solution
girl with height, h cm, where h  150, is given by (a) Required ratio
the formula m = 45 + 0.9(h – 150). Find the ideal = 80 : 120
mass of the girl when she is 17 years old. =2:3
(b) Required ratio
Solution = 2 : 4
= 1 : 2
Heights of a girl at different ages

(c) Yes, the graph misleads the readers. The vertical
160
scale does not start from zero. The height of the
rectangle is not proportional to the number of
Height (cm)

friends.
150

140

0
11 12 13 14 15 16 17 18
Age (Year)

Chapter 16  Data Handling


216
Review Exercise 4 (c) Area of ABCD = 9 × 5
1. Solve the inequality 3x , 17 and represent the solution = 45 cm2
20
on a number line. The required percentage = × 100%
45
4
Solution = 44 %
9
3x , 17
1
x , × 17 4. In the figure, AB = AD = 2 m and BC = 3 m. Find the
3
17 area of ABCD C
x , (a) in m2,
3
2
(b) in cm .
Solution D
3
0 17
2
3

2. Find the negative integers which satisfy the inequality A 2 B


–4x < 20. Solution
1
(a) Area of ABCD = × (2 + 3) × 2
Solution 2

–4x < 20 = 5 m2

x > – × 20
1 (b) Area of ABCD = 5 × 10 000 cm2
4 = 50 000 cm2
x > –5
The required negative integers are
–1, –2, –3, –4 and –5. 5. In the figure, the shape is formed by 3 semicircles, where
AB = BC = CD = t cm.

3. In the figure, ABCD is a rectangle where AB = 9 cm,


BC = 5 cm, AE = CF = 4 cm and AF = EC = 5 2 cm.
Find
A C D
(a) the area of AECF, B
(b) the perpendicular distance from F to EC,
(c) the area of AECF as the percentage of the area of
ABCD. (a) Express the area of the shape in terms of r
D F 4 C and t.
(b) If the area of the shape is 20 cm2, find, correct to
5 2 3 significant figures,
5 (i) the value of t,
5 2 (ii) the perimeter of the shape.

A 4 E B Solution
9 (a) Area of the shape
2 2
3t t 1
Solution = r× + rt 2 – r × ×
2 2 2
(a) AECF is a parallelogram.
r 9t 2 t2
Area of AECF = 4 × 5 = × + t2 –
2 4 4
= 20 cm2 3r
= t 2 cm2
(b) Let h cm be the perpendicular distance from 2
F to EC. 3r
(b) (i) t 2 = 20
5 2 × h = 20 2
1 40
h = 20 × t2 = 3r
5 2
4 t= 40
= 3r
2
= 2.83  (correct to 3 s.f.) t = 2.06  (correct to 3 s.f.)
\  The required distance is 2.83 cm.

217
(ii) Perimeter of the shape (c) Let r cm be the new radius.
3t t rr 2 × 12 = 72r
=r× +r×t+r× 2
2 r 2 = 6
= 3rt r = 6
= 3r × 40
= 2.45  (correct to 3 s.f.)
3r
= 19.4 cm  (correct to 3 s.f.) \  The new radius is 2.45 cm.

6. The figure shows a L-shaped prism. AB = 10 cm, 8. The pictogram shows the mean sulphur dioxide level in
BC = 3 cm, CD = 6 cm, FA = 8 cm and BG = 13 cm. the air in Singapore from March 2011 to June 2011.
Find
(a) the total surface area of the prism, March 2011
(b) the volume of the prism.
K J April 2011

May 2011
I H
F E June 2011
G
8 6 C represents 2 ng/m3
D 13
3
A 10 B (a) State the mean sulphur dioxide level in April
2011.
(b) In which month was the air most polluted by sulphur
Solution
dioxide during these four months?
(a) Area of the cross-section ABCDEF
=8×4+6×3 Solution
= 50 cm2 (a) Mean sulphur dioxide level in April 2011
Perimeter of ABCDEF 1
= 10 + 3 + 6 + 5 + 4 + 8 =2×4
2
= 36 cm = 9 µg/m3
\  Total surface area of the prism
= 36 × 13 + 2 × 50 (b) It is most polluted in May.
= 568 cm2
(b) Volume of the prism 9. The bar graph shows the sales of magazines A, B and C
= 50 × 13 in a bookstall on a day.
= 650 cm3 Sales of magazines on a day

7. A solid metal cylinder has a base radius of 3 cm and


height of 8 cm. 15
Sales (copies)

(a) Find the total surface area of the cylinder, in terms


of r. 10
(b) Find the volume of the cylinder, in terms of r.
(c) If the solid is melted and recast into a solid cylinder
5
of height 12 cm, find the base radius of the new
cylinder.
0
A B C
Solution Magazines
(a) Total surface area
= 2r × 3 × 8 + 2r × 32
(a) Express the total number of copies of magazines
= 66r cm2
B and C sold as a percentage of the total number
= r × 32 × 8
(b) Volume of magazines sold.
= 72r cm3
(b) If the information is represented by a pie
chart, find the angle representing the sales of
magazine A.

Review Exercise 4
218
Solution 11. In the figure, ABCD is a trapezium and AECD is
(a) Total number of magazines sold a parallelogram. The ratio of the area of AECD to
= 9 + 15 + 12 the area of ABCD is 6 : 11. The area of BCE is
= 36 40 cm2.
The required percentage (a) Find the area of ABCD.
15 + 12 (b) If EB = 10 cm, find the height from D to AB and
= × 100%
36 the length of AE.
= 75%
(b) The required angle D C
9
= 360° ×
36
= 90°

10. The line graph shows the total international call minutes A E B
of Singapore from 2006 to 2010.
Solution
Total international call minutes of Singapore (a) Let 11x cm2 be the area of ABCD.
Then the area of AECD = 6x cm2
11x – 6x = 40
14 000 5x = 40
x = 8
12 000 \  Area of ABCD = 11 × 8
Total international call minutes (million)

= 88 cm2
10 000 (b) (i) Let h cm be the height from D to AB.
1
× 10 × h = 40

2
8000
5h = 40
h = 8
6000 \  The height is 8 cm.
(ii) Let y cm be the length of AE.
4000 By considering the area of AECD,
y × 8 = 88 – 40
2000 8y = 48
y = 6
\  AE = 6 cm.
O 2006 2007 2008 2009 2010 2011
Year 12. The figure shows the cross-section of a prism of length
15 cm. It is a semicircle of diameter 20 cm surmounted
(a) Describe the general trend of the total international on a rectangle of width 20 cm and height h cm. Given
call minutes from Singapore from 2006 to 2010. that h is the smallest integer such that the area of the
(b) Find the total international call minutes from rectangle is larger than the area of the semicircle, find,
Singapore in 2009 from the graph. in terms of r,
(c) Estimate the total international call minutes from (a) the value of h,
Singapore in 2011 from the graph. (b) the total surface area of the prism,
(c) the volume of the prism.
Solution
(a) The general trend is increasing from 2006 to 2010.
(b) The reading = 10 000 million minutes.
(c) The estimated total international call from Singapore
in 2011 is 14 400 million minutes.
h

20

219
Solution (b) The time taken
(a) Area of the rectangle = Area of the semicircle = 37500r ÷ 2000
1 = 58.9 s  (correct to 3 s.f.)
20 × h = × r × 102
2
(c) Let h cm be the length of the trough.
20h = 50r
Area of PQRS
5r
\  h = 1
2 = × (60 + 18) × 20
2
(b) Area of the cross-section = 780 cm2
= r × 102 780 × h = 37500r
= 100r cm2 h = 151.0
Perimeter of the cross-section \  h = 151  (correct to 3 s.f.)
5r \  The length of the trough is 151 cm.
= 20 + 2 × + r × 10
2
(d) External surface area of the trough
= (20 + 15r) cm
= (18 + 29 + 29) × 151.0 + 2 × 780
\  Total surface area = 13 000 cm2  (correct to 3 s.f.)
= (20 + 15r) × 15 + 2 × 100r
= 300 + 225r + 200r
14. The line graphs below show the profits of two companies,
= (300 + 425r) cm2
A and B, from 2010 to 2012.
(c) Volume of prism
= 100r × 15 Profits of company A
= 1500r cm3

13. Figure A shows a cylindrical water tank of base radius 4.2


Profits (million $)
25 cm and height 60 cm. Figure B shows a trough which
is a prism whose cross-section is a trapezium PQRS with 4.1
PQ = RS = 29 cm, PS = 60 cm, QR = 18 cm and the
depth is 20 cm.
4.0

3.9

60
60 P S 0 2010 2011 2012
29 20 29
25 Year
Q 18 R

Figure A Figure B
Profits of company B
The tank is full of water. The water is then drained from
the tank to the trough at the rate of 2000 cm3 per second
and it just completely fills the trough.
7
(a) Express the initial volume of water in the tank in
Profits (million $)

(i) cm3, (ii) m3.


(b) What is the time taken to fill the trough? 6
(c) Find the length of the trough.
(d) Find the external surface area of the trough. 5
Give your answers correct to 3 significant figures.
4
Solution
(a) (i) Initial volume
= r × 252 × 60 0 2010 2011 2012
= 37500r
Year
= 117 809.7
= 118 000 cm3  (correct to 3 s.f.)
(a) Find the percentage change in the profits of
(ii) Initial volume company A
118 000 (i) from 2010 to 2011,
= m3
1 000 000
(ii) from 2010 to 2012.
= 0.118 m3  (correct to 3 s.f.)

Review Exercise 4
220
(b) Find the percentage increase in the profits of (b) The area of the HDB flat increases as the household
company B from 2010 to 2012. size increases.
(c) Explain why the line graphs give the impression (c) The average household size decreases for all
that the financial performance of company A is different types of dwelling from the year 2000 to
better than that of company B. 2010.

Solution
(a) (i) Percentage change
3.96 – 4.0
= × 100%
4.0
= –1%
(ii) Percentage change
4.2 – 4.0
= × 100%
4.0
= 5%
(b) Percentage increase
6–5
= × 100%
5
= 20%
(c) The vertical scales of two graphs are different. The
graph of company A exaggerates the change.

15. The bar graph shows the average household size by the
type of dwelling in the years 2000 and 2010.
Average household size by type of dwelling

2000
5 4.7
2010
4.4
Household size (persons)

4.1 4.0
4 3.9
3.7 3.7
3.4
3.1
3 2.8
2.2 2.1
2

0 HDB HDB HDB HDB Condo Landed


1- & 2- 3-Room 4-Room 5-Room & & Properties
Room Executive Private
Flats
Types of dwelling

(a) Find the percentage change in the average household


size of HDB 3-room flats from 2000 to 2010.
(b) What is the relationship between the household
size and the types of dwelling for the HDB flats?
(c) Describe the general change in the average
household size from 2000 to 2010.

Solution
(a) Percentage change
2.8 – 3.1
= × 100%
3.1
= –9.86%  (correct to 3 s.f.)

221
Problems in Real-world Contexts
A. Rate and Discount in a Supermarket
The advertisement on the right shows the price of fresh tiger jumbo prawns in a promotion.
1. What is the promotional price of the tiger prawns in dollars per kg?
2. Find the percentage discount in this promotion, giving your answer correct to 3
significant figures.
3. Sally bought 1.5 kg of the prawns. How much did she save from the usual price?

Solution
1. Promotional price in dollars per kg
= $3.68 per 100 g
= $36.80 per 1000 g
= $36.80 per kg
2. Percentage discount
4.29 – 3.68
= × 100%
4.29
= 14.2%  (correct to 3 s.f.)
3. Amount of savings
= $1.5(4.29 – 3.68)
= $0.92  (correct to 3 s.f.)

B. Exchange Rate
The following table shows the currency exchange rates at 5:08 p.m. on 9th April 2012.

Source: http://sg.finance.yahoo.com/currencies/investing.html#cross-rates

1. Write down two statements about the exchange rate between Singapore dollars (SGD) and Euros (EUR) from the
table.
2. A cell of the table, as shown above with a  , has been accidentally masked by a correction tape. Could you
find out its value? Give your answer correct to 4 decimal places.
3. Su Ling went for a tour in Australia. If she exchanged 3000 Singapore dollars, how many Australian dollars would
she get?
4. Ramesh exchanges 1000 Singapore dollars to a currency A, then to another currency B, and finally exchanging the
money back to Singapore dollars according to the exchange rates in the table above. He thinks that he will get back
1000 Singapore dollars. Do you agree with him? Justify your answer.

Solution
1. 1 EUR = 1.6488 SGD
1 SGD = 0.6065 EUR
2. 1 USD = 1.2610 SGD
1 SGD = 0.7930 (correct to 4 d.p.)
The value in the cell is 0.7930.

Problems in Real-world Contexts


222
3. 1 SGD = 0.7708 AUD
3000 SGD = 3000(0.7708) AUD
= 2312.40 AUD
4. Assume Ramesh exchanges 1000 SGD to GBP, then to JPY and back to SGD.
1000 SGD = 1000(0.4995) GBP
= 499.50 GBP
= 499.50(129.0527) JPY
= 64 461.82 JPY
= 64 461.82(0.0155) SGD
= 999.16 SGD
Ramesh will not get back 1000 SGD after exchanging currency twice.

C. Oversupply of Shoebox Units


A shoebox unit is an apartment with a land area of 500 square feet (approximately 46 m2) or less. In the year 2011,
shoebox units were one of the most popular property types in Singapore. The bar graph on the right shows the current
and projected numbers of completed shoebox units in the years to come.

Source: The Straits Times, April 19, 2012

1. At the end of 2012, what is the number of completed shoebox units in the suburban area? Calculate this as a
percentage of the total number of completed units in the same year.
2. At the end of which year do you see the largest increase in the number of completed shoebox units?
3. (a) Describe the trend of the number of completed shoebox units from the end of 2011 to the end of 2015.
(b) Could you predict the number of completed shoebox units at the end of year 2016? Explain how you arrive
at your prediction.

Solution
1. Number of completed shoebox units at the end of 2012 = 900
900
Required percentage = × 100%
4100
= 22.0% (correct to 3 s.f.)

223
2.
End 2012 End 2013 End 2014 End 2015
Increase 4100 – 2400 = 1700 5400 – 4100 = 1300 6800 – 5400 = 1400 8200 – 6800 = 1400
1700 1300 1400 1400
Percentage × 100% × 100% × 100% × 100%
2400 4100 5400 6800
Increase
= 70.8% = 31.7% = 25.9% = 20.6%

The largest increase occurred at the end of 2012.

3. (a) There is an increase in the number of shoebox units. The largest increase was at the end of 2012 and from the
end of 2013 onwards, the number of shoebox units decreased.
(b) Increment from end 2013 to end 2014 = 1400
Increment from end 2014 to end 2015 = 1400
We may estimate that the increment from end 2015 to end 2016 to be 1400.
\ estimated number of completed shoebox units at the end of 2016 = 9600

D. Paper Clip
The diagram shows a paper clip which is made from a circular wire of diameter 1 mm. The dimensions are in centimetres.
The clip consists of 3 semicircles and 4 straight edges.

Suppose you are the manufacturer of the paper clip and you would like to know more about the material required to make
the clip.
1. Calculate the total length of the wire needed to make a clip. State the assumptions you have to make in your
calculation.
2. What is the volume (in cm3) of the material used for making each clip?
3. The clips are packed in a box of 200 pieces. It is known that the density of the material is 7.6 grams/cm3. Find the
total mass of the clips in a box.
4. The wire of the clip is then bent into the following shapes:
(i) a square
(ii) a rectangle
(iii) an equilateral triangle
(iv) a circle
(a) Find a possible set of dimension(s) for each shape.
(b) Find the area of each shape.
(c) Given the same perimeter (in this case, the length of clip is fixed), which shape gives the largest area?

Solution
1. Assume that all 3 semicircles are of diameter 1 cm.
Total length of wire = 3 × r × 0.5 + 4 + 3 + 3 + 2
= 16.71  (correct to 4 s.f.)
= 16.7 cm  (correct to 3 s.f.)
2. Volume = r × 0.052 × 16.71
= 0.131 cm3  (correct to 4 s.f.)
= 0.13 cm3  (correct to 3 s.f.)
3. Total mass = 7.6 × 0.13 × 200
= 197.6 g  (correct to 3 s.f.)

Problems in Real-world Contexts


224
2.
Square Rectangle Equilateral triangle Circle
Length = 16.71 ÷ 4 Length + Breadth Length = 16.71 ÷ 3 2rr = 16.71
= 4.1775 = 16.71 ÷ 2 = 5.57 cm r = 2.66 cm
= 4.18 = 8.355
(a)
\ 3.36 cm by 5 cm
Dimension(s)
4.36 cm by 4 cm
5.36 cm by 3 cm
etc.
4.17752 = 17.5 cm2 3.36 × 5 = 16.8 cm2 (5.57)2 3 rr 2 = r(2.659)2
(b) = 13.4 cm2
4.36 × 4 = 17.4 cm2 4 = 22.2 cm2
Area
5.36 × 3 = 16.08 cm2

(c) From the table above, the circle gives the largest area.

E. Medals Won in 2008 Summer Olympic Games in Beijing


The following table shows the number of medals won by the top 10 countries.

Rank Country Gold Silver Bronze


1 China 51 21 28
2 United States 36 38 36
3 Russia 23 21 28
4 Great Britain 19 13 15
5 Germany 16 10 15
6 Australia 14 15 17
7 South Korea 13 10 8
8 Japan 9 6 10
9 Italy 8 10 10
10 France 7 16 17

1. Represent the data above using a graph, including the gold, silver and bronze medals won by the countries.
Hint: You may use a spreadsheet program to help you choose the appropriate graph.
2. How would you compare the results?
3. Besides ranking by gold medals, what other ways would you suggest ranking the countries? Will the ranking be
different? Which way of ranking do you think is most reasonable? Explain your answer.

Solution

Medals won in 2008 Summer Olympic Games


Gold Silver Bronze
60

50

40

30

20

10

0
ina tes ssi
a in any ali
a rea an ly nce
Ch Sta Ru rita rm str Ko Jap Ita Fra
i t ed ea tB Ge Au u th
Un Gr So

225
2. The countries may be ranked by the number of gold medals won.
3. Since countries may rank differently for each type of medal, ranking countries by the total number of medals is most
reasonable.

F. Prepaid Mobile Phone Plans


The charges for prepaid mobile phone plans offered by two communications companies are as follows.

Local calls will be charged at 16 cents per minute during day time and 8 cents per minute during
Company A
night time.

Local calls will be charged at 22 cents for the first minute and 8 cents per minute thereafter for the
Company B
whole day.

Your friend from the United States is visiting Singapore for 7 days. He would like you to suggest a phone plan to purchase
during his visit.

1. What would you need to consider when helping your friend decide on which phone plan to purchase?
2. Assume that he makes 10 phone calls in the day and 5 phone calls at night for each day, and each call lasts about
3 minutes. Calculate his daily call charges per day for each plan.
3. In general, he makes x phone calls in the day and y phone calls at night for each day, and each phone call lasts about
t minutes. Express the daily charges in terms of x, y and t for each plan.
4. Use the two formulae obtained in Question 3 to decide on which phone plan to purchase. Justify your decision made.

Solution
1. Company A Company B
Daily call charge = $ 0.16 × 10 × 3 + $0.08 × 5 × 3 Daily call charge = $0.22 × 15 + $0.08 × 15 × 2
= $4.80 + $1.20 = $3.30 + $2.40
= $6.00 = $5.70

2. Company A Company B
Daily call charge = $0.16 × x × t + $0.08 × y × t Daily call charge = $0.22 × (x + y) +
= $0.16xt + $0.08yt $0.08 × (x + y) × (t – 1)
= $0.14x + $0.14y +
$0.08xt + $0.08yt

4. Assume the caller does not make any calls during the day, i.e. x = 0.
Company A: $0.08yt
Company B: $0.14y + $0.08yt
Company A offers a better phone plan.

Assume the caller does not make any calls during the night, i.e. y = 0.
Company A: $0.16xt
Company B: $0.14x + $0.08xt
= x(0.16t – 0.14 – 0.08t)
Difference
= x(0.08t – 0.14)
If 0.08t – 0.14 = 0,
0.08t = 0.14
t = 1.75

If the phone call lasts for less than 1.75 min, Company A offers a better phone plan. If the phone call lasts for more
than 1.75 min, Company B offers a better phone plan.

Problems in Real-world Contexts


226
G. Population of Singapore
Population statistics provides useful information for the government to determine new development programs as well as
long term plans for education, health care, community facilities services, etc. In some cases, we may need to predict the
population in order to proceed with the long term plans.

Year Population
2001 4 138 000
2002 4 176 000
2003 4 114 800
2004 4 166 700
2005 4 265 800
2006 4 401 400
2007 4 588 600
2008 4 839 400
2009 4 987 600
2010 5 076 700

The table above shows the population of Singapore from 2001 to 2010.
1. Represent the data above using a graph.
Hint: You may use a spreadsheet program to help you plot and draw the graph.
2. Describe the trend of population growth of Singapore over these 10 years.
3. Based on your description in Question 2, estimate the population in the year 2011.
4. Could you predict the population of Singapore in the year 2020 using the data above? What are some possible events
or happenings you would need to consider as part of your assumptions when you make your prediction?

Solution
1.

Population of Singapore
6000000
5500000
5000000
4500000
4000000
Population

3500000
3000000
2500000
2000000
1500000
1000000
500000

0
2001 2002 2003 2004 2005 2006 2007 2008 2009 2010 2011

2. The graph shows that the population of Singapore increased quite steadily from 2001 to 2011 except for a decrease
from 2002 to 2003. The rate of increase was faster from 2007 to 2008.
3. We can project the line graph to the year 2011 to get an estimate as shown in the graph.
\  estimate population in 2011 = 5.2 million
4. The population for the year 2020 can be predicted if the rate of population increase remains the same from 2010 to
2020

227
H. Fund Raising
Your school is staging a carnival to raise funds on a Saturday from 10 a.m. to 5 p.m. . Your class has decided to set up a
stall to sell hamburgers. You are tasked to come up with a plan for the number of hamburgers to be sold and the price of
each hamburger. Your target is to raise as much money as possible for the school.

The following are points that you may consider to help you in your planning:

• W here and how are you going to purchase your hamburgers (bread and filling)? How much money would you
need? You may try to get somebody to sponsor your bread and filling.
• How many hamburgers do you expect to sell per hour? How many hamburgers would you need to prepare during
the seven hours of the carnival?
• You may have a target amount of money you would like to raise. After considering the expected cost prices that
you would incur, would you be able to achieve your target?

1. What other information would you need to know or consider?


2. What are the assumptions made?
3. What would be your plan? Explain your method and solution.
4. Discuss with your classmates whether your plan makes sense. Are there other considerations that may affect the
amount of money raised?

Solution
1. Other information to consider would be
• the costs of the bread and filling
• how much each burger costs to make
• the price of each burger to be sold
• target amount of money to be raised
• the estimated number of people coming for the carnival
• the number of people needed to prepare the burger
• giving discounts
2. Assume that all burgers would be sold.
3. Answer depends on method chosen.
4. Answer depends on method chosen.

Problems in Real-world Contexts


228

Você também pode gostar